Transcript
Page 1: Exercices équations fonctionnelles

1

Equations fonctionnelles : exercices corrigés.

1. Equations ensemblistes.

2. Equations dans N, Z, Q.

3. L’équation de Cauchy.

4. Logarithmes et exponentielles.

5. Equations récursives.

6. Equations intégro-différentielles.

7. Fonctions eulériennes.

8. Farrago final.

à L. G. Vidiani , Pierre-Jean Hormière ____________

Une équation fonctionnelle est une équation dont l’inconnue est une fonction. La résoudre, c’est trouver toutes les fonctions vérifiant certaines relations algébriques, assorties parfois de conditions supplémentaires : monotonie, continuité, dérivabilité, etc. En algèbre, les équations fonctionnelles les plus simples sont la recherche des endomorphismes ou des automorphismes d’un monoïde, d’un groupe, d’un anneau, etc., et la factorisation d’un morphisme à travers un autre ; ces sujets sont évoquées dans chacun des chapitres correspondants. En analyse, les équations différentielles et intégrales sont les plus importantes équations fonctionnelles. Les premières sont traitées dans un chapitre ad hoc, les secondes dans des ouvrages spécialisés. On s’intéresse ici aux autres équations fonctionnelles. La plupart se ramènent à des équations différentielles, par diverses méthodes (Euler, etc.). Dans d’autres cas, on construit la fonction par extensions successives du domaine de définition, notamment par densité. D’autres, enfin, relèvent d’approches récursives, qui sont évoquées ici sans épuiser le sujet : toute propriété satisfaite par une fonction donne lieu à une équation fonctionnelle.

Voici des méthodes simples pour attaquer la résolution d’une équation fonctionnelle d’inconnue f :

• Chercher ou deviner des solutions : fonctions constantes, affines, monômes ou polynômes, expo-nentielles, trigonométriques, exponentielles-polynômes, etc. Si elles existent, les fonctions cherchées sont presque toujours des fonctions classiques ! • Calculer des valeurs particulières de f : f(0), f(1), f(−1), etc. Elles peuvent suggérer une discussion.

• Etudier les propriétés de f : injectivité, surjectivité, bijectivité, parité, monotonie, signe, périodicité, continuité, dérivabilité, etc. Il faut parfois aussi considérer les itérées de f : fof, etc., ainsi que ses points fixes.

• Prêter attention à la nature de l’ensemble de définition : − N, Z et Q suggèrent des récurrences, des méthodes arithmétiques, additives ou multiplicatives. − R suggère des méthodes de monotonie, de continuité, de densité, des équations différentielles, etc.

• Prêter attention à la structure de l’ensemble des solutions : si l’équation est linéaire (resp. affine), les fonctions f cherchées forment un espace vectoriel (resp. affine). • Exploiter les symétries, les dissymétries, les particularités, de l’équation fonctionnelle.

• Se ramener à des équations fonctionnelles classiques par diverses techniques : changer de variable ou de fonction inconnue, utiliser les mêmes méthodes, etc.

• Ne pas oublier les réciproques.

De façon plus abstraite, une solution d’une équation fonctionnelle est presque toujours un point fixe d’un opérateur fonctionnel. Ainsi, la factorielle est l’unique fonction f : N → N vérifiant f(0) = 1 et (∀n ∈ N*) f(n) = n.f(n−1). Mais c’est aussi l’unique fonction telle que f = T(f), où T : g ∈ FFFF(N, N) → h ∈ FFFF(N, N), h définie par h(0) = 1 et (∀n ∈ N*) h(n) = n.g(n−1).

Page 2: Exercices équations fonctionnelles

2

Les Olympiades et le Concours général de mathématiques proposent souvent des équations fonctionnelles aux énoncés simples, dont la résolution requiert de l’ingéniosité, mais aussi de l’entraînement et une bonne culture. Les oraux des Concours d’entrée aux écoles d’ingénieurs sont davantage axés sur le programme d’analyse de taupe.

Ce chapitre est inachevé _________

1. Equations fonctionnelles ensemblistes.

Exercice 1 : involutions. Une application s : E → E est dite involutive si s o s = idE . Soient A, B, C trois parties disjointes de réunion E, f : B → C une bijection. Montrer que la recollée

de f , f−1

et idA est une involution de E, et que l’on obtient ainsi toutes les involutions de E. Exemple : Trouver toutes les involutions de {1, 2, 3, 4, 5}.

Solution :

• Si X et X’ sont des ensembles disjoints, f : X → Y et g : X’ → Y’ deux applications, il y a une seule application h : X ∪ X’ → Y ∪ Y’ prolongeant f et g. On l’appelle recollée de f et g et on la note ici h = f ⊕ g. Si de plus Y et Y’ sont disjoints et si f et g sont bijectives, il en est de même de h,

et l’on a h−1

= f−1

⊕ g−1

.

• La recollée s = idA ⊕ f ⊕ f−1

est une bijection de E = A ∪ B ∪ C sur E = A ∪ C ∪ B. Elle est égale à la bijection réciproque, donc elle est involutive.

• Réciproquement, soit s une involution de E, A l’ensemble des points fixes de s. Supposons A ≠ E. Pour tout x∈E−A, P(x) = {x, s(x)} est une paire incluse dans E−A, et deux tels ensembles P(x) et P(y) sont, soit égaux, soit disjoints. Ainsi, (P(x)) définit une partition de E−A. Choisissons un élément x dans chacune des paires, et notons B l’ensemble formé. Alors (A, B, C = s(B)) est une

partition de E et s est la recollée de idA , f = CBs et f

−1 .

• Il y a 26 involutions de E = {1, …, 5} : l’identité, 25C = 10 transpositions, et 15 bitranspositions.

A a 1, 3 ou 5 éléments. Si A = {a}, il y a 3 paires {B, C} partitionnant E − {a} : en tout 5.3 = 15 cas. Exercice 2 : factorisation d’applications.

1) Soient f : E → F et g : E → G deux applications. On dit que g se factorise à gauche à travers f s’il existe une application h : F → G telle que g = h o f. a) Montrer que, pour que g se factorise à gauche à travers f, il faut et il suffit que : ∀(x, x’) ∈ E×E f(x) = f(x’) ⇒ g(x) = g(x’). b) Montrer que, si f est surjective, l’application h est unique et donnée par h = g o s, où s est une section de f. c) Montrer l’équivalence f injective ⇔ f inversible à gauche.

2) Soient f : F → E et g : G → E deux applications. On dit que g se factorise à droite à travers f s’il existe une application h : G → F telle que g = f o h. a) Montrer que, pour que g se factorise à droite à travers f, il faut et il suffit que g(G) ⊂ f(F). b) Montrer que, si f est injective, l’application h est unique et donnée par h = r o g, où r est une rétraction de f. c) Montrer l’équivalence f surjective ⇔ f inversible à droite.

3) Soient a : G → H, b : E → F, v : E → H trois applications. Montrer l’équivalence des propriétés suivantes : i) Il existe une application u : F → G telle que v = a o u o b ;

ii) v(E) ⊂ a(G) et ∀(x, x’) ∈ E2 b(x) = b(x’) ⇒ v(x) = v(x’).

Solution : 1) a) Si g = h o f, alors f(x) = f(x’) ⇒ h(f(x)) = h(f(x’)) ⇒ g(x) = g(x’).

Page 3: Exercices équations fonctionnelles

3

Réciproquement, si f(x) = f(x’) ⇒ g(x) = g(x’), cherchons h telle que g = h o f. Soit y ∈ f(E). Il existe x ∈ E tel que y = f(x). Il est légitime de poser g(x) = h(y), car g(x) ne dépend pas du x choisi. h est bien définie de f(E) dans G. Il suffit de la prolonger à F.

2) a) Si g = f o h, il est clair que g(G) ⊂ f(F). Réciproquement, si g(G) ⊂ f(F), cherchons h telle que g = f o h. Pour tout z ∈ G il existe y ∈ F tel que g(z) = f(y). Pour chaque z, choisissons un tel y, et nommons-le h(z). Alors g(z) = f(h(z)).

Remarques : 1) L’axiome du choix est requis en 2 a). 2) Les versions linéaires de ces énoncés sont traitées dans les problèmes d’algèbre linéaire.

Exercice 3 : Soit f : R*+ → R. Montrer l’équivalence des propriétés suivantes :

i) (∀x > 0) f(x) = f(x1 ) ;

ii) Il existe une fonction u : [1, +∞[ → R telle que (∀x > 0) f(x) = u(21 (x +

x1 )).

(Proposé Olympiades 1987)

Solution : Il s’agit d’un problème de factorisation d’une application à travers une autre (ex 2, 1.a).

ii) ⇒ i) est évident. Montrons i) ⇒ ii). La fonction g(x) = 21 (x +

x1 ) est une surjection de R*+ sur [1,

+∞[, telle que g(x) = g(y) ⇔ y = x ou 1/x.

Soit z∈[1, +∞[. Notons u(z) = f(x), où x est l’une quelconque des racines de l’équation z = g(x). Quelle que soit la racine choisie, la valeur de f(x) sera la même, en vertu de f(x) = f(1/x). La fonction u ainsi définie convient. Et elle est unique.

Si l’on veut une fonction explicite, on notera que x = z ± 1²−z , et l’on pourra poser :

u1(z) = f(z + 1²−z ) ou u2(z) = f(z − 1²−z ). Mais ces deux fonctions sont égales. A noter que si f est continue, u aussi. Exercice 4 : cns pour qu’une permutation soit un carré. On se place dans le groupe SSSSn des permutations de {1, 2, …, n}.

1) Si γ = [x1, x2, …, xq] est un cycle de longueur q, calculer γ2 en distinguant deux cas.

2) Soit σ ∈ Sn . Montrer l’équivalence des propriétés suivantes :

a) ∃τ ∈ Sn σ = τ2 ;

b) Pour tout entier pair 2k ∈ [1, n], le nombre des orbites de σ de longueur 2k est pair.

3) Exemple :

a) Soit σ =

3572681487654321 . Calculer ε(σ) ; σ est-elle un carré dans S8 ?

b) Soit f la permutation

7261085349110987654321 . Trouver une permutation g de {1, …, 10}

telle que g o g = f. Combien y en-a-t-il en tout ?

Solution : 1) Si γ est un cycle de longueur impaire, γ2

est un cycle de même longueur, à savoir :

γ2 = [x1, x3, x5, …, xq, x2, x4, …, xq−1] .

Si γ est de longueur paire q = 2k, γ2 se scinde en deux cycles disjoints de longueur k :

γ2 = [x1, x3, x5, …, xq−1] o [x2, x4, x6, …, xq] .

Page 4: Exercices équations fonctionnelles

4

2) Quand on élève une permutation τ décomposée en cycles à supports disjoints au carré, chaque cycle de longueur impaire donne naissance à un cycle de longueur impaire de même support, et chaque cycle de longueur paire se scinde en deux cycles à support disjoint de longueur moitié.

Du coup, pour tout k, les cycles de longueur 2k de σ = τ2 sont en nombre pair, puisqu’ils proviennent

de cycles de longueur 4k. Réciproquement, si σ possède cette propriété, décomposons-la en cycles disjoints. Les cycles de longueur impaire sont carrés de cycles de même longueur. Les cycles de longueur 2k peuvent être groupés par deux et proviennent par exemple d’un cycle de longueur double. 3) Exemples. a) Si ε(σ) = −1, σ ne peut être un carré, mais la réciproque est fausse. Ainsi, la permutation proposée est telle que ε(σ) = 1, mais σ = [1, 4, 6, 7, 5, 2] o [3, 8] n’est pas un carré. b) f = [2, 9] o [3, 4] o [6, 8] o [7, 10] est le carré de g = [2, 3, 9, 4] o [6, 7, 8, 10] par exemple. Mais aussi de [2, 4, 9, 3] o [6, 7, 8, 10] , [2, 3, 9, 4] o [6, 10, 8, 7] , [2, 4, 9, 7] o [6, 10, 8, 7]. f est aussi le carré de [2, 6, 9, 8] o [3, 7, 4, 10] et des trois autres, comme ci-dessus. et aussi de [2, 7, 9, 10] o [3, 6, 4, 8] et des trois autres. En tout 12 racines carrées. Mais nous ne nous sommes pas occupés de {1, 5}, qui peuvent provenir de [1] o [5] ou de [1, 5]. Je trouve donc 24 racines carrées de f. Je n’en vois pas d’autres.

Exercice 5 : Existe-t-il une fonction f : Z → Z telle que (∀n ∈ Z) ( f o f )(n) = − n ?

Solution : Cet exercice fait suite au précédent, puisqu’on cherche une racine carrée d’une involution de Z. 1) Analyse. Notons que, si f existe, f est impaire, car (f o f o f)(n) = f(−n) = − f(n). Donc f(0) = 0. De plus f

4 = idZ. Tout n ≠ 0 génère un cycle [ n, f(n), − n, − f(n) ] à 4 éléments.

2) Exemple. Posons f(2) = 1 , f(1) = −2 , f(−2) = −1 , f(−1) = 2 , et de même pour 3, 4, −3, −4, etc. Donc : f(0) = 0 , f(2n) = 2n − 1 , f(2n − 1) = − 2n , f(− 2n) = 1 − 2n , f(1 − 2n) = 2n pour tout n ≥ 1. Cette fonction n’est pas la seule. cf. mon problème sur l’équation de Babbage.

2. Equations fonctionnelles dans N, Z, Q.

Exercice 1 : Soit f : N → N strictement croissante. Quelle est la valeur minimale de f(2007) ?

Solution : On montre par récurrence forte que (∀n ∈ N) f(n) ≥ n. La valeur minimale de f(n) est n ; elle correspond au cas où f induit l’identité sur {1, 2, …, n}. Exercice 2 : Soient f : N → N surjective et g : N → N injective telles que, pour tout n∈N, f(n) ≥ g(n). Prouver que f = g. [ Olympiades Roumanie 1986 ]

Solution : Utilisons le principe de descente infinie de Fermat. Supposons qu’il existe un entier n tel que g(n) < f(n) (1). Comme f est surjective, il existerait un entier k tel que g(n) = f(k) (2). Il découle de (1) et (2) que k ≠ n. Comme g est injective, g(k) ≠ g(n) = f(k) (3). (1) et (2) impliquent g(k) ≤ f(k) = g(n) < f(n). De plus, en vertu de (3) g(k) < f(k) = g(n) < f(n). En recommençant indéfiniment, on fabriquerait une suite infinie strictement décroissante d’entiers naturels. Exercice 3 : Trouver toutes les fonctions f : N → N vérifiant

(∀n ∈ N ) f(n) + ( f o f )(n) + ( f o f o f )(n) = 3n. [ Oral X 2001 ]

Page 5: Exercices équations fonctionnelles

5

Solution : 1) L’identité convient visiblement. On va montrer que c’est la seule. 2) f est injective, car f(m) = f(n) ⇒ f(m) + f(f(m)) + f(f(f(m))) = f(n) + f(f(n)) + f(f(f(n))) ⇒ 3m = 3n. 3) On a f(0) + f(f(0)) + f(f(f(0))) = 0, donc f(0) = 0.

4) Montrons par récurrence forte que f(n) = n. Supposons f(k) = k pour 0 ≤ k ≤ n. L’injectivité de f implique que f(n + 1), f(f(n + 1)) et f(f(f(n + 1))) sont tous trois ≥ n + 1. Par exemple f(f(n + 1)) = k < n impliquerait f(f(n + 1)) = f(k), donc f(n + 1) = f(k), donc n + 1 = k ! Mais alors f (n + 1) + f (f (n + 1)) + f (f (f (n + 1))) = 3(n + 1) implique f(n + 1) = n + 1. Exercice 4 : Le code du code. Pour noter ses mots de passe numériques, Bob a décidé d’inventer un code secret. Chaque nombre entier positif n est codé par un autre entier, supérieur ou égal à n, qui sera désigné dans la suite par « code de n ». Si a est plus grand que b, le code de a sera supérieur au code de b. Mais ce n’est pas tout. Bob souhaite que pour tout nombre n, le code du nombre entier « code de n » soit égal à 3n. Un tel code existe-t-il ? Si oui, quelles sont les valeurs possibles du code de 100 ?

Solution : [ Le monde, Affaire de logique, 30 mars 2016 ]. On cherche une fonction C : N → N vérifiant : i) ∀n ∈ N n ≤ C(n) ; ii) ∀a, b b ≤ a ⇒ C(b) ≤ C(a) ; iii) ∀n ∈ N (C o C)(n) = 3n. Nous allons montrer qu’une telle fonction existe et est unique. 1) Je dis que C(0) = 0. En effet, 0 ≤ C(0) ≤ C(C(0)) = 0. 2) C n’a pas d’autre point fixe, car n = C(n) ⇒ n = C(n) = C(C(n)) = 3n ⇒ n = 0. Donc ∀n ≥ 1 n < C(n). 3) Il découle de iii) que C est injective, car C(m) = C(n) ⇒ C(C(m)) = C(C(n)) ⇒ 3m = 3n ⇒ m = n. 4) Il en résulte que C est strictement croissante. 5) Il découle de iii) que (C o C o C)(n) = 3C(n) = C(3n). 6) Je dis que C(1) = 2 et C(2) = 3. En effet, en vertu de 2), 1 < C(1) < C(C(1)) = 3 implique C(1) = 2. Du coup C(2) = C(C(1)) = 3. 7) Par récurrence, C(3

k) = 3

k.C(1) = 2.3

k et C(2.3

k) = 3

k.C(2) = 3.3

k .

8) Supposons 3k ≤ n < 2.3

k , autrement dit n = 3

k + q , 0 ≤ q < 3

k .

Alors C(3k) = 2.3

k ≤ C(n) < C(2.3

k) = 3.3

k.

Comme les intervalles [3k, 2.3

k[ et [2.3k, 3.3

k[ ont même nombre d’éléments et C est strictement

croissante, C(n) = 2.3k + q .

9) Supposons 2.3k ≤ n < 3

k+1 , autrement dit n = 2.3

k + q , 0 ≤ q < 3

k .

Alors en vertu de ce qui précède, n = C(3k + q) = 2.3

k ,

Donc C(n) = C(C(3k + q)) = 3

k+1 + 3.q .

10) Le mieux est de raisonner en base 3, car le 8) signifie que le développement triadique de n commence par 1, le 9) qu’il commence par 2. Si n = 1a2 … ap , alors C(n) = 2a2 … ap .

Si n = 2a2 … ap , alors C(n) = 1a2 … ap 0. 11) Il reste à montrer réciproquement que cette fonction satisfait les trois propriétés de départ. Laissons cela au lecteur.

Conclusion : Il existe une et une seule fonction C satisfaisant aux trois conditions. Elle est donnée en fonction du développement en base 3 par : (I) C(0) = 0

(II) Si n = 1a2 … ap, alors C(n) = 2a2 … ap (III) Si n = 2a2 … ap, alors C(n) = 1a2 … ap 0.

Application : 100 = 10201 (en base 3), donc C(100) = 20201 (en base 3) = 181 (en base 10). Exercice 5 : Soit f : N → N bijective. Prouver qu’il existe trois entiers naturels a, b, c tels que : a < b < c et f(a) + f(c) = 2f(b). (Concours général 1995)

Page 6: Exercices équations fonctionnelles

6

Solution : • E = { n ∈ N ; f(0) < f(n) } est une partie non vide de N* (pourquoi ?) qui admet un plus petit élément, noté b. On a 0 < b et f(0) < f(b). • Soit n = 2 f(b) – f(0) ; f étant bijective, il existe c tel que f(c) = n. On a f(c) + f(0) = 2 f(b), donc

f(c) = f(b) + (f(b) – f(0)) > f(b) > f(0), donc c ∈ E, b = min E ≤ c, et même b < c.

• Posons a = 0. On a bien a < b < c et f(a) + f(c) = 2f(b) .

Exercice 6 : Soit f une application : N → N vérifiant (∀n ∈ N) f(n + 1) > f(f(n)). Montrer que f est l’identité. (Olympiades 1977)

Solution 1 : On a f(1) > f(f(0)), f(2) > f(f(1)), f(3) > f(f(2)), etc. 1) Montrons pour commencer que f(0) = 0. • Tout d’abord, je dis que ∀n ≥ 1 f(n) > f(0), autrement dit f(0) est le plus petit élément de f(N). En effet, f(N) = { f(0), f(1), f(2), … } est une partie non vide de N, donc a un plus petit élément. Ce plus petit élément n’est pas f(n + 1), car f(n + 1) > f(f(n)). Ce ne peut être que f(0). • Je dis que min{ f(1), f(2), … } > min{ f(f(0)), f(f(1)), … } ≥ f(0). Soit f(p) le plus petit élément de A = { f(1), f(2), … }. On a f(p) > f(f(p − 1)) ≥ min{ f(f(0)), f(f(1)), … } ≥ min f(N) = f(0). Du coup, min f(f(N)) n’est ni f(1), ni f(2), ni f(3)… ; ce ne peut être que f(0). f(0) = min f(N) = min f(f(N)). • Il existe donc p tel que f(0) = f(f(p)). Montrons par absurde que f(0) = 0.

Si f(0) > 0, posons f(0) = n + 1. Alors f(f(0)) = f(n + 1) > f(f(n)) ≥ f(0), donc f(0) ≠ f(f(0)). D’autre part, si k > 1, f(k) > f(f(k − 1)) ≥ 1, donc f(k) > 1. Posons f(k) = q + 1. Alors f(f(k)) = f(q + 1) > f(f(q)) ≥ f(0), donc f(0) ≠ f(f(k)). Conclusion : f(0) ≠ f(f(k)) pour tout k. Contradiction !

2) Montrons que f(1) = 1, etc. Il découle de ce qui précède que f(1), f(2), f(3), etc. sont tous ≥ 1. f laisse donc stable N* et vérifie pour tout n ∈ N* f(f(n + 1)) > f(f(n)). Il suffit de réappliquer ce qui précède à la restriction de f à N*, et de réitérer indéfiniment.

Autre solution, par descente infinie de Fermat.

• Je dis que ∀n ≥ 1 f(n) ≥ 1. En effet s’il existe n ≥ 1 tel que f(n) = 0, alors 0 = f(n) > f(f(n − 1)) ≥ 0 : impossible !

• Je dis que f(0) = 0. Supposons par absurde f(0) > 0, et définissons la suite x0 = 1 , xn+1 = f(xn – 1).

Cette suite est bien définie car xn ≥ 1 par récurrence.

Exercice 7 : Soit f : N×N → N telle que : • f(0, 0) = 0. • (∀m ∈ N) f(m + 1, 0) = f(0, m) + 1. • ∀(m, n) ∈ N*×N f(m − 1, n + 1) = f(m, n) + 1.

Calculer f(7, 5). Montrer que f est bijective. Formules générales donnant f(m, n) et f−1

(x) ?

Solution : 1) L’existence et l’unicité de f se montrent par récurrence forte sur s = m + n. Si s = 0, m = n = 0 et f(0, 0) = 0. Si f existe et est unique sur { (m, n) ; m + n ≤ s }, alors f est bien définie sur { (m, n) ; m + n = s + 1 }. Cela se montre par récurrence sur n : f(s + 1, 0) = f(0, s) + 1, et, si f est connue pour { (m, n) ; m + n = s + 1 et n ≤ a }, alors on doit poser : f(s – a, a + 1) = f(s – a + 1, a) + 1.

1 Solution dédiée à Daniel Depardieu, statisticien émérite et sympathique (20 et 21 août 1988).

Page 7: Exercices équations fonctionnelles

7

2) On montre que f(7, 5) = 83, et que f(m, n) = 2

)1)(( +++ nmnm + n , par la même récurrence.

3) Le caractère bijectif peut se montrer en calculant la bijection réciproque.

On montrera que : f(m, n) = x ⇔ (m, n) = (2

)3)()(( +xgxg −x , x−2

)1)()(( +xgxg ) ,

où g(x) = E(2

181 ++− n ), E désignant la fonction partie entière.

Exercice 8 : Soit f : N×N → N* définie par : • (∀y ∈ N) f(0, y) = 2y + 1 ; • ∀(x, y) ∈ N×N f(x + 1, y) = 2 f(x, y) .

Montrer que f est bijective. Calculer f−1

(31888).

Solution : On a aussitôt f(x, y) = 2x

(2y + 1).

Si n = f(x, y), x = v2(n) est l’exposant de 2 dans la factorisation de n.

Le caractère bijectif découle de l’unicité de cette factorisation. f−1

(31888) = f−1

(24.1993) = (4, 996).

Exercice 9 : Fonction d’Ackermann (1925). 1) Montrer qu’il existe une et une seule fonction f : N

2 → N vérifiant les trois axiomes :

f(n, m) = m + 1 si n = 0 f(n, m) = f(n−1, 1) si m = 0 et n > 0 f(n, m) = f(n−1, f(n, m−1)) si m > 0 et n > 0

2) a) Calculer f(2, 2). Calculer f(1, n), f(2, n), f(3, n) pour tout n. b) Calculer f(4, 1) et f(5, 0), f(4, 2), f(4, 1981).

Solution : 1) Montrons l’existence et l’unicité de la fonction f, par récurrence double sur x, puis sur y. • Si x = 0, les valeurs de f(x, y) sont connues. • Supposons connues les valeurs de f(x, y), pour tout y. Les valeurs de f(x + 1, y) vont se déterminer par récurrence sur y : ♦ f(x + 1, 0) = f(x, 1). ♦ Supposons connues les valeurs de f(x + 1, z) pour z ≤ y. Alors f(x + 1, y + 1) = f(x, f(x + 1, y)).

2) Calculons les valeurs de f sur les premières verticales de N×N. f(0, y) = y + 1 pour tout y. f(1, 0) = f(0, 1) = 2 et f(1, y) = f(0, f(1, y −1)) = f(1, y − 1) + 1 ; donc f(1, y) = y + 2 pour tout y. f(2, 0) = f(1, 1) = 3 et f(2, y) = f(1, f(2, y − 1)) = f(2, y – 2) + 2 ; d’où f(2, y) = 3 + 2y pour tout y. f(3, 0) = f(2, 1) = 5 et f(3, y) = f(2, f(3, y − 1)) = 3 + 2f(3, y −1). (f(3, y)) est une suite récurrente arithmético-géométrique : ω = −3 est point fixe de x → 2x + 3. f(3, y) − ω = 2 ( f(3, y) − ω ) implique f(3, y) − ω = 2

y.( f(3, 0) − ω ) , donc

f(3, y) = 8.2y − 3 = 2

y+3 − 3 pour tout y .

f(4, 0) = f(3, 1) = 13 et f(4, 1) = f(3, f(4, 0)) = f(3, 13) = 216

− 3 = 65533. f(5, 0) = f(4, 1) = 65533.

f(4, y) + 3 = 2f(4, y−1)+3

, donc f(4, y) = 2^2^…^2 − 3 , où il y a y + 3 « 2 ». Quant à f(4, 1981), ce nombre est si grand qu’on ne peut le calculer !!! Déjà f(4, 2) est énorme !

Remarque : cette fonction est un exemple de fonction « non récursive primitive ». Exercice 10 : On cherche les fonctions f : N* → N vérifiant : f(2n) = f(n) + 1 , f(2n + 1) = 0. Montrer l’existence et l’unicité de f. Reconnaître cette fonction. Généraliser.

Page 8: Exercices équations fonctionnelles

8

Exercice 11 : On cherche les fonctions f : N → N vérifiant f(0) = 0 , f(2n) = f(n) , f(2n + 1) = f(n) + 1. Montrer l’existence et l’unicité de f. Reconnaître cette fonction.

Solutions des exercices 10 et 11 : L’existence et l’unicité de f se montrent par récurrence forte sur N* ou N. La fonction de l’exercice 9 est f(n) = v2(n), exposant de 2 dans la factorisation de n. Celle de l’exercice 10 est f(n) = s(n), nombre de 1 figurant dans le développement binaire de n. Prenant appui sur l’unicité, il suffit de vérifier que v2 et s satisfont les propriétés de f.

Remarque : D’une façon générale, une propriété récurrente qui renvoie 2n et 2n+1 à n s’éclaire quand on introduit le développement binaire de n.

Exercice 12 : Trouver toutes les fonctions f : N → N telles que f(1) = 1 et ∀n ∈ N f(2n) = 2f(n).

Solution : Il vient f(0) = 0, et f(2x.(2y + 1)) = 2

x f(2y + 1). f est entièrement définie par les valeurs

qu’elle prend sur les nombres impairs > 1, valeurs que l’on peut choisir arbitraires. Exercice 13 : On désigne par f l’application de N* dans lui-même définie par f(1) = 1, f(3) = 3 et, pour tout n ≥ 1, f(2n) = f(n) f(4n + 1) = 2f(2n + 1) − f(n) f(4n + 3) = 3f(2n + 1) − 2f(n).

Déterminer le nombre des entiers n, 1 ≤ n ≤ 1988, pour lesquels f(n) = n. (Olympiades 1988)

Solution : 1) Ce programme Maple est parfaitement concluant, mais peu profond : > f:=proc(n) > local q,r; > q:=iquo(n,4);r:=irem(n,4); > if n = 1 then 1;elif n = 3 then 3; > elif r = 0 then f(q); > elif r = 2 then f(2*q+1); > elif r = 1 then 2*f(2*q+1)-f(q); > else 3*f(2*q+1)-2*f(q);fi;end; > s:=0:for n from 1 to 1988 do if f(n)=n then s:=s+1;fi; od;print(s);

92 2) Solution plus sérieuse. • La fonction f existe et est unique. Cela se montre par récurrence forte sur n. • On constate que f(n) est le « miroir » de n en écriture binaire. Autrement dit, si n = ak … a1a0 en

base 2, alors f(n) = a0a1 … ak. Il suffit pour le montrer d’établir que la fonction miroir satisfait les mêmes axiomes que f. • Par suite, on cherche les entiers palindromes en base 2 compris entre 1 et 1988 = 11111000100.

Or il y a 2k−1

palindromes à 2k chiffres exactement, et 2k palindromes à 2k + 1 chiffres exactement.

Parmi les palindromes à 11 chiffres, il faut en exclure 2, 11111111111 et 11111011111. En tout 1 + 2 + 2 + 4 + 4 + 8 + 8 + 16 + 16 + (32 − 2) = 92 points fixes. Exercice 14 : Soit f une application de N* dans N vérifiant les propriétés suivantes : • Pour tout couple (m, n) ∈ N*×N*, f(m + n) − f(m) − f(n) prend l’une des valeurs 0 ou 1 ; • f(2) = 0 , f(3) > 0 et f(9999) = 3333. Déterminer f(1982). (Olympiades 1982)

Page 9: Exercices équations fonctionnelles

9

Solution : 1) La fonction f vérifie f(m) + f(n) ≤ f(m + n) ≤ f(m) + f(n) + 1. Faisant m = n = 1, f(2) = 0 impose f(1) = 0. 0 < f(3) ≤ f(2) + f(1) + 1 = 1 impose f(3) = 1. 1 = f(3) + f(1) ≤ f(4) ≤ f(3) + f(1) + 1 = 2 et f(4) ≤ f(2) + f(2) + 1 = 1 imposent f(4) = 1. 1 = f(4) + f(1) ≤ f(5) ≤ f(4) + f(1) + 1 = 2 et f(5) ≤ f(2) + f(3) + 1 = 2 imposent f(5) = 1 ou 2. 2 = f(3) + f(3) ≤ f(6) ≤ f(3) + f(3) + 1 = 3.

2) Heuristiquement, f est presque additive, et f(x) ≈ x/3, donc f(n) ≈ q(n), où q(n) est le quotient euclidien par 3. En tout cas, cette fonction q vérifie bien : q(2) = 0 , q(3) > 0 et q(m) + q(n) ≤ q(m + n) ≤ q(m) + q(n) + 1. Je dis que, pour tout n, q(n) ≤ f(n) ≤ q(n) + 1. C’est vrai pour n ≤ 6. Supposons-le vrai pour n ≤ 3k. k = q(3k−2) + 1 ≤ f(3k–2) + f(3) ≤ f(3k−2+3) = f(3k+1) = f(3k–1+2) ≤ f(3k−1) + f(2) + 1 = k + 1. k = q(3k−1) + 1 ≤ f(3k–1) + f(3) ≤ f(3k−1+3) = f(3k+2) ≤ f(3k) + f(2) + 1 = k + 2. Exercice 15 : Déterminer s’il existe une application f : N* → N* vérifiant : • f(1) = 2 ; • f(f(n)) = f(n) + n pour tout n de N* ; • f(n) < f(n + 1) pour tout n de N*. (Olympiades 1993)

Solution : 1) Les deux premiers axiomes permettent de calculer les itérés de 1 par f : f(1) = 2 , f(2) = f(1) + 1 = 3 , f(3) = f(2) + 2 = 5 , f(5) = f(3) + 3 = 8 , … On reconnaît les nombres de Fibonacci f0 = 0 , f1 = 1 , fn+2 = fn+1 + fn :

Si f(n)

= f o … o f (n fois), alors par récurrence sur n f(n)

(1) = fn+2 et f(fn+1) = fn+2 pour n ≥ 1.

2) Il existe beaucoup d’applications strictement croissantes prolongeant f. f(4) = 6 ou 7, (f(6), f(7)) = (9, 10), (9, 11), (9, 12), (10, 11), (10, 12) ou (11, 12), etc. Cela découle de ce que fn+2 − fn+1 > fn+1 − fn.

3) Parmi elles, il en existe encore beaucoup qui vérifient, pour tout n, f(f(n)) = f(n) + n. Si f(4) = 6 , f(6) = 10 , f(10) = 16 , f(16) = 26 , etc. (suite à la Fibonacci) Il reste à choisir f(7) = 11 ou 12, etc. Si f(4) = 7 , f(7) = 11 , f(11) = 18 , f(18) = 29 , etc. Il reste à choisir f(6) = 9 ou 10, etc. Démontrer cela demanderait du soin. 4) Voici une fonction explicite utilisant la numération fibonaccienne d’un entier :

Lemme de Zeckendorff : Tout n ∈ N* s’écrit de façon unique sous la forme n = ∑+∞

=+

02

iii fε ,

où (∀i) εi ∈ {0, 1} et εi.εi+1 = 0. Admettons ce lemme, bien connu… de ceux qui le connaissent !…

Alors la fonction f, définie par f(n) = ∑+∞

=+

03

iii fε , vérifie les 3 conditions.

Exercice 16 : Fonctions additives. 1) Trouver toutes les fonctions f : N → N telles que ∀(m, n) ∈ N×N f(m + n) = f(m) + f(n). 2) Trouver toutes les fonctions f : Z → Z telles que ∀(m, n) ∈ Z×Z f(m + n) = f(m) + f(n). 3) Trouver toutes les fonctions f : Q → Q telles que ∀(m, n) ∈ Q×Q f(m + n) = f(m) + f(n).

Solution : Il s’agit de trouver les endomorphismes des monoïdes (N, +) et des groupes (Z, +) et (Q, +). 1) On a f(0) = 0. Posant a = f(1), il vient par récurrence sur n (∀n∈N) f(n) = an. Réciproque facile.

Page 10: Exercices équations fonctionnelles

10

2) On a f(0) = 0 et f est impaire. Posant a = f(1), il vient par récurrence sur n (∀n ∈ N) f(n) = an, puis (∀n ∈ Z) f(n) = an par imparité. Réciproque facile.

3) On a f(0) = 0 et f est impaire. Posant a = f(1), il vient par récurrence (∀n ∈ N) f(n) = an, puis (∀n

∈ Z) f(n) = an par imparité. Enfin, pour tout (p, q) ∈ Z×N* q.f(qp

) = f(p) = ap, donc f(qp

) = aqp

.

Finalement (∀x ∈ Q) f(x) = ax .

Exercice 17 : Soit f une application : Q → Q telle que ∀(x, y) ∈ Q×Q f(x + y) = f(x) + f(y) + 2547. On suppose que f(2004) = 2547. Déterminer f(2547). (Olympiades françaises, 2005)

Solution : 1ère idée : On a f(0) = −2547. La donnée de f(1) permet de calculer f(n) pour tout n ∈ N. Une récurrence facile montre en effet que f(n) = n.f(1) + 2547.(n − 1). La donnée de f(2004) fournit f(1), puis f(2547)… On a répondu à la question sans se préoccuper de l’existence et de l’unicité de f. Cela dit, on peut noter que f(nx) = n.f(x) + 2547.(n − 1) pour tout n ∈ N, n ∈ Z, puis tout n ∈ Q. f est une fonction affine sur Q. 2ème idée : Notons a = 2547. Le mieux est d’ajouter a à l’équation. Elle s’écrit g(x + y) = g(x) + g(y) pour tous rationnels x et y : nous voilà ramenés à l’équation de Cauchy dans Q ! On sait que g(x) = x.g(1), donc f est affine : f(x) = x.g(1) − a.

La donnée de g(2204) = 2a fournit g(1) = 20042547.2 . Finalement f(2547) =

3341311705.

On a montré au passage l’existence et l’unicité de f. Exercice 18 : Trouver les fonctions f : N → N telles que ∀(m, n) ∈ N×N 2 f(m + n) = f(m) + f(m + 2n). Mêmes questions en remplaçant N par Z et Q.

Solution : C’est, en quelque sorte, l’équation de Jensen discrète.

• Dans N, on a f(n + 2) – 2f(n + 1) + f(n) = 0, f(n + 1) – f(n) est constante, et f(n) = f(0) + n (f(1) − f(0)) ; bref, f est affine. Réciproque facile. • Mêmes conclusion dans Z • Dans Q, on a encore f(nx) = f(0) + n(f(x) − f(0)), et on en déduit que f(r) = f(0) + r(f(1) − f(0)) (poser x = p/q et n = q).

Exercice 19 : Déterminer toutes les fonctions f : Z → Z telles que : ∀(x, y) ∈ Z×Z f(x + y) + f(x − y) = 2 f(x) f(y) . Lesquelles sont bornées ?

Solution : Il s’agit d’une équation de d’Alembert discrète. 1) Si f est constante, f = c, c = 0 ou 1. 2) x = y = 0 donne f(0) = 0 ou 1. 3) Si f(0) = 0, y = 0 donne 2f(x) = 0, donc f ≡ 0.

4) Sinon, f(0) = 1 ; x = 0 donne f(y) + f(−y) = 2f(y), donc f est paire. De plus, pour tout n, f(n + 2) − 2 f(1)f(n + 1) + f(n) = 0 : suite récurrente linéaire ! On forme aussitôt l’équation caractéristique. • Si f(1) > 1, posons f(1) = ch θ (θ > 0). Il vient f(n) = ch(nθ) : or f est bornée ! • Si f(1) < − 1, posons f(1) = − ch θ (θ > 0). Il vient f(n) = (−1)

n.ch(nθ) : or f est bornée !

• Si f(1) = 0, alors f(n) = 1 si n ≡ 0 (mod 4), f(n) = −1 si n ≡ 2 (mod 4), f(n) = 0 si n est impair. • Si f(1) = 1, alors f(−1) = 1 et f(n) ≡ 1.

Page 11: Exercices équations fonctionnelles

11

• Si f(1) = −1, alors f(n) = (−1)n.

En résumé, on trouve 4 fonctions : les fonctions constantes 0 et 1, la fonction f(n) = (−1)n

et la fonction g(n) = 1 si n ≡ 0 (mod 4), g(n) = −1 si n ≡ 2 (mod 4), g(n) = 0 si n est impair.

Réciproque à faire. Les deux dernières découlent de ce que f(n) = cos(nπ), g(n) = cos(2πn ).

Remarque : Cherchons toutes les fonctions f : Z → Z satisfaisant la condition voulue. On trouve la fonction nulle, et les fonctions f(n) = Tn(a), où a ∈ Z, et Tn est le n-ième polynôme de Tchebychev.

Exercice 20 : Déterminer toutes les fonctions f : Z → Z telles que : ∀(x, y) ∈ Z×Z f(x + y) + f(x − y) = 2 [ f(x) + f(y) ] .

Solution : 1) Si l’on fait x = y = 0, il vient f(0) = 0. 2) Si l’on fait x = 0, il vient f(y) + f(−y) = 2 f(y), donc f(−y) = f(y) et f est paire. 3) Si l’on fait y = 1, il vient f(x + 1) + f(x − 1) − 2.f(x) = 2.f(1) , i.e. (∆2

f )(x) = 2.f(1).

On en déduit ∆3 f = 0, donc f est une fonction polynomiale de degré ≤ 2.

Compte tenu de 1) et 2) f est de la forme f(x) = ax2.

4) Réciproque facile. Exercice 21 : 1) Déterminer les f : Z → Z telles que ∀n ∈ Z ( f o f )(n) = n + 1. 2) Déterminer les f : Z → Z telles que ∀n ∈ Z ( f o f )(n) = n + 2.

Solution : [ Oral X PC 2010, RMS n° 364 ] 0) Dans les deux cas, f o f est bijective, donc si f existe, f est une bijection. 1) Dans le premier cas, en formant de deux façons ( f o f o f )(n), on obtient f(n) + 1 = f(n + 1). On en déduit ∀p ∈ Z f(p) = f(0) + p . Mézalor ( f o f)(p) = 2f(0) + p = p + 1. Cela impliquerait f(0) = ½ : impossible ! Autre solution : f(n) et f(n + 1) n’ont pas même parité. De deux choses l’une : • Soit f conserve la parité ; mais alors ( f o f )(n) a même parité que n et ne peut valoir n + 1. • Soit f change la parité ; mais alors ( f o f )(n) a même parité que n et ne peut valoir n + 1.

Bilan : Il n’y a pas de fonction f : Z → Z telle que ∀n ∈ Z ( f o f )(n) = n + 1.

2) On voit aussitôt que f(n) = n + 1 convient. On a, pour tout n : ( f o f o f )(n) = f(n) + 2 = f(n + 2). On en déduit par récurrence que ∀p ∈ Z f(2p) = f(0) + 2p , f(2p + 1) = f(1) + 2p . De plus, f étant bijective, f(0) et f(1) sont de parités différentes.

• Supposons f(0) = 2a , f(1) = 2b + 1. Alors ∀p ∈ Z f(2p) = 2a + 2p , f(2p + 1) = 1 + 2b + 2p . En écrivant ( f o f )(2p + 1) = f(1 + 2b + 2p) = 1 + 4b + 2p = 2p + 3 , on aboutit à b = ½ : impossible.

• Supposons f(0) = 1+ 2a , f(1) = 2b. Alors ∀p ∈ Z f(2p) = 1 + 2a + 2p , f(2p + 1) = 2b + 2p . En écrivant ( f o f )(2p + 1) = f(2b + 2p) = 1 + 2a + 2b + 2p = 2p + 3 , on aboutit à a + b = 1. En écrivant ( f o f )(2p) = f(1 + 2a + 2p) = 2a + 2b + 2p = 2p + 2 , on aboutit à a + b = 1.

Si a = 0, on retrouve la translation f(n) = n + 1.

Bilan : il y a une infinité de fonctions cherchées : les fonctions ∀p ∈ Z f(2p) = 1 + 2a + 2p , f(2p + 1) = 2 − 2a + 2p , où a est élément de Z.

Exercice 22 : Prouver qu’il n’existe pas d’application f : N → N telle que, pour tout n ∈ N, f(f(n)) = n + 1987. (Olympiades 1987)

Page 12: Exercices équations fonctionnelles

12

Solution : Soit f : N → N une application telle que, pour tout n ∈ N, f(f(n)) = n + 1987. Alors f(n) + 1987 = f(f(f(n))) = f(n + 1987) , et, par récurrence sur k, ∀(n, k) ∈ N×N f(n + 1987k) = f(n) + 1987k.

Par suite m ≡ n (mod 1987) ⇒ f(m) ≡ f(n) (mod 1987). La classe de f(n) mod 1987 ne dépend que de la classe de n mod 1987.

On peut donc factoriser f en une application g : Z/1987Z → Z/1987Z telle que g o g = id.

Comme 1987 est impair, l’involution g admet au moins un point fixe 0n .

Revenant à f, on a f(n0) = n0 + 1987.k0, donc :

• d’une part, f(f(n0)) = n0 + 1987 ;

• d’autre part, f(f(n0)) = f(n0) + 1987.k0 = n0 + 1987.2k0 … Impossible !

Remarque : Dans cet énoncé on peut remplacer 1987 par n’importe quel entier impair. En revanche, il existe une application f : N → N telle que, pour tout n ∈ N, f(f(n)) = n + 2p, à savoir f(n) = n + p.

Exercice 23 : Trouver toutes les fonctions f : N* → N* telles que ∀(m, n)∈N*×N* f(mn) = f(m)f(n).

Solution : Cherchons les fonctions f : N* → N* telles que ∀(m, n) ∈ N*×N* f(mn) = f(m)f(n), autrement dit les endomorphismes du monoïde (N*, ×). Tout d’abord f(1) = 1. En vertu du théorème fondamental de l’arithmétique, il suffit de se donner les valeurs prises par f sur les nombres premiers, valeurs que l’on peut choisir arbitraires.

f sera de la forme : n = ∏∈Pp

nvpp )( → ∏∈Pp

nvppf )()( .

Exercice 24 : Trouver toutes les bijections f : N* → N* telles que ∀(m, n)∈N*×N* f(mn) = f(m)f(n). Quelle est la plus petite valeur possible de f(1998) ? de f(4290) ?

Solution : Cherchons les bijections f : N* → N* telles que ∀(m, n) ∈ N*×N* f(mn) = f(m)f(n). Cet exercice fait suite au précédent. Je dis que, pour tout nombre premier p, f(p) doit être premier. En effet, f étant injective, p ≠ 1 ⇒ f(p) ≠ f(1) = 1. Si f(p) était composé, f(p) = a.b (a, b > 1), écrivons a = f(c), b = f(d). On aurait c > 1, d > 1, f(p) = f(cd), et p = cd serait composé.

En conclusion, f est de la forme n = ∏∈Pp

nvpp )( → ∏∈Pp

nvpp )()(σ , où σ est une permutation de l’ensemble

P des nombres premiers. Réciproque immédiate. On a 1998 = 2×3

3×37, donc f(1998) = σ(2)×σ(3)3×σ(37) ≥ 120,

valeur minimale correspondant par exeample à σ(2) = 3, σ(3) = 2 et σ(37) = 5. On a 4290 = 2×3×5×11×13, donc f(4290) = σ(2)×σ(3)×σ(5)×σ(11)×σ(13) ≥ 2×3×5×7×11.

Exercice 25 : Trouver toutes les fonctions f : N* → N* telles que : ∀(m, n) ∈ N*×N* f(mn) = f(m)f(n) et f(f(m)) = m. Quelle est la plus petite valeur possible de f(2007) ?

Solution : Les fonctions f : N* → N* telles que ∀(m, n) ∈ N*×N* f(mn) = f(m)f(n) et f(f(m)) = m

sont de la forme n = ∏∈Pp

nvpp )( → ∏∈Pp

nvpp )()(σ , où σ est une involution de l’ensemble P des nombres

premiers. Réciproque immédiate. De telles involutions sont faciles à fabriquer : cf. § 1. ex. 1. Il suffit de partitionner P en trois ensembles A, B et C, B et C étant équipotents, de fixer les éléments de A et d’échanger les éléments de B et C. On a 2007 = 3

2×223, donc f(2007) = σ(3)2×σ(223) ≥ 18 : échanger 2 et 223, laisser fixe 3.

Exercice 26 : On considère toutes les applications f de N* dans lui-même vérifiant :

Page 13: Exercices équations fonctionnelles

13

f( t2

f(s)) = s (f(t))2

. Déterminer la plus petite valeur possible de f(1998). (Olympiades 1998)

Solution : L’énoncé suggère que plusieurs fonctions répondent à la question. Tel est bien le cas. 1) Recherche de solutions. • Il n’y a pas de constantes, mais il y a l’identité et les f : x → ax (a ∈ N*).

• Ce ne sont pas les seules. Il est clair que si f : N* → N* vérifie ∀(a, b) ∈ N*×N* f(ab) = f(a).f(b) et f(f(a)) = a , alors f est solution. Or nous venons d’étudier ces fonctions, dans l’exercice précédent. 2) Recherche de toutes les solutions. • Commençons par imposer à f de vérifier f(1) = 1. s = 1 donne f(t

2) = f(t)

2 .

t = 1 donne f(f(s)) = s ; f est une involution, donc une bijection. Du coup, posant u = f(s), s = f(u), et f(t

2u) = f(u).f(t)

2 .

Montrons que f(ab) = f(a).f(b) , ou, ce qui revient au même, que f(ab)2 = f(a)

2f(b)

2.

f(ab)2 = f(a

2b

2) = f(a

2)f(b)

2 = f(a)

2f(b)

2.

• Si f(1) = c est quelconque, on montrera de même que f(f(n)) = c2n et f(n)

2 = f(cn

2), puis que f(c) est

du type précédent.

En conclusion, les fonctions f cherchées sont de la forme n = ∏∈Pp

xvpp )( → c∏∈Pp

xvpp )()(σ , où σ est une

permutation de l’ensemble PPPP des nombres premiers.

3) Retour au problème posé : 1998 = 2×3

2×37, f(1998) = c.σ(2)σ(3)2σ(37) est minimum si c = 1, σ(2) = 3, σ(3) = 2 et σ(37) = 5.

Au final, il vient min f(1998) = 120 .

Exercice 27 : Construire une application f : Q*+ → Q*+ vérifiant, pour tous x et y : f(x.f(y)) = yxf )(

.

(Olympiades 1990)

Solution : Paul Bourgade donne une solution lapidaire de cet exercice. Je préfère une solution plus longue et détaillée. Nous imposons à f de vérifier f(1) = 1.

1) x = 1 donne f(f(y)) = y1 ; il en découle que f est une permutation de Q*+.

Du coup, f(x.f(y)) = f(x).f(f(y)), mais en posant z = f(y), qui décrit Q*+, f(x.z) = f(x).f(z).

f est un automorphisme du groupe multiplicatif (Q*+, ×).

2) Réciproquement, si f est un endomorphisme du groupe (Q*+, ×) tel que pour tout y, f(f(y)) = y1 ,

alors f(x.f(y)) = f(x).f(f(y)) = yxf )(

pour tous (x, y).

3) Quels sont les endomorphismes du groupe (Q*+, ×) ?

Soit P l’ensemble des nombres premiers. Tout rationnel x > 0 s’écrit de manière unique sous la forme

x = ∏∈Pp

xvpp )( , où les exposants vp(x) sont des entiers relatifs presque touts nuls.

Dès lors, f(x) = ∏∈Pp

xvppf )()( : f est entièrement déterminé par les valeurs qu’il prend sur les nombres

premiers. Réciproquement, si (f(p))p∈P est une famille de rationnels > 0 indexée par P, alors

Page 14: Exercices équations fonctionnelles

14

x = ∏∈Pp

xvpp )( → f(x) = ∏∈Pp

xvppf )()( définit un endomorphisme de (Q*+, ×).

4) Mais on veut de plus que f(f(p)) = p1 et f(f(

p1 )) = p pour tout p ∈ P.

Pour cela, partageons P = { p1 = 2 < p2 = 3 < p3 = 5 < … } en deux ensembles disjoints et infinis, par

exemple A = { p1 = 2 < p3 = 5 < p5 = 11 < … } et B = { p2 = 3 < p4 = 7 < p6 = 13 < … }.

Posons f(p1) = p2 , f(p2) = 1

1p

, f(p3) = p4 , f(p4) = 3

1p

, etc.

Soit f l’endomorphisme de (Q*+, ×) ainsi défini (cf. 2). Je dis que f(f(y)) = y1 pour tout y, puisque

les endomorphismes f o f et y → y1 coïncident sur une partie génératrice du groupe (Q*+, ×). CQFD.

Remarque : D’un point de vue ensembliste, f est une racine carrée de l’involution s : y → y1 : sujet

abordé dans mon problème sur l’équation de Babbage. Exercice 28 : Trouver toutes les applications f de N dans N vérifiant : ∀(m, n) ∈ N×N f(m + f(n)) = f( f(m) ) + f(n) . (Olympiades 1996)

Solution : • m = n = 0 donne f(f(0)) = f(f(0)) + f(0), donc f(0) = 0.

• n = 0 donne f(m) = f(f(m)) ; f est donc idempotente. Conséquences : i) f induit l’identité sur E = f(N). ii) ∀(m, n) ∈ N×N f(m + f(n)) = f(m) + f(n). iii) ∀(m, n) ∈ N×E f(m + n) = f(m) + n. iv) ∀(m, n, q) ∈ N×E×N f(m + qn) = f(m) + qn (recurrence sur q). v) 0 ∈ E et ∀(m, n) ∈ E×E m + n ∈ E. • Si E = {0}, f = 0. • Sinon, soit a = min E. Je dis que E ⊂ aN . En effet, soit b ∈ E, b = aq + r, 0 ≤ r < a, la division euclidienne de b par a. r + aq = b = f(b) = f(r) + qa (par iv), donc f(r) = r, donc r = 0 par minimalité de a. • Soit n ∈ N, n = aq + r, 0 ≤ r < a la division euclidienne de n par a. f(n) = f(r + aq) = f(r) + aq. Comme f(r) ∈ E, écrivons f(r) = ag(r) pour 0 ≤ r ≤ a−1 ; on a g(0) = 0. Alors f(n) = a(q + g(r)).

Conclusion : f = 0 ou il existe un entier a > 0 et une fonction g : [0, a−1] → N telle que g(0) = 0 et que f soit de la forme : n = aq + r (0 ≤ r < a) → f(n) = a(q + g(r)).

Si a = 1, on trouve l’identité. Réciproque facile, mais à faire…

Exercice 29 : Soit f une application : N → N vérifiant f(1) > 0 et : ∀(m, n) ∈ N××××N f( m

2 + n

2 ) = f(m)

2 + f(n)

2.

1) Calculer f(k) pour 0 ≤ k ≤ 12. 2) Calculer f(n) pour tout n. (Concours général 1994)

Solution : en préparation… Les fonctions considérées dans les deux exercices suivants sont dites multiplicatives, ou parfois faiblement multiplicatives, par opposition avec les fonctions complètement multiplicatives. On nomme ainsi les fonctions f : N* → N*, Z ou R telles que f(1) = 1 et m ∧ n = 1 ⇒ f(mn) = f(m).f(n). Elles sont entièrement données par les valeurs qu’elles prennent sur les nombres primaires

(puissances de nombres premiers), puisque si n = ∏ ikip , f(n) = ∏ )( ik

ipf .

Exemples : indicateur d’Euler, fonction de Möbius, etc.

Page 15: Exercices équations fonctionnelles

15

Exercice 30 : Déterminer les fonctions f : N → N strictement croissantes, telles que f(2) = 2 et, pour tous entiers m, n premiers entre eux : f(mn) = f(m)f(n). (Putnam 1963)

Solution : 1) L’identité répond à la question. Nous allons montrer qu’il n’y en a pas d’autre. 2) f étant strictement croissante, f(0) = 0 , f(1) = 1 , f(2) = 2. Montrons que f(3) = 3. Posons f(3) = 3 + a. Alors f(6) = f(2).f(3) = 6 + 2a, d’où f(5) ≤ 5 + 2a. Du coup, f(10) = f(2).f(5) ≤ 10 + 4a , d’où f(9) ≤ 9 + 4a et f(18) ≤ 18 + 8a. D’où f(15) ≤ 15 + 8a. Mais f(15) = f(3).f(5) ≥ (3 + a)(5 + a) = 15 + 8a + a

2 . D’où a = 0 et f(3) = 3. Ouf !

3) Montrons que (∀n ∈ N) f(2n + 1) = 2

n + 1 par récurrence.

C’est vrai pour n = 0 et 1. Supposons-le vrai au rang n.

Alors f(2n+1

+ 2) = f(2).f(2n + 1) = 2

n+1 + 2.

Or il est clair que, si f est strictement croissante et vérifie f(x) = x et f(y) = y pour x < y, alors f(z) = z pour z ∈ [x, y]. Du coup, f(2

n+1 + 1) = 2

n+1 + 1.

4) On en déduit (même principe) que f est l’identité. Exercice 31 : Soit f : N* → N* telle que :

• Pour tous entiers a et b premiers entre eux, f(ab) = f(a).f(b) ;

• Pour tous nombres premiers p et q, f(p + q) = f(p) + f(q). Montrer que f(2) = 2 , f(3) = 3 et f(1999) = 1999. (Olympiades Irlande 1999)

Solution : 1) Une telle fonction f existe : l’identité. 2) Il est immédiat que f(1) = 1. De plus f(6) = f(2).f(3) = f(3) + f(3), donc f(2) = 2 (car f(3) non nul). 3) p = q = 2 donne alors f(4) = 4. p = 2, q = 3 donne f(5) = 2 + f(3). p = 5, q = 2 donne f(7) = 2 + f(5) = 4 + f(3). p = 5, q = 7 donne f(12) = 6 + 2f(3). Par ailleurs f(12) = f(3) f(4) = 4 f(3). Donc f(3) = 3. 4) 1999 est premier, et 1999 + 3 = 2002 = 2×7×11×13. Calculons f(11) et f(13). Il découle de 3) que f(3) = 3 et f(5) = 5. On en déduit f(15) = f(3) f(5) = 3.5 = 15. f(15) = f(13) + f(2), donc f(13) = 13. f(13) = f(11) + f(2), donc f(11) = 11. Finalement f(2002) = 2002 et f(1999) = f(2002) − f(3) = 1999.

Remarque : On peut conjecturer que f est l’identité. J’ai montré f(n) = n pour 1 ≤ n ≤ 16. Mais l’induction semble ardue…

Exercice 32 : Trouver toutes les fonctions f : Z → Z telles que, pour tous entiers a, b, c vérifiant a + b + c = 0, on ait l’égalité suivante : f(a)

2 + f(b)

2 + f(c)

2 = 2 f(a) f(b) + 2 f(b) f(c) + 2 f(c) f(a) . (Olympiades 2012)

Solution : 0) Si f est solution de cette équation fonctionnelle, λf aussi, où λ ∈ Z.

On vérifie que f(x) = x2 est solution de cette équation fonctionnelle. En effet, si a + b + c = 0, alors

a, b et c sont solutions d’une équation de la forme x3 – px + q = 0. Alors N2

2 = 2N4, car

N2 = S12 – 2S2 = 2p et N4 = 2p

2. On en déduit le résultat annoncé.

Il en résulte que les fonctions f(x) = λx2, λ ∈ Z, sont solutions de l’équation fonctionnelle.

1) Faisons a = b = c = 0. Il vient 3.f(0)2 = 6.f(0)

2 , donc f(0) = 0.

2) Faisons b = −a , c = 0. Il vient f(a)2 + f(−a)

2 = 2 f(a) f(−a) , donc f(−a) = f(a) ; f est paire.

3) Posons a = x , b = y , c = − x – y. Comme f est paire, il vient :

Page 16: Exercices équations fonctionnelles

16

f(x + y)2 + f(x)

2 + f(y)

2 = 2 f(x + y).[ f(x) + f(y) ] + 2 f(x) f(y) .

Donc f(x + y)2 − 2 f(x + y).[ f(x) + f(y) ] + [ f(x) − f(y) ]

2 = 0 (*)

C’est une équation du second degré en f(x + y), ayant une racine réelle : son discriminant est ≥ 0. Ainsi, ∀(x, y) 4 f(x).f(y) ≥ 0. Si f est nulle, pas de souci. Si f prend une valeur > 0, ce qu’on peut supposer quitte à changer f en – f, alors f est à valeurs ≥ 0 .

L’équation (*) donne : f(x + y) = f(x) + f(y) ± 2 )()( yfxf .

3. L’équation de Cauchy. Exercice 0 : Equation de Cauchy. Soit f : R → R une application telle que : ∀(x, y) ∈ R

2 f(x + y) = f(x) + f(y) (C)

1) Montrer que f(0) = 0, f est impaire et ∀(λ, x) ∈ Q×R f(λ.x) = λ.f(x) .

2) Montrer que, si f est continue ou monotone, il existe a ∈ R tel que (∀x ∈ R) f(x) = ax.

Solution :

1) procéder par étapes : λ∈N par récurrence, Z par imparité, Q en prenant λ = qp

, (p , q)∈Z×N*.

2) peut se traiter par deux méthodes : la méthode de Cauchy et la méthode d’Euler.

La méthode de Cauchy consiste à raisonner par densité. Elle suppose f continue ou monotone.

• Supposons f continue. La formule ∀λ ∈ Q f(λ) = λ.f(1) implique, par densité de Q et continuité de f, que ∀r ∈ R f(r) = r.f(1). Si l’on pose a = f(1), il vient ∀x ∈ R f(x) = a.x. Si f est continue en 0, f est continue en tout point. Le résultat subsiste.

• Supposons f monotone. Tout réel x est limite de deux suites adjacentes (rn) et (sn) de rationnels.

Ces suites vérifient f(rn) = a.rn et f(sn) = a.sn. Elles encadrent f(x) par monotonie de f, et elles tendent vers a.x. Donc f(x) = a.x.

Retrouvons ces résultats par la méthode d’Euler.

• Si f est continue, intégrons l’équation en y ∈ [0, 1], à x fixé.

Il vient, après changement de variable : (∀x) ∫+1

).(x

xdttf = f(x) + ∫

1

0.).( dyyf

Cela montre que f est C1. Par suite, en dérivant partiellement l’équation en x, f’(x + y) = f’(x), donc f’

est constante f’(x) = a. Comme f(0) = 0, f(x) est de la forme ax.

• Cette méthode reste vraie si f est réglée, ou intégrable-Riemann, sur tout segment [a, b] (a < b).

On a en effet (∀x) ∫+

+

bx

axdttf ).( = ( b − a ).f(x) + ∫

b

adyyf .).(

Il en résulte que f est continue, puis C1, et on conclut comme ci-dessus

En particulier, si f est monotone, elle est réglée sur tout segment.

Exercice 1 : Montrer que, si f vérifie (C) ∀(x, y) ∈ R×R f(x + y) = f(x) + f(y) et est bornée sur un segment [a, b] (a < b), elle est bornée sur [0, b − a]. En déduire qu’elle est continue en 0. Conclusion ?

Solution : Supposons | f(x) | ≤ M pour x ∈ [a, b].

Alors pour y ∈ [0, b − a], f(y) = f(y + a) − f(a), donc | f(y) | ≤ M + | f(a) | = µ.

f est donc bornée sur [0, α], où α = b − a. Par imparité, elle est bornée par µ sur [−α, α] .

Soit ε > 0. Choisissons n ∈ N* tel que nµ

≤ ε, et posons η = nα .

Page 17: Exercices équations fonctionnelles

17

Alors |x| ≤ η ⇒ | nx | ≤ α ⇒ | f(nx) | ≤ µ ⇒ | f(x) | ≤ nµ

≤ ε

Conclusion : f est continue en 0, et par translation, sur R.

Exercice 2 : Montrer que, si f vérifie (C) et est discontinue en 0, son graphe est dense dans R2.

Solution : Soit f une fonction vérifiant (C). Je dis que le graphe G de f est un sous Q-espace vectoriel de R2

. En effet, si (a, f(a)) et (b, f(b)) sont éléments de G, et si s et t ∈ Q, s(a, f(a)) + t(b, f(b)) = (sa + tb, f(sa + tb)) ∈ G. Si f est discontinue en un point, f n’est pas de la forme f(x) = x.f(1). Il existe donc un réel a tel que f(a) ≠ a.f(1). Les vecteurs u = (1, f(1)) et v = (a, f(a)) sont éléments de G et R-libres. (u, v) est une R-base de R

2, donc VectQ(u, v) est dense dans R2

et inclus dans G. Cqfd.

Exercice 3 : Trouver les triplets (f, g, h) de fonctions continues R → R vérifiant : ∀(x, y) ∈ R

2 f(x + y) = g(x) + h(y).

Peut-on affaiblir les hypothèses ?

Solution : On peut reprendre la méthode d’Euler, intégrer f(x + y) = g(x) + h(y) en y ∈ [0, 1].

∫+1

).(x

xdttf = g(x) + ∫

1

0).( dyyh . Cela montre que g est C

1. Par symétrie, h est C

1, et f(x) = g(x) + h(0)

aussi. Dérivons en x et en y : f’(x + y) = g’(x) = h’(y), donc f’(z) = g’(x) = h’(y) = a. Donc g(x) = ax + b , h(y) = ay + c , f(z) = az + b +c. Réciproque facile. Autre solution : posons g(0) = b, h(0) = c. Alors f(x) = g(x) + c, f(y) = b + h(y). f(x + y) = f(x) − c + f(y) − b. Posons k = b + c. Il vient f(x + y) − k = f(x) − k + f(y) − k.

f − k est continue et satisfait à l’équation de Cauchy : f(x) = ax + k , g(x) = ax + b , h(x) = ax + c.

Si l’on suppose f, g ou h continue en un point, alors f est continue en un point car f = g + c = h + b, et f − k aussi. Même conclusion.

Exercice 4 : Trouver les fonctions f : R → R telles que ∀(x, y) f(x2 − y

2) = f(x)

2 − f(y)

2.

Solution : • Faisant x = y, il vient f(0) = 0. Faisant y = 0, il vient f(x

2) = f(x)

2, donc y ≥ 0 ⇒ f(y) ≥ 0, et f(1) = 0 ou 1.

Faisant x = 0, il vient f(−y2) = − f(y)

2 = − f(y

2) ; donc f est impaire.

Je dis enfin que f(u + v) = f(u) + f(v) pour tout couple (u, v).

• Si u ≥ 0 ≥ v, posons u = x2 et v = −y

2. Alors f(u + v) = f(x

2 − y2) = f(x)

2 − f(y)

2 = f(x

2) + f(−y

2) =

f(u) + f(v). Idem si v ≥ 0 ≥ u. • Si u et v sont ≥ 0, posons u = x

2, v = y

2 et u + v = z

2. Alors f(u) = f(z

2 − y2) = f(z)

2 − f(y)

2 = f(z

2) −

f(y2) = f(u + v) − f(v), donc f(u + v) = f(u) + f(v). Idem si u et v sont négatifs par imparité.

En résumé, f satisfait à l’équation de Cauchy. De plus, elle est croissante, car y ≥ z ⇒ y − z ≥ 0 ⇒ f(y − z) = f(y) − f(z) ≥ 0 ⇒ f(y) ≥ f(z). Donc f est de la forme f(x) = ax. Comme f(1) = 0 ou 1, on a soit f(x) ≡ 0, soit f(x) ≡ x . Exercice 5 : Soit n un entier ≥ 2. Trouver les fonctions f : R → R vérifiant :

∀(x, y) ∈ R2 f( x + y

n ) = f(x) + ( f(y) )

n .

Solution : 1) x = y = 0 donne f(0) = 0. 2) x = 0 donne f(y

n) = f(y)

n . Par conséquent, f( x + y

n ) = f(x) + f(y

n).

Page 18: Exercices équations fonctionnelles

18

3) Supposons n pair . Soit z ≥ 0 ; il existe y tel que z = y

n. Alors f( x + z

) = f( x + y

n ) = f(x) + f(y

n) = f(x) + f(z).

On en déduit, en remplaçant x par x – z, que f(x – z) + f(z) = f(x). Si x = 0, on voit que f est impaire. Au final, f satisfait l’équation de Cauchy.

De plus, pour tout y , f(yn) = f(y)

n . On en déduit que f(1) = 0 ou 1.

Soit x ≥ 0. Il existe y ≥ 0 tel que x = yn. Alors f(x) = f(y)

n ≥ 0.

On en déduit que f est croissante. On sait qu’alors f(x) = ax, où a = f(1). Donc f(x) = 0 ou f(x) = x.

4) Supposons n impair . Comme t → tn est surjective, f satisfait l’équation de Cauchy.

f(1) = f(1)n implique f(1) = 0, 1 ou −1.

Maintenant, considérons l’égalité ∀(r, x, y) ∈ Q×R×R f((rx + y)n) = ( r.f(x) + f(y) )

n .

Développons les deux membres par le binôme. Comme ce sont deux polynômes en r, ils ont mêmes

coefficients. En particulier égalons les coefficients en rn−2

: f(x)n−2

f(y)2 = f(x

n−2 y2). Faisons x = 1.

Si f(1) = 0, il vient : f(y2) = 0. f est nulle sur R+ et sur R− par imparité.

Si f(1) = 1, il vient : f(y2) =

f(y)

2 , donc u ≥ 0 ⇒ f(u) ≥ 0 ; f est croissante : on est sauvé !

Si f(1) = −1, il vient : f(y2) = −

f(y)2 , donc u ≥ 0 ⇒ f(u) ≤ 0 ; f est décroissante : on est sauvé !

Les réciproques étant faciles, on conclut que :

Conclusion : Si n est pair, les fonctions trouvées sont f(x) = 0 et f(x) = x. Si n est impair, les fonctions trouvées sont f(x) = 0 , f(x) = x et f(x) = − x.

Exercice 6 : Trouver les endomorphismes d’anneau de R.

Solution : f satisfait l’équation de Cauchy f(x + y) = f(x) + f(y) . De plus, f(1) = 1 et f(xy) = f(x)f(y).

On a f(x2) = f(x

2), donc y ≥ 0 ⇒ f(y) ≥ 0, puis y ≥ z ⇒ y − z ≥ 0 ⇒ f(y − z) = f(y) − f(z) ≥ 0 ⇒ f(y) ≥

f(z). f est croissante, donc de la forme f(x) = ax. Comme f(1) = 1, f = idR. Remarque : Ce résultat est paradoxal, car il signifie que, bien que R soit une extension de Q de degré infini, le groupe de Galois de R sur Q est trivial : propriété que l’on pourrait croire réservée aux seuls corps premiers.

Exercice 7 : (généralisation) : Soit f : R → R vérifiant

f(1) = 1 , ∀(x, y) ∈ R2 f(x + y) = f(x) + f(y) , ∀x ∈ R

* f(x) ∈ R

* et f(

x1 ) =

)(1xf

.

1) Montrer que ∀x ∈ R−{0, 1} f ()1(

1xx − ) =

)1()(1

xfxf − .

2) En déduire que (∀x) f(x2) = f(x)

2 , puis que f = idR .

Solution : [Oral ENS]. 1) Soit x ≠ 0 et 1. D’une part, f()1(

1xx − ) = f(

x1 +

x−11 ) = f(

x1 ) + f(

x−11 )

= )(

1xf

+ )1(

1xf − =

)(1xf

+ )()1(

1xff − =

)(1xf

+ )(1

1xf− =

)1()(1

xfxf − .

D’autre part, f()1(

1xx − ) =

))1((1

xxf − = ²)()(

1xfxf − .

2) Donc f(x) − f(x2) = f(x).f(1 − x) = f(x).(1 − f(x)) = f(x) − f(x)

2.

Donc f(x2) = f(x)

2 pour x ≠ 0 et 1. Pour x = 0 et 1, c’est évident.

3) Du coup, f est croissante, et f(x) = ax ; f(1) = 1 impose f(x) ≡ x.

NB : Ce résultat est un cas particulier d’un théorème de Hua (cf. Artin, Algèbre géométrique). Exercice 8 : Trouver les endomorphismes continus de groupe, puis d’anneau, de C.

Page 19: Exercices équations fonctionnelles

19

Solution : 1) Un morphisme continu f de (R, +) dans (C, +) est de la forme f(x) = ax pour a ∈ C.

En effet x → Re f(x) et x → Im f(x) sont des endomorphismes continus de (R, +), donc de la forme Re f(x) = αx, Im f(x) = βx , α et β réels. 2) Soit g un endomorphisme continu de (C, +). Alors z = x + iy , g(z) = g(x) + g(iy). x→ g(x) et y → g(iy) sont des homomorphismes continus de (R, +) dans (C, +) ; d’après ce qui précède, ils sont de la forme g(x) = ax et g(iy) = by, a et b complexes. En résumé, g(z) = ax + by, a et b complexes. Réciproque facile.

3) Soit h un endomorphisme continu d’anneau de (C, +, ×). On a h(1) = 1 et h(i)

2 = h(i

2) = h(−1) = −1, donc h(i) = ± i.

h(z) = h(x) + h(i).h(y) = ax ± iay = x ± iy, donc h(z) ≡ z ou h(z) ≡ z .

Exercice 9 : Trouver les homomorphismes continus de groupe de (Rn , +) dans (R , +).

Solution : Soit f un homomorphisme continu de groupe de (Rn , +) dans (R , +).

L’application fi : xi → f(0, …, 0, xi, 0, …, 0) est un morphisme continu de (R , +) dans (R, +), donc

fi(xi) = aixi. Du coup si x = (x1, …, xn) , f(x) = ∑ )( ii xf =∑ ii xa . Réciproque facile.

Exercice 10 : Equation fonctionnelle de Jensen.

1) Montrer que les fonctions g : R → R vérifiant ∀(x, y) ∈ R×R 2

)()( ygxg + = g(

2yx+

) (J)

forment un espace vectoriel, et s’écrivent de manière unique sous la forme g = f + cte, où f vérifie l’équation de Cauchy. Quelles sont celles qui sont continues ? monotones ? 2) Soit I un intervalle de R. Etudier de même les fonctions g : I → R vérifiant :

∀(x, y) ∈ I × I 2

)()( ygxg + = g(

2yx+

) .

Solution : 1) Equation dans R. ♣ Notons que (J) s’écrit aussi ∀(x, y) g(x + y) − 2g(x) + g(x − y) = 0 (J’). ♦ La structure vectorielle de l’ensemble G des solutions ne pose aucun problème. Notons que si g est élément de G, g’(x) = g(−x) aussi. ♥ Cherchons d’abord les fonctions g telles que g(0) = 0.

x = 2x, y = 0 donne g(2x) = 2g(x), puis g(x + y) = g(222 yx+

) = 2

)2()2( ygxg + =

2)(2)(2 ygxg +

=

g(x) + g(y) : g est de Cauchy. Réciproque immédiate. Dans le cas général, écrire g = g(0) + f, où f relève du 1), donc f est de Cauchy ; etc.

♠ Autre approche : g est somme d’une fonction paire h et d’une fonction impaire k éléments de G, en vertu de ♦. Si h est paire et vérifie (J’), alors 2h(y) = h(y) + h(− y) = 2h(0) , donc h est constante.

Si k est impaire et vérifie (J’), alors k(x + y) + k(x − y) = 2k(x) et, k(x + y) – k(x – y) = k(x + y) + k(y – x) = 2k(y) , par échange de x et y. Donc (somme et différence) k(x + y) = k(x) + k(y). k satisfait l’équation de Cauchy. Ainsi g = k + cte, où k satisfait l’équation de Cauchy. Si g est continue, k aussi et g est affine. 2) Sur I, on raisonnera en utilisant les rationnels dyadiques ou la méthode d’Euler. Fixer x ∈ I, choisir un sous-segment [a, b] de I (a < b) et intégrer en y ∈ [a, b]. Exercice 11 : 1) Trouver toutes les fonctions continues g : R → R vérifiant :

∀(x, y) ∈ R×R 3

)()(2 ygxg + = g(

32 yx+ ) .

Page 20: Exercices équations fonctionnelles

20

2) Soit I un intervalle de R. Quelles sont les fonctions continues g : I → R vérifiant :

∀(x, y) ∈ I × I 3

)()(2 ygxg + = g(

32 yx+ ) .

Solution : Méthodes d’Euler et de Cauchy s’appliquent. g est affine.

Exercice 12 : Trouver toutes les fonctions continues f : R → R telles que :

∀(x, y) [ f(x) + f(y) ] f(2

yx+) = 2 f(x) f(y) . [ Oral Mines 1997 ]

Solution. Plaçons-nous d’abord sur un intervalle I de R. Si f est à valeurs ≠ 0, l’équation s’écrit

g(2

yx+) =

2)()( ygxg +

, où g = 1/f. Il résulte de l’ex. 8 que g(x) = ax + b, donc f(x) = bax+

1 . Mais

une telle fonction n’est définie sur R que si elle est constante. Il reste à montrer que si f s’annule en un point, f est nulle. En résumé, on trouve les constantes. Exercice 13 : Déterminer toutes les fonctions f : R → R continues en 0 et telles que : ∀(x, y) f(x + y) = f(x) + f(y) + 2xy .

Solution : Les fonctions vérifiant cette condition forment un espace affine, contenant la fonction x2.

Par soustraction, elles s’écrivent f(x) = x2

+ g(x) , où g satisfait l’équation de Cauchy.

Si f est supposée continue en 0, g l’est aussi, et f est de la forme f(x) = x2

+ ax. On peut aussi utiliser la méthode de renforcement d’Euler.

Exercice 14 : Déterminer toutes les fonctions f : R → R continues et telles que : ∀(x, y, z) f(x + y + z) + f(x) + f(y) + f(z) = f(x + y) + f(y + z) + f(z + x) .

Solution : 1ère méthode : on se ramène par étapes à l’équation de Cauchy.

i) Fixer y, et montrer que g : x → f(x + y) − f(x) − f(y) satisfait à l’équation de Cauchy. Du coup, il existe une constante ϕ(y) telle que ∀(x, y) f(x + y) − f(x) − f(y) = ϕ(y) x.

ii) ϕ(y) = f(1 + y) − f(1) − f(y) , donc ϕ est continue. De plus, ϕ satisfait l’équation de Cauchy. Du coup, il existe une constante a telle que ϕ(y) = ay, et f(x + y) − f(x) − f(y) = axy.

Enfin, la fonction h(x) = f(x) – 2a x

2 est continue et satisfait l’équation de Cauchy…

Remarque : au vu du résultat, il doit être possible de considérer les parties paire et impaire de f.

2ème méthode : renforcement d’Euler. Intégrons les deux membres sur (y, z) ∈ D = [0, 1]×[0, 1]. Il vient :

∫∫ ++D

dydzzyxf ).( + f(x) + 2∫1

0f = dyyxf ).(

1

0+∫ + ∫∫ +

Ddydzzyf ).( + dzxzf ).(

1

0+∫

Or dyyxf ).(1

0+∫ = dttf

x

x).(

1

∫+

= dzxzf ).(1

0+∫ , et le changement de variable (s, y) = (x + y, y) donne

∫∫ ++D

dxdyzyxf ).( = dssxfs .)(.2

0∫ + = dttfxtx

x.)().(

2

∫+

− = dttftx

x.)(.

2

∫+

− x dttfx

x.)(

2

∫+

.

f(x) = x dttfx

x.)(

2

∫+

− dttftx

x.)(.

2

∫+

+ 2 dttfx

x).(

1

∫+

+ ∫∫ +D

dxdyzyf ).( − 2∫1

0f .

Ceci montre que f est de classe C1, et, de proche en proche, C

∞ .

Revenons à l’équation initiale, et dérivons en x, puis en y : f’’( x + y + z) = f’’( x + y) . Du coup, f’’ = cte, et f(x) = ax

2 + bx + c. Mais f(0) = 0, donc f(x) = ax

2 + bx. Réciproque facile.

Conclusion : Les fonctions cherchées sont de la forme f(x) = ax2 + bx.

Page 21: Exercices équations fonctionnelles

21

Exercice 15 : Déterminer toutes les fonctions f : R → R continues et telles que :

∀(x, y) 2 f(x + y)3 = f(x).f(y).[ f(x) + f(y) ] .

Solution : Faisons x = y, il vient 2 f(x + y)

3 = 2 f(x)

3 , donc f(2x) = f(x).

Montrons par récurrence sur n ∈ N* que f(nx) = f(x). C’est vrai pour n = 1 et 2. Si c’est vrai au rang n, alors :

2 f((n + 1)x)3 = f(x).f(nx).[ f(x) + f(nx) ] = 2 f(x)

3 , donc f((n + 1)x) = f(x) .

Du coup, si r = qp

∈ Q*+ , q f(qp

x) = f(px) = f(x), donc f(rx) = f(x).

Par suite, f(r) = f(1) pour tout r = qp

∈ Q*+ ; par densité et continuité, f(x) = f(1) pour tout x ≥ 0.

De même, f(x) = f(−1) pour tout x ≤ 0. Par continuité, f est constante.

Exercice 16 : Soit E un espace euclidien de dimension ≥ 2. On cherche les fonctions f : E → R

continues et telles que ∀(x, y) ∈ E2 x ⊥ y ⇒ f(x + y) = f(x) + f(y) .

1) On suppose f paire. a) Montrer que ∀(x, y) ||x|| = ||y|| ⇒ f(x) = f(y) ;

b) Montrer que ∃a ∈ R ∀x ∈ E f(x) = a.||x||2 .

2) On suppose f impaire. Montrer que f est une forme linéaire sur E.

3) Conclure. [ Oral X 1994 ]

Solution : 1) Supposons f paire. a) Montrons que ||x|| = ||y|| ⇒ f(x) = f(y). x = y = 0 donne f(0) = 0. x ≠ 0 et y = αx donne y = ±x, donc f(y) = f(x) par parité.

Enfin, si x et y sont libres de même norme, alors 2

yx+ ⊥ ±

2yx−

, et

f(x) = f(2

yx+ +

2yx−

) = f(2

yx+) + f(

2yx−

) = f(2

yx+) + f(

2xy−

) = f(2

yx++

2xy−

) = f(y). cqfd.

b) En conclusion, f(x) ne dépend que de ||x||, ou encore de ||x||2.

Posons f(x) = g( ||x||2

), où g : R+ → R. Cette fonction g est continue et additive.

En effet, si a et b sont ≥ 0, on peut trouver x et y orthogonaux et tels que ||x||2 = a et ||y||

2 = b.

Alors par Pythagore et par hypothèse : g(a + b) = g( ||x||

2 + ||y||

2 ) = g( ||x + y||

2 ) = f(x + y) = f(x) + f(y) = g(||x||

2) + g( ||y||

2) = g(a) + g(b).

En vertu de l’équation de Cauchy, ∃a ∀t g(u) = at, donc f(x) = a ||x||2.

2) Si x et y sont positivement liés, soit e unitaire orthogonal à ces vecteurs. ∃λ ∈ R λe + x ⊥ y − λe ( cela équivaut en effet à (x | y) = λ2

; faire un dessin. ) Alors f(x + y) = f(x + λe + y − λe) = f(x + λe) + f(y − λe) = f(x) + f(λe) + f(y) + f(−λe) = f(x) + f(y) par imparité. Et f(x − y) = f(x) + f(−y) = f(x) − f(y). Du coup, f(x + y) = f(x) + f(y) dès que x et y sont liés. Du coup, f(λx) = λ f(x) pour λ ∈ Q, donc pour tout λ ∈ R par continuité.

Soit alors (e1, …, en) une base orthonormée de E.

x = ii ex∑ ⇒ f(x) = )( ii exf∑ = ii exf∑ )( : f est linéaire.

3) Les fonctions cherchées forment un sous-espace vectoriel E de FFFF(E, R). De plus, si f ∈ E, x → f(−x) aussi. Donc toute fonction f ∈ E est somme d’une fonction paire et d’une fonction impaire appartenant à E.

Page 22: Exercices équations fonctionnelles

22

En conclusion, f est de la forme f(x) = a ||x||2 + u(x), où u est une forme linéaire sur E.

Réciproque facile en vertu de Pythagore.

Exercice 17 : Soit E un plan euclidien. On cherche les fonctions f : E → R continues et telles que f(A) + f(C) = f(B) + f(D) pour tout rectangle ABCD.

Montrer que f(M) = a + (U |OM ) + c.||OM ||2

répond à la question. Réciproque ? [Oral ENS 2003]

Solution : Se ramener à l’exercice précédent en considérant Φ(OM ) = f(M) – f(O).

Exercice 18 : Trouver les fonctions f : R → R telles que ∀(x, y) ∈ R2 f(x + y) ≤ f(x) + f(y) et

f(x)/x → 1 quand x → 0.

Solution : [ Oral X MP 2012, RMS n° 213 ]

Soit x réel. Pour tout h ≠ 0, f(x + h) ≤ f(x) + f(h) et f(x) = f(x + h – h) ≤ f(x + h) + f(−h).

Pour tout h > 0, il vient : hhf

−− )(

≤ h

xfhxf )()( −+ ≤

hhf )(

.

Pour tout h < 0, il vient : hhf )(

≤ h

xfhxf )()( −+ ≤

hhf

−− )(

.

Si l’on fait tendre h vers 0+ et vers 0−, f est dérivable à droite et à gauche en x, de dérivée égale à 1. Donc f est dérivable en x et f’(x) = 1. Donc f est de la forme f(x) = x + c. La condition f(x)/x → 1 quand x → 0 donne c = 0. Au final, ∀x f(x) = x. Exercice 19 : Equation de Cauchy approchée.

Soit f une fonction continue : R → R telle que : ∃M ≥ 0 ∀(x, y) ∈ R2 | f(x + y) − f(x) − f(y) | ≤ M.

Montrer que f s’écrit de façon unique sous la forme f = g + h , où h est une homothétie et g une fonction continue bornée sur R.

[ Indication : On pourra considérer la suite de fonctions Un(x) = n

n xf2

).2(.]

Solution : (Un) est une suite uniformément convergente de fonctions continues.

En effet, | Un+1(x) − Un(x) | = 121

+n )2(2)2( 1 xfxf nn −+ ≤ 12 +nM : la série de fonctions ∑ −+ )( 1 nn UU est

normalement convergente. La limite h de cette suite est donc continue.

De plus, | Un(x + y) − Un(x) − Un(y) | ≤ 12 +nM ; à la limite, h est additive. h est donc une homothétie.

g = f − h = U0 − h = −∑ −+ )( 1 nn UU est continue et bornée. L’unicité se montre par soustraction.

NB : Les fonctions f considérées forment un espace vectoriel, somme directe de la droite des homothéties et du sous-espace des fonctions continues bornées. Si f n’est plus supposée continue, on a encore f = g + h, où g est bornée et h est additive. Exercice 20 : Soit f une fonction continue : R → R telle que :

(∀x ∈ R) lim y→+∞ f(x + y) − 2 f(x) + f(x − y) = 0.

On note g et h les parties paire et impaire de f, définies par g(x) = 2

)()( xfxf −+, h(x) =

2)()( xfxf −−

.

1) Montrer que g est constante.

Page 23: Exercices équations fonctionnelles

23

2) Montrer que (∀x ∈ R) limn→+∞ nnxh )(

= 2 h(2x ).

3) En conclure que f est affine.

Solution : équation de Jensen approchée.

0) Les fonctions en question forment un sous-espace vectoriel J de C(R, R). De plus, si f ∈ J, il en est de même de f^(x) = f(−x), donc de g et h.

1) 2g(y) − 2g(0) = g(y) – 2g(0) + g(−y) → 0 quand y → +∞, donc g(y) → g(0) quand y → +∞. Donc g(x + y) − 2g(x) + g(x − y) = g(y + x) − 2g(x) + g(y – x) → 2g(0) – 2g(x) quand y → +∞. Or g(x + y) − 2g(x) + g(x − y) → 0 quand y → +∞. Conclusion : g(x) = g(0).

2) h étant impaire, h(x + y) + h(x − y) = h(y + x) – h(y – x) → 2h(x) quand y → +∞. Fixons x > 0. Alors h((n + 1)x) – h((n – 1)x) = h(nx + x) – h(nx – x) → 2h(x) quand n → +∞. Du coup, h((2n + 2)x) − h(2nx) → 2h(x) quand n → +∞.

En vertu du théorème de Cesàro, nnxh )2(

→ 2h(x) quand n → +∞.

Remplaçant x par 2x , il vient

nnxh )( → 2h(

2x ) quand n → +∞.

Cela reste vrai pour x < 0 par imparité, et bien sûr pour x = 0. Remplaçons x par (px)/q, et n par kq : quand k tend vers l’infini :

D’une part, kqkpxh )(

= kq

qpxkqh )(

tend vers 2h(q

px2

).

D’autre part kqkpxh )(

= qp

kpkpxh )(

tend vers qp2

h(2x ).

D’où 2h(q

px2

) = qp2

h(2x ), et par densité et continuité, 2h(

2xλ ) = 2λh(

2x ) pour tout λ réel.

x = 2 donne h linéaire. Variante : revenir à h(n(x + y)) − 2h(nx) + h(n(x − y)) = 0, diviser par n et faire tendre n vers +∞. h vérifie l’équation de Jensen, est impaire et continue... 3) Finalement f = g + h est affine.

4. Logarithmes et exponentielles.

Exercice 1 : Trouver toutes les fonctions continues f : R → R vérifiant ∀(x, y) f(x + y) = f(x).f(y).

Solution : f(0) = f(0)2 implique f(0) = 0 ou 1 ; f(0) = 0 implique aussitôt f ≡ 0.

f(0) = 1 implique f(x).f(−x) = 1, donc (∀x) f(x) ≠ 0. En vertu du théorème des valeurs intermédiaires, f(x) est de signe constant, en l’occurrence > 0 car f(0) > 0. Passons au logarithme : ln f(x + y) = ln f(x) + ln f(y) , et ln f est continue. Donc (équation de Cauchy), (∃a) (∀x) ln f(x) = ax et f(x) = axe .

Exercice 2 : Trouver toutes les fcts continues f : R*+ → R*+ vérifiant ∀x, y > 0 f(x.y) = f(x).f(y).

Solution : g(x) = ln f(x) est un logarithme, donc ln f(x) = a.ln x, et f(x) = exp(a.ln x) = xa.

En résumé, f est une puissance.

Exercice 3 : Trouver les triplets (f, g, h) de fonctions continues R → R vérifiant resp. :

∀(x, y) ∈ R2 f(x + y) = g(x).h(y).

∀(x, y) ∈ R2 f(x.y) = g(x) + h(y).

Page 24: Exercices équations fonctionnelles

24

∀(x, y) ∈ R2 f(x.y) = g(x).h(y).

Solution : Traitons la première question. • Supposons d’abord que g et h ne s’annulent jamais. Du coup, f non plus.

Alors f(x) = g(x).h(0) = g(0).h(x), donc f(x + y) = )0()(.

)0()(

gyf

hxf

= )0(

)()(f

yfxf.

La fonction ϕ(x) = )0()(

fxf

est continue, jamais nulle, telle que ϕ(0) = 1 et ϕ(x + y) = ϕ(x)ϕ(y).

Elle est donc > 0 (TVI) et (∃a) (∀x) ln ϕ(x) = ax.

Conclusion : f(x) = A axe , g(x) = B axe , h(x) = C axe (A, B, C non nuls, A = B.C).

• Si g s’annule en un point x0, alors (∀y) f(x0 + y) = g(x0).h(y) = 0, donc f ≡ 0.

Alors ∀(x, y) 0 = f(x + y) = g(x).h(y). Soit g ≡ 0, soit h ≡ 0 (car si g(x1) ≠ 0 ⇒ (∀y) h(y) = 0).

Conclusion : (0, g, 0), (0, 0, h).

Exercice 4 : Trouver toutes les fonctions continues f : R*+ → R vérifiant :

∀ x, y > 0 f(x.y) = y f(x) + x f(y).

Solution : On pourrait utiliser la méthode d’Euler, mais il est plus simple de diviser par xy. Il vient g(x.y) = g(x) + g(y), où g(x) = f(x)/x. g est continue, donc est un logarithme, g(x) = a.ln x ; d’où f(x) = a.x.ln x .

Exercice 5 : Trouver une fonction f : R → R telle que : i) ∀(x, y) f(x + y) ≥ f(x).f(y) ii) (∀x) f(x) ≥ 1 + x.

Solution : 1) On pense aussitôt à l’exponentielle. 2) x = 0 ⇒ f(0) ≥ f(0)

2 et f(0) ≥ 1 ⇒ f(0) = 1.

3) x = y ⇒ f(2x) ≥ f(x)2 ≥ 0. Donc f est à valeurs ≥ 0.

4) De plus f(x) = 0 ⇒ f(2x ) = 0 ⇒ f( n

x2

) = 0 pour tout n. Or au voisinage de 0, f est ≥ 1/2.

Donc f est à valeurs > 0.

5) Pour tout x ≥ 0 et tout entier n ≥ 1, f(nx) ≥ f(x)n ≥ (1 + x)

n, donc f(x) ≥ (1 +

nx )

n.

A la limite f(x) ≥ exp x. Ceci ne sert pas dans la suite.

6) Pour tout y, f(y) > 0, donc, quels que soient x et y : f(x + y) ≥ (1 + x) f(y). Appliquons ceci au couple (−x, x + y) ; il vient : f(y) ≥ (1 – x) f(x + y).

Donc, pour tout x < 1 et tout y : (1 + x) f(y) ≤ f(x + y) ≤ xyf

−1)(

(*).

Quand x tend vers 0, f(x + y) → f(y) par encadrement ; donc f est continue.

7) Mieux ! repartons de la relation (*). Elle s’écrit : x.f(y) ≤ f(x + y) − f(y) ≤ x

x−1 f(y).

Si 0 < x < 1, il vient f(y) ≤ x

yfyxf )()( −+ ≤

x−11 f(y).

Si l’on fait tendre x vers 0, on voit que f est dérivable à droite en y et f’d(y) = f(y).

Si x < 0, il vient x−1

1 f(y) ≤ x

yfyxf )()( −+ ≤ f(y).

Si l’on fait tendre x vers 0, on voit que f est dérivable à gauche en y et f’g(y) = f(y). En résumé, f est dérivable en tout point et f’ = f. Comme f(0) = 1, f est l’exponentielle.

Page 25: Exercices équations fonctionnelles

25

Exercice 6 : Trouver toutes les fcts continues f : R → R vérifiant ∀(x, y) f( ²² yx + ) = f(x).f(y).

Solution : 1) x = y = 0 donne f(0) = 0 ou 1.

y = 0 donne f(|x|) = f(0).f(x) ; x = y donne f( |x| 2 ) = f(x)2.

2) f(0) = 0 implique f(|x|) = 0, donc f = 0 sur R+, puis f ≡ 0.

3) Désormais f(0) = 1. Donc f est paire.

f( |x 2 | ) = f(x)2 , donc f est à valeurs ≥ 0 sur R

+, donc sur R.

4) Si f(x0) = 0, alors f(20x ) = 0, donc f(

n

x)2(

0 ) = 0, et par continuité f(0) = 0 ; impossible.

Donc f est à valeurs > 0.

5) g(x) = ln f( x ) est continue et additive sur R+, car g(x

2 + y

2) = g(x

2) + g(y

2).

Donc g(x) = ax sur R+, et par parité g(x) = a|x|. Finalement f(x) = exp(ax

2).

Conclusion : f = 0 ou f(x) = exp(ax2) . Réciproque facile.

Exercice 7 : Trouver toutes les fcts continues f : R → R vérifiant ∀(x, y) )()()()(

yyfyxfyxfxxf

++++ = 0.

Solution : L’équation s’écrit f(2x).f(2y) = f(x + y)2 ; elle implique f(2x).f(0) = f(x)

2.

• Si f(0) = 0, f est nulle. Sinon, je dis que (∀x) f(x) ≠ 0, car f(x) = 0 ⇒ f(x/2) = 0 ⇒ … ⇒ f(x/2

n) = 0 ⇒ f(0) = 0 à la limite.

Plus sobrement, si f s’annule en un point y, alors f(x +2y

) est nul pour tout x, donc f = 0.

• En vertu du théorème des valeurs intermédiaires, f est de signe constant. Quitte à remplacer f par cf,

supposons f(0) = 1. Passant au log, g(x) = ln f(x) est continue et vérifie 2

)()( ygxg + = g(

2yx+

). C’est

l’équation de Jensen. • On sait qu’alors g est affine (§1, ex. 8). En résumé f(x) = c.exp(ax) . Réciproque facile.

Exercice 8 : entropie de Shannon, théorie de l’information .

1) Soit H une fonction ]0, 1] → R vérifiant :

(E1) ∀p ∈ ]0, 1] H(p) ≥ 0 (E2) ∀p, q ∈ ]0, 1] H(pq) = H(p) + H(q) (E3) H(½) = 1.

Montrer que ∀p ∈ ]0, 1] H(p) = − log2 p (Wiener, 1948).

2) Soit Φ une fonction ]0, 1] → R vérifiant :

(E1) ∀p ∈ ]0, 1] Φ(p) ≥ 0 (E2) ∀p, q ∈ ]0, 1] Φ(pq) = q Φ(p) + p Φ(q) (E3) Φ(½) = ½.

Montrer que ∀p ∈ ]0, 1] Φ(p) = − p log2 p .

Solution : 1) La fonction H(p) = − log2 p répond à la question. Réciproquement, si H satisfait (E1), (E2) et (E3), H est décroissante, donc intégrable sur tout segment ⊂ ]0, 1] (car fonction réglée). Intégrons H(pq) = H(p) + H(q) pour q ∈ [½ , 1].

Il vient : p1 ∫

p

pduuH

2/).( =

2)(pH

+ ∫1

2/1).( dqqH .

Il en résulte que H est continue, puis C1, C

2, … et C

∞.

Dérivons en p : q.H’(pq) = H’(p), d’où (p = 1) H’(q) = q

H )1'( ; comme H(1) = 0, H(q) = H’(1).ln q.

Page 26: Exercices équations fonctionnelles

26

Mais H(½) = 1 impose H(q) = − log2 q. 2) Considérer H(p) = Φ(p)/p .

Exercice 9 : Trouver les endomorphismes continus de (U, ×), puis de (C*, ×).

Solution : Les endomorphismes continus de (U, ×) sont les f : z → zk

, où k ∈ Z. Cela se déduit aisément de la proposition du § 2.1. On peut aussi développer en série de Fourier la

fonction continue g(t) = f(eit). On montrera que (∀a ∈ R) (∀n ∈ Z) cn(g) = g(a).e

−ina cn(g), et on

notera que les cn(g) ne sont pas tous nuls…

Les endomorphismes continus de (C*, ×) sont de la forme h(reiθ

) = ra.e

ikθ ( a ∈ C, k ∈ Z ).

[Indication : si h est l’un d’eux, étudier ses restrictions à R*+ et à U]. Exercice 10 : Equation de d’Alembert. Trouver les fonctions continues f : R → R vérifiant : ∀(x, y) f(x + y) + f(x − y) = 2 f(x).f(y) .

Solution : 0) On songe immédiatement aux fonctions 0, 1, cos et ch. 1) Faisant x = y = 0, il vient f(0) = 0 ou 1. Si f(x) = 0, faisant y = 0, il vient f(x) = 0. Désormais f(0) = 1. Alors, faisant x = 0, on voit que f est paire.

2) Utilisons la méthode de renforcement d’Euler. • f étant continue en 0, ∃a > 0 ∀x ∈ [0, a] f(x) > 0.

• Intégrons en y sur [0, a], et faisant des changements de variables, il vient

∫+ax

xdttf ).( + ∫ −

x

axduuf ).( = 2 f(x) ∫

adyyf

0).( , i.e. ∫

+

ax

axdttf ).( = 2 f(x) ∫

adyyf

0).(

où ∫a

dyyf0

).( > 0 en vertu de la stricte croissance de l’intégrale des fonctions continues.

On en déduit aussitôt que f est C1, et par récurrence, C

2, …, C

∞.

• Revenons à f(x + y) + f(x − y) = 2f(x).f(y) et dérivons partiellement deux fois, en x puis en y. Il vient 2 f’’( x) f(y) = f’’( x + y) + f’’( x − y) = 2 f(x) f’’( y). Faisant y = 0, on a f’’( x) = 2 f’’(0) f(x), d’où : f(x) = a.cos(ωx) + b.sin(ωx) , f(x) = ax + b ou f(x) = a.ch(ωx) + b.sh(ωx) ( ω > 0 ). Mais f est paire et f(0) = 1.

Finalement, f(x) = 0 ou f(x) = 1 ou f(x) = cos(ωx) ou f(x) = ch(ωx) ( ω > 0 ) .

• Réciproque à faire, mais déjà mentionnée en 0) !

3) Utilisons la méthode de densité de Cauchy. ♣ f étant continue en 0, ∃a > 0 ∀x ∈ [0, a] f(x) > 0.

♦ On peut calculer f(na) pour tout n ∈ N, donc, par parité, pour tout n ∈ Z. En effet la suite (f(na)) est récurrente d’ordre 2 : f((n + 2)a) − 2f(a).f((n + 1)a) + f(na) = 0. Il s’agit d’une récurrence linéaire ; formons l’équation caractéristique : − si f(a) = 1, on trouve f(na) = 1 pour tout n. − si f(a) ∈ ]0, 1[, posons f(a) = cos ω (0 < ω < π/2). Alors on trouve f(na) = cos(nω). − si f(a) > 1, posons f(a) = ch ω (ω > 0). On trouve f(na) = ch(nω).

NB : dans tous les cas, f(nx) = Tn(f(x)), où Tn est le n-ième polynôme de Tchebychev.

♥ Il découle de l’équation de d’Alembert que f(2x ) = ±

21)( +xf

. Comme f( qa2

) > 0, on a aussitôt :

Page 27: Exercices équations fonctionnelles

27

− si f(a) = 1, f(2a ) = 1 et par récurrence sur q, f( q

a2

) = 1. Donc f( )2

apq = 1 .

− si f(a) ∈ ]0, 1[, f(2a ) = cos

2ω et par récurrence sur q, f( q

a2

) = cos q2ω . Donc f( )

2ap

q = cos( )2

ωqp

.

− si f(a) > 1, f(2a ) = ch

2ω et par récurrence sur q, f( q

a2

) = ch q2ω . Donc f( )

2ap

q = ch( )2

ωqp

.

Dans les trois cas, on a appliqué le résultat de ♦, non à a, mais à qa2

.

♠ Il reste à conclure par densité des rationnels dyadiques.

4) Que peut-on dire si l’on abandonne l’hypothèse de continuité ? Si ϕ : R → R vérifie la condition de Cauchy ϕ(x + y) = ϕ(x) + ϕ(y), il est clair que f(x) = cos ϕ(x) et g(x) = ch ϕ(x) vérifient l’équation de d’Alembert.

Exercice 11 : Trouver tous les couples ( f, g) de fonctions continues de R dans R vérifiant :

i) ∀(x, y) ∈ R2 f(x + y) = f(x).f(y) + g(x).g(y) et g(x + y) = g(x).f(y) + g(y).f(x) .

ii) ∀(x, y) ∈ R2 f(x + y) = f(x).f(y) − g(x).g(y) et g(x + y) = g(x).f(y) + g(y).f(x) .

iii) ∀(x, y) ∈ R2 f

2(x) − g

2(x) = 1 et g(x + y) = g(x).f(y) + g(y).f(x) .

iv) ∀(x, y) ∈ R2 f

2(x) + g

2(x) = 1 et f(x + y) = f(x).f(y) − g(x).g(y) .

Solution partielle : i) On songe aux fonctions (ch, sh) mais aussi à (0, 0). Faisons la somme et la différence : il vient s(x + y) ≡ s(x).s(y) et d(x + y) ≡ d(x) + d(y), où s = f + g et d = f − g.

Or les seules fonctions continues h : R → R telles que h(x + y) ≡ h(x).h(y) sont h = 0 et h(x) = eax

.

(cf. ex 1). Du coup, s(x) = 0 ou eax

, d(x) = 0 ou ebx

. D’où quatre situations :

f = g = 0 , f = g = 2

axe , f = − g =

2

bxe , f(x) =

2

bxax ee + et g(x) =

2

bxax ee −.

Ce dernier cas s’écrit aussi f(x) = eαx

.ch(βx) , g(x) = eαx

.sh(βx).

ii) Même technique, mais il faut poser s = f + ig , d = f – ig , et utiliser la proposition suivante, établie dans le chapitre (§ 4.2) :

Proposition : Les morphismes continus de (R, +) dans (U, ×) sont de la forme f(x) = eibx

.

Les morphismes continus de (R, +) dans (C*, ×) sont de la forme g(x) = e(a+ib)x

. Exercice 12 : « Caractères du second degré sur le groupe (R, +) ». On cherche les applications continues f : R → U vérifiant ∀(x, y, z) ∈ R×R×R f(x + y + z) f(x) f(y) f(z) = f(x + y) f(y + z) f(z + x).

1) Montrer que, pour tout y ∈ R, g : x → )()()(

yfxfyxf +

est un morphisme continu de (R, +) dans (U, ×).

2) Montrer que f est de la forme f(x) = )²( bxaxie + ( a et b réels ).

Solution : 1) g est continue, et g(x + z) = )()()(

yfzxfyzxf

+++

= )()()(

yfxfyxf +

)()()(

yfzfyzf +

= g(x).g(z).

On en déduit (cf.cours) qu’il existe un réel b, dépendant de y, tel que (∀x ∈ R) g(x) = eib(y)x

. Exercice 13 : On cherche les fonctions f de R dans R, continues en 0, et telles que :

Page 28: Exercices équations fonctionnelles

28

∀(x, y) ∈ R2 f(x + y) =

)().(1)()(yfxfyfxf

++

.

1) Montrer que (∀x) | f(x) | ≤ 1 ( faire x = y ). 2) Montrer que, si f prend l’une des valeurs 1 ou –1, f est constante. 3) Ce cas étant exclu, trouver f. Solution : 0) On songe aux fonctions 0, ±1 et th.

1) x = y donne f(2x) = )²(1)(2

xfxf

+ . Comme |²1

2tt

+ | ≤ 1 pour tout t, |f(2x)| ≤ 1, donc pour tout x, |f(x)| ≤ 1.

2) Si f prend la valeur ±1, f(y0) = ± 1, alors f(x + y0) = )(11)(

xfxf

±±

= ±1, donc f est constante.

3) Désormais |f(x)| < 1 pour tout x. Posons f(x) = th g(x), i.e. g(x) = Argth f(x). g est continue et vérifie th g(x + y) = th(g(x) + g(y)) , donc g(x + y) = g(x) + g(y). On est ramené à l’équation de Cauchy : ∃a ∀x g(x) = ax. Conclusion : f = +1, −1 ou ∃a ∀x f(x) = th(ax) (La fonction nulle correspond à a = 0) Exercice 14 : un groupe matriciel.

On cherche toutes les fonctions continues f : ]−1, 1[ → R telles que les matrices M(u) = f(u).

11u

u

où u ∈ ]−1, 1[, forment un sous-groupe de Gl2(R).

1) Prouver que f vérifie l’équation fonctionnelle ∀(u, v) ∈ ]−1, 1[ (1 + uv).f(u).f(v) = f(uvvu

++

1).

On pose ∀x ∈ R g(x) = f(th x). 2) Trouver l’équation fonctionnelle vérifiée par g, à l’aide de la fonction ch. 3) Trouver toutes les fonctions g puis les fonctions f cherchées.

Solution : 1) det M(u) ≠ 0 implique f(u) ≠ 0 pour tout u ∈ I = ]−1, +1[.

2) On a M(u).M(v) = f(u).f(v)(1 + uv)

11w

w , où w = uvvu

++

1∈ I car |u + v| < 1 + uv.

Par stabilité, M(u).M(v) = M(w) , et f(u).f(v).(1 + uv) = f(uvvu

++

1).

3) La fonction g = f o th est continue de R dans R* et vérifie g(x).g(y).ch(x+y) = ch x.ch y.g(x+y)

pour tout couple (x, y), autrement dit )()(yxchyxg

++

= )()(xchxg

)()(ychyg

.

La fonction h = g/ch est un morphisme continu de (R, +) dans (R*, ×), donc h est de la forme h(x) =

eax

, et g(x) = eax

.ch x. Revenons à f : f(u) = g(Argth u) = g(21 ln

uu

−+

11 ) =

21 [ 2

1

)11(

+

−+ a

uu + 2

1

)11(

−+ a

uu ].

4) Réciproque facile, mais à faire. Exercice 15 : Trouver les fonctions continues f : C* → C telles que ∀z, z’∈C* f(z.z’) = f(z) + f(z’).

Trouver les applications f : Gln(C) → C continues, telles que ∀A, B ∈ Gln(C) f(A.B) = f(A) + f(B).

Solution : 1) Les fonctions cherchées sont de la forme f(z) = a.ln |z| , où a ∈ C. En effet : • La restriction de f à R*+ est un morphisme continu de R*+ dans C. Elle est donc de la forme f(r) = a.ln r , où a ∈ C. • La restriction de f à U est le morphisme trivial, car f(U) est un sous-groupe additif compact de C, donc f(U) = {0}. • Conclure via la représentation trigonométrique d’un complexe.

2) Les fonctions cherchées sont de la forme f(A) = a.ln | det A | , où a ∈ C.

Page 29: Exercices équations fonctionnelles

29

D’abord, λ ∈ C* → f(diag(λ, 1, …, 1)) satisfait à 1), donc (∃a) (∀λ) f(diag(λ, 1, …, 1)) = a.ln|λ|. De plus, si A et B sont semblables, f(A) = f(B). Donc f(diag(λ, 1, …, 1)) = f(diag(1, …, 1, λ, 1, …, 1)) = a.ln |λ| pour tout λ ∈ C*.

Finalement si A = diag(λ1, λ2, …, λn) , f(A) = a∑ iλln = a ln |det A|.

Conclure par densité des diagonalisables inversibles dans Gln(C).

Exercice 16 : sous-groupes à un paramètre de Gln(R).

1) Soit A∈Mn(R) ; montrer que l’application f : t ∈ R → exp(tA) ∈ Gln(R) est continue vérifiant :

∀(s, t) f(s + t) = f(s).f(t).

2) Soit t ∈ R → M(t) ∈ Mn(R) une application continue vérifiant ∀(s, t) M(s + t) = M(s).M(t) , et

M(0) = I. Montrer ∃A ∈ Mn(R) (∀t ∈ R) M(t) = exp(tA).

Solution : 1) C’est du cours ! 2) La méthode d’Euler s’applique si l’on montre au préalable que M est de classe C

1.

Or Gln(R) est ouvert, donc (∃α > 0) ∀P ∈ Mn(R) || P − I || ≤ α ⇒ P ∈ Gln(R).

Par continuité de M, (∃η > 0) |t| ≤ η ⇒ || M(t) − I || ≤ α. Par sommes de Riemann, P = η1 ∫

η

0).( dttM

appartient à l’enveloppe convexe fermée de { M(t) ; 0 ≤ t ≤ η }. Comme B(I, η) est convexe fermée, || P − I || ≤ α, donc P ∈ Gln(R).

Intégrons M(s + t) = M(s).M(t) en t∈[0, η]. Il vient ∫+ηs

sduuM ).( = M(s).∫

η

0).( dttM , donc M est C

1.

Dérivons en s : M’(s + t) = M’(s).M(t) ; en particulier M’(t) = A.M(t), où A = M’(0). On conclut que M(t) = exp(tA) par la méthode de variation des constantes : t → M(t).exp(−tA) a une dérivée nulle et vaut I en 0.

5. Equations récursives.

Exercice 1 : Montrer que les espaces vectoriels suivants sont isomorphes :

F = { f ∈ F(]1, +∞[, R) ; (∀x > 1) f(x) = f(x2) }

G = { g ∈ F(]0, +∞[, R) ; (∀y > 0) g(y) = g(2y) } H = { h ∈ F(R, R) ; h est ln 2 - périodique }

K = { k ∈ F(]0, 1[, R) ; ∀x ∈ ]0, 1[ k(x) = k(x2) }.

Solution : Il est immédiat que : L’application qui à f ∈ F associe k définie par k(t) = f(1/t) est un isomorphisme de F sur K. L’application qui à f ∈ F associe g définie par g(y) = f(exp y) est un isomorphisme de F sur G. L’application qui à g ∈ G associe h définie par h(z) = g(exp z) est un isomorphisme de G sur H.

N.B. : 1) C’est un simple résultat de transmutation ou conjugaison. 2) On peut adjoindre des hypothèses de continuité ou de dérivabilité.

Exercice 2 : Trouver les fonctions continues f : R → R telles que (∀x) f(sin x) = f(x).

Solution : Pour tout x, notons (un) la suite u0 = x, un+1 = sin un. Il est facile de montrer que cette suite tend toujours vers 0. De sorte que f(x) = f(0) : f est constante.

En revanche, si l’on cherche toutes les fonctions f : R → R telles que (∀x) f(sin x) = f(x), alors f est 2π-périodique et telle que f(π − x) = f(x). Je dis que f est entièrement définie par sa restriction g à {0} ∪]1, π/2], restriction qui est arbitraire. Le détail de la preuve est laissé au lecteur.

Page 30: Exercices équations fonctionnelles

30

Exercice 3 : Trouver les fonctions f : R → R continues et telles que (∀x) f(x + 1) – f(x) = x2.

Solution : L’équation fonctionnelle est affine. Cherchons une solution particulière f vérifiant f(x + 1) − f(x) = x

2, sous la forme d’un polynôme de

degré 3. On trouve f0(x) = 3

3x −

2²x +

6x . Par soustraction, f est solution si et seulement si f − f0 = p

est continue 1-périodique. En résumé, les solutions sont f = f0 + p, où p est continue 1-périodique. Elles forment un sous-espace affine de C(R, R), de dimension infinie, ce qui est naturel car l’équation fonctionnelle est linéaire : elle s’écrit ∆ f = x

2, où ∆ est l’opérateur aux différences finies.

Voir aussi mon Farrago.

Exercice 4 : Trouver toutes les fonctions f : R → R dérivables en 0, telles que (∀x) f(2x) = 2 f(x).

Solution : On a f(0) = 0 et f(2n

x) = 2n

f(x), donc f(x) = n

nxf2

)2( → ax, où a = f’(0), quand n → +∞.

Par suite f(x) = ax. Réciproque obvie.

Exercice 5 : Trouver toutes les fonctions f : R → R dérivables en 1, telles que (∀x) f(x2) = f(x)

2.

Solution : laissée en exercice. Exercice 6 : Trouver une fonction continue strictement croissante f : R → R telle que

(∀x) f(x2) = f(x)

4.

Solution : [ Source : Makarov, etc., III.5, ex. 5.12 ] 1) Analysons le résultat. a) On a f(0) = 0 ou 1, f(1) = 0 ou 1, donc f(0) = 0, f(1) = 1. b) Supposons donnée une fonction continue strictement croissante f sur R+ telle que f(0) = 0 et

(∀x ≥ 0) f(x2) = f(x)

4. Alors son unique prolongement impair répond à la question.

c) Supposons donnée une fonction continue strictement croissante g sur [1, +∞[ telle que g(1) = 1 et (∀x ≥ 1) g(x

2) = g(x)

4. Alors la fonction f définie sur R+ par :

f(x) = g(x) si x ≥ 1 , f(x) = 1/g(1/x) si 0 < x ≤ 1 remplit les conditions de b). [ En particulier, g tend vers +∞ en +∞, donc f tend vers 0 en 0+ ]

2) Analyse (suite). Tout revient à trouver une fonction g comme en 1.c). Posons x = e

y , il vient (∀y ≥ 0) g(e

2y) = g(e

y)4

, puis ln g(e2y

) = 4 ln g(ey) ,

puis ln ln g(e2y

) = ln 4 + ln ln g(ey) .

Autrement dit, φ(y) = ln ln g(ey) vérifie φ(2y) = φ(y) + ln 4.

On peut creuser encore le sujet. Mais on peut aussi noter que φ(y) = 2 ln y répond à la question.

Cela fournit la fonction g(x) = )²(lnxe .

3) Synthèse. La fonction obtenue, qui n’est certes pas la seule, est donc définie par :

f(x) = )²(lnxe− si 0 < x ≤ 1 , f(x) = )²(lnxe si 1 ≤ x. Enfin, f est impaire.

En résumé, on trouve : f(x) = sgn(x).x

xln

.

> f:=x->signum(x)*abs(x)^(abs(ln(abs(x)))); > plot(f(x),x=-3..3,thickness=2,color=green);

Page 31: Exercices équations fonctionnelles

31

Exercice 7 : Trouver toutes les fonctions f : R → R dérivables en 0, telles que :

(∀x) f(2x) = 2 f(x) )²(1 xf+ .

Solution . [ Oral Mines 2001 ] 0) f(x) ≡ 0 et f(x) ≡ sh x conviennent. 1) Posons f(x) = sh g(x), i.e. g(x) = Argsh f(x). g est dérivable en 0 comme composée. De plus, sh g(2x) = 2.sh g(x).ch g(x) = sh(2g(x)), donc g(2x) = 2g(x). En vertu de l’exercice 4, g(x) = ax, donc f(x) = sh(ax) . Réciproque obvie.

Exercice 8 : On cherche les fonctions f : R → R dérivables en 0, vérifiant (∀x) f(2x) = )²(1)(2

xfxf

+ .

1) Quelles valeurs peut prendre f ? Montrer que (∀x) | f(x) | ≤ 1.

2) On suppose f(0) = 0. Montrer que (∀x) f(x) < 1 , puis que (∃a) (∀x) f(x) = th(ax).

3) On suppose f(0) = 1. Montrer que ∀ε ∈ ]0, 1[ ∃η > 0 |x| ≤ η ⇒ f(x) ≥ 1 − ε. Etablir que (∀x) f(x) ≥ 1 − ε . En déduire f = 1. Cas où f(0) = −1 ?

Solution : [ Oral X 1983, RMS n° 37 ]

Exercice 9 : On étudie les fonctions f : R → R vérifiant (∀x) f(2x) = 2f(x)2 − 1 (E).

1) Indiquer 4 fonctions élémentaires vérifiant cette équation fonctionnelle.

2) Soit h(x) = 2x. Une partie A de R est dit h-stable si h(A) = A. Indiquer comment fabriquer de nombreuses fonctions f vérifiant (E).

3) Montrer qu’il existe une et une seule fonction f vérifiant (E), telle que f(0) = 1 et deux fois dérivable en 0 et telle que f(0) = 1.

Solution : 1) Exemples de solutions.

Page 32: Exercices équations fonctionnelles

32

Les fonctions constantes solutions sont f(x) = 1 et f(x) = −1/2. Ce sont d’ailleurs les seules solutions polynomiales. Il y a aussi les fonctions f(x) = ch x et f(x) = cos x, plus généralement f(x) = ch(αx) et f(x) = cos(βx). On peut fabriquer des exemples plus compliqués. Constatons que f(0) = 1 ou –1/2.

cos(αx) ou ch(αx) ou −1/2 pour x > 0 f(x) = 1 ou –1/2 pour x = 0 cos(βx) ou ch(βx) ou −1/2 pour x < 0

cos(αx) ou ch(αx) ou −1/2 pour x ∈ Q* f(x) = 1 ou –1/2 pour x = 0 cos(βx) ou ch(βx) ou −1/2 pour x ∉ Q

cos(αx) ou ch(αx) ou −1/2 pour x ∈ A* f(x) = 1 ou –1/2 pour x = 0 cos(βx) ou ch(βx) ou −1/2 pour x ∉ A , où A est l’anneau des rationnels dyadiques.

2) Peut-on décrire toutes les solutions de (E) ? Notons que si f existe, f(0) = 1 ou –1/2 et f est à valeurs dans [−1, +∞[.

Considérons, dans la droite réelle R, la relation d’équivalence x R y ⇔ ∃n ∈ Z y = 2n x.

Cette relation d’équivalence admet une infinité de classes d’équivalence : une classe est formée du singleton {0}, les autres rencontrent ]−2, −1] ∪ [1, 2[ en un et un seul point. On peut donc indexer ces classes d’équivalence par D = ]−2, −1] ∪ {0} ∪ [1, 2[. Pour se donner f, il suffit de se donner sa restriction à chacune des classes. Tout d’abord f(0) = 1 ou –1/2.

Soit maintenant x0 un élément de D’ = ]−2, −1] ∪ [1, 2[. Donnons-nous la valeur de f(x0).

• Si f(x0) > 1, posons f(x0) = ch θ0 , θ0 > 0. Je dis que ∀n ∈ Z f(2n x0) = ch(2

n θ0).

Cela se montre par récurrence sur n .

La récurrence montante ne pose aucun problème : f(2n x0) = ch(2

n θ0) implique aussitôt f(2

n+1 x0) =

… = ch(2n+1

θ0).

La récurrence descendante est plus délicate : f(2n x0) = ch(2

n θ0) implique f(2

n−1 x0) = ± ch(2

n−1 θ0),

mais comme f est à valeurs ≥ −1, nécessairement f(2n−1

x0) = ch(2n−1

θ0).

• Si |f(x0)| ≤ 1, posons f(x0) = cos θ0 , θ0 ∈[0, π]. Je dis que ∀n ∈ N f(2n x0) = cos(2

n θ0).

3) Soit f une solution de (E) deux fois dérivable en 0 et telle que f(0) = 1.

Par Taylor-Young, f(x) = 1 + ax + bx2 + o(x

2) au V(0), où a = f’(0) et b =

2)0(''f

.

Reportant dans (E), il vient a = 0, donc f’(0) = 0.

• Supposons d’abord b > 0, et posons b = ω2, ω > 0. Alors f(x) > 1 pour 0 < |x| ≤ r , r > 0.

Comme f(x) > 1 implique aussitôt f(2x) > 1, on a f(x) > 1 pour tout x ≠ 0. Posons alors f(x) = ch g(x), ou plutôt g(x) = Argch f(x) ; g est à valeurs ≥ 0. ch g(2x) = f(2x) = 2.f(x)

2 – 1 = 2.ch g(x) – 1 = ch(2g(x)) , donc g(2x) = 2g(x).

De plus, g est dérivable en 0, car g(x) = Argch f(x) = Argch(1 + ω2 x

2) = 1 + f(x

Exercice 10 : Trouver les fonctions f : R → R dérivables en 0 et telles que (∀x) f(2x) = 2 f(x).cos x.

Solution : 1) On songe aussitôt à la fonction sin, aux fonctions a.sin (car les fonctions considérées forment un espace vectoriel).

Page 33: Exercices équations fonctionnelles

33

2) Montrons la réciproque. D’abord f(0) = 0. Notons a = f’(0).

Par itérations, il vient f(2n

x) = 2n.f(x).cos(x).cos(2x) … cos(2

n−1x) ; cela se vérifie par récurrence.

Multiplions par sin(2n−1

x) : f(2nx).sin x = f(x).sin(2

n x), d’où f(x).sin n

x2

= f( nx2

).sin x.

Fixons x ; f(x) = )2/sin(

)2/(n

n

xxf

sin x → a.sin x quand n → +∞. Donc f(x) = a.sin x.

Exercice 11 : Trouver les fonctions continues f : R → R telles que (∀x) f(x) = f(2x + 1).

Solution : Notons s(x) = 2x + 1 pour tout réel x. On a f(x) = f(s(x)) = f(sn(x)) pour tout n ∈ Z.

Or s est l’homothétie de centre –1 et de rapport 2, donc s−1

= Hom(–1, 1/2).

Lorsque n → − ∞ , sn(x) tend vers –1. Donc f(x) = f(−1) : f est constante.

Plus généralement, si (E, d) est un espace complet, et s une application contractante de E dans E, les fonctions continues E → E vérifiant f(x) = f(s(x)) sont les fonctions constantes. Exercice 12 : Trouver les fonctions f : R → R continues en 0 et telles que :

• (∀x) f(x) = f(2x ) +

2x . • (∀x) f(x) = f(

2x ) −

)4²)(1²(²3

++ xxx .

Solution : Les deux équations sont affines.

• La première admet pour solution particulière f0(x) = x. Du coup, g = f – f0 vérifie

g(x) = g(2x ) = … = g( n

x2

) → g(0) par continuité de g en 0. Donc f(x) = x + cte.

• La seconde admet pour solution particulière f0(x) = 1²

1+x

(décomposer en éléments simples).

Comme ci-dessus, g = f – f0 est constante, et f(x) = 1²

1+x

+ cte.

Exercice 13 : Soit D = R − {−3/2}. Trouver les fonctions f : D → R continues en −½ , et telles que :

(∀x ∈ D) f(x) = f (6412

+−

xx ).

Solution : Exercice 14 : Soit D = R −{ −1, −2}. Trouver les fonctions f : D → R continues en 0, et telles que :

(∀x ∈ D) f(x) = f(2x ) +

)2)(1( ++ xxx . [ Oral X 1984 ]

Solution.

1) Cherchons d’abord les fractions rationnelles F ∈R(X) telles que F(X) = F(2X ) +

)2)(1( ++ XXX .

F(X) +1

1+X

= F(2X ) +

22+X

, autrement dit la fraction G(X) = F(X) +1

1+X

vérifie G(X) = G(2X ).

On en déduit pour tout n, que G(X) = G( nX2

) . La fonction rationnelle associée prend une infinité de

fois la valeur G(1), donc G est constante et F(X) = c − 1

1+X

.

Page 34: Exercices équations fonctionnelles

34

2) Dans le cas général, f(x) +1

1+x

= f(2x ) +

22+x

, autrement dit la fonction g(x) = f(x) + 1

1+x

vérifie g(x) = g(2x ), et g est continue en 0. Donc g est constante et f(x) = c −

11+x

.

Il y a cependant un problème de domaine de définition. Le domaine de définition D de f est tel que x ∈ D ⇒ x/2 ∈ D et −1 et −2 ∉ D. Donc D ⊂ R − {−1, −2, −4, … }.

Exercice 15 : Trouver les f : R → R continues et telles que (∀x) f(x)3 − 6.f(x)

2 + 11.f(x) – 6 = 0.

Solution : On a, pour tout x, f(x) ∈ {1, 2, 3}. Or le théorème des valeurs intermédiaires impose que f est constante.

Exercice 16 : Trouver les fonctions f : R → R continues et telles que (∀x) f(x)2 − x.f(x) – 1 = 0.

Solution : Par résolution de l’équation du second degré, on trouve f(x) = 2

4²+± xx ,

ou plutôt f(x) = 2

4²).( ++ xxx ε, où ε est une fonction continue sur R, à valeurs dans {±1}.

En vertu du théorème des valeurs intermédiaires, ε est constante. Il y a deux fonctions solutions.

Exercice 17 : Trouver les fonctions f : R* → R

* continues et telles que (∀x) f(x) +

)(1xf

= x +x1 .

Solution : Par résolution d’une équation du second degré, on trouve f(x) = x ou 1/x. Mais un examen des graphes de ces fonctions suggèrent, non pas 2, mais 16 fonctions continues solutions, se raccordant toutes en 1. Par exemple sur R*+, on trouve f1(x) = x, f2(x) = 1/x,

f3(x) = x si 0 < x ≤ 1, 1/x si 1 ≤ x, f4(x) = 1/x si 0 < x ≤ 1, x si 1 ≤ x.

De même, il y a 4 possibilités R*−, d’où 4×4 = 16 possibilités. Comme dans l’exercice précédent, une justification rigoureuse passe par le théorème des valeurs intermédiaires. Par exemple si 0 < x < 1, f(x) ≠ 1, donc, soit f(x) < 1 pour tout x ∈ ]0, 1[ et alors f(x) = x, soit f(x) > 1 pour tout x ∈ ]0, 1[ et alors f(x) = 1/x. Idem si x > 1, etc.

Exercice 18 : Trouver les fonctions f : R → R de classe C1, telles que (∀x) ( f o f )(x) = 3 +

2x .

Même problème en supposant seulement f dérivable au point 6.

Solution : 1) Il est naturel de chercher les fonctions affines solutions f(x) = ax + b.

On en trouve deux (comme papa) : a = ε22 , b =

13+a

où ε = ±1.

2) ( f o f ) o f = f o ( f o f ) donne f(3 + 2x ) = 3 +

2)(xf

, i.e. f o T = T o f , où T(x) = 3 + 2x .

3) Si f est C1, dérivons ! Il vient

21 f’ (3 +

2x ) =

2)(' xf

, donc f’ (3 + 2x ) = f’ (x) et f’ est constante.

Donc f est affine.

4) Si f est seulement dérivable en 6, point fixe de T, noter que Tn

o f = f o Tn et faire quelques

calculs.

Exercice 19 : Trouver les fonctions continues f : R → R telles que (∀x) ( f o f )(x) = − x.

Page 35: Exercices équations fonctionnelles

35

Solution : Il y a bien une fonction f vérifiant la condition requise : c’est f(x) = ix... mais elle n’est pas à valeurs réelles ! On subodore qu’il n’y a pas de fonction réelle vérifiant cette condition. Indeed, f serait continue et bijective, donc strictement monotone. Mais alors, dans tous les cas, f o f serait strictement croissante… or elle est strictement décroissante ! La même conclusion subsiste si l’on remplace –x par n’importe quelle injection décroissante R → R.

Exercice 20 : Trouver une cns portant sur a pour qu’il existe une fonction f : R → R continue et telle que (∀x) ( f o f )(x) = ax + b.

Solution : simple généralisation de l’exercice précédent. La cns est a ≥ 0. • Si a < 0, f n’existe pas. En effet, f serait une bijection continue, donc strictement monotone ; f o f serait strictement croissante, or elle décroît ! • Si a = 0 , f existe : prendre f(x) = x + b/2.

• Si a > 0 , f existe : prendre f(x) = cx + d, où c = a , d = 1+c

b .

Exercice 21 : Soient a et b deux réels, h(x) = ax + b, S l’ensemble des fonctions dérivables f : R → R telles que f o f = h. 1) Montrer que si a < 0, S est vide. Désormais on suppose a > 0 et a ≠ 1. 2) Montrer que h est une homothétie ; préciser son centre et son rapport.

3) Soit f ∈ S. Montrer que h−1

o f o h = f. En déduire une expression de f ; commencer par 0 < a < 1.

Solution :

Exercice 22 : Soient a et b > 0, a + b = 1, E = C(R, R). A toute f ∈ E on associe T f : x → f(ax + b). 1) Montrer que toute valeur propre λ de T vérifie λ ∈ ]−1, 0[ ∪ ]0, +1]. 2) Déterminer Ker(T – I).

3) Soit |λ| < 1. Montrer que λ est valeur propre de T ; trouver les fonctions f ∈ Ker(T − λI) de

classe C∞

(On montrera que ce sont des polynômes).

Solution :

0) L’application affine s : x → ax + b a un unique point fixe ω = a

b−1 , et l’on a, pour tout n ∈ Z et

tout réel x : sn(x) = ω + a

n ( x − ω ). Du coup, s

n(x) → ω quand n → +∞. [ s est contractante ! ].

1) λ est valeur propre de T ss’il existe f ≠ 0 telle que T f = λf. • λ ≠ 0, car T f = 0 ⇒ f o s = 0 ⇒ f = 0 (s est bijective). En fait, T est linéaire injective, et même bijective. • |λ| > 1 impliquerait T

n f = λn

.f , donc (∀x) λn.f(x) = ( f o s

n )(x) → f(ω). On en déduit f(x) = 0.

Impossible ! • λ = −1 est également impossible, car on aurait (∀x) (−1)

n.f(x) = ( f o s

n )(x) → f(ω), ce qui impose

par absurde f(x) = 0. 2) Je dis que Ker(T – Id) = { fonctions constantes }. En effet, Tf = f ⇔ f o s = f ⇒ (∀n) f o s

n = f . Alors f(x) = f(s

n(x)) → f(ω) quand n → +∞, donc f est

constante. Réciproque évidente.

3) Soit 0 < |λ| < 1. Cherchons f ∈ Ker(T − λI) − {0} de classe C∞

.

λn f(x) = f(ω + a

n (x − ω)) = f(ω) + a

n (x − ω) f’(ω) + o(a

n).

Cela impose f(x) = 0.

Exercice 23 : On considère les fonctions f : [0, 1] → R vérifiant ∀x ∈ [0, 1] f(x) = f(2x ) + f(

21+x ).

Page 36: Exercices équations fonctionnelles

36

a) Montrer que si f est de classe C2, f est de la forme f(x) = a (1 – 2x).

b) Montrer que ce résultat subsiste si f est de classe C1.

c) Montrer que ce résultat tombe en défaut si f est continue.

Solution : [ Source : Makarov, etc. , III.5, ex. 5.9 ]

a) Dérivons deux fois ! Il vient f’’( x) = 41 f’’(

2x ) +

41 f’’(

21+x ).

Notant M = || f’’ ||∞, on a aussitôt M ≤ 21 M, donc M = 0 ; f est affine, etc.

Variante : Une récurrence sur n montre que f’’( x) = n41 ∑

≤≤

+nk

nkxf

21

)2

('' .

Or cela tend vers 0 par sommes de Riemann ou par majoration (f’’ est bornée). Donc f’ est constante, etc.

b) Dérivons une fois ! Il vient f’(x) = 21 f’(

2x ) +

21 f’(

21+x ), d’où f’(x) = n2

1 ∑≤≤

+nk

nkxf

21

)2

(' , par

récurrence. Comme nk2

≤ nkx

2+ ≤ n

k2

1+ , n21 ∑

≤≤

+nk

nkxf

21

)2

(' → ∫1

0).(' dxxf par sommes de Riemann.

Donc f’ est constante, etc. Autre idée : si f’(a) = max f’(a), f’( nka

2+ ) = f’(a) pour tous k et n, donc f’

est constante sur une partie dense de [0, 1]. c) Les fonctions continues sur [0, 1] vérifiant l’équation fonctionnelle ne sont pas toutes affines.

Ainsi le lecteur s’assurera que f(x) = ∑+∞

=0

)2sin(21

n

nn xπ répond à la question.

Remarque : on trouvera des compléments sur le sujet dans le pb. d’Agrégation interne 1996.

Exercice 24 : Trouver les fonctions continues f : R → R telles que (∀x) ( f o f )(x) = exp x. [ Oral ENS 2007 ]

Solution : Il s’agit en quelque sorte de trouver une « racine carrée fonctionnelle » de l’exponentielle. Si j’en crois l’érudition de mon vieux maître L.-G. Vidiani, ce problème fut résolu en son temps par le grand mathématicien anglais G. H. Hardy.

Analyse du problème. Si f existe : i) f est injective, car f(x) = f(y) ⇒ ( f o f )(x) = ( f o f )(y) ⇒ exp x = exp y ⇒ x = y. ii) f est continue injective sur R, donc strictement monotone. iii) On a (∀x) ( f o f o f )(x) = f(exp x) = exp(f(x)).

iv) On en déduit (∀y > 0) f(y) = exp(f(ln y)) . Par continuité, f(0) ≥ 0.

v) De plus, f(1) = exp f(0) ≥ f(0) + 1 > f(0). Donc f est strictement croissante. vi) f est sans point fixe, car f(x) = x impliquerait x = ( f o f )(x) = exp x : impossible.

vii) En vertu du théorème des valeurs intermédiaires, on a, soit (∀x) f(x) < x , soit (∀x) f(x) > x. Comme f(0) ≥ 0, on a (∀x) f(x) > x. viii) Notons a = f(0) . Je dis que 0 < a < 1. 0 < f(0) découle de vii). Et 1 ≤ a impliquerait f(1) ≤ f(a) = f(f(0)) = 1, contredisant vii).

ix) Notons α et β les limites respectives de f(x) en −∞ et +∞ . (∀y > 0) f(y) = exp f(ln y) donne, quand y → +∞, β = exp β ; cela impose β = +∞. Si l’on fait x → −∞ dans la relation f(exp x) = exp(f(x)), il vient : f(0) = exp α, donc α = ln a < 0.

Synthèse : Soient a un réel, 0 < a < 1, g une bijection continue strictement croissante [0, a] → [a, 1].

Définissons les suites (an)n≥−1 et (bn)n≥−1 par a0 = 0 , an+1 = exp(an) , b0 = a , bn+1 = exp(bn) .

Page 37: Exercices équations fonctionnelles

37

On a : a−1 = −∞ < b−1 = ln a < a0 = 0 < b0 = a < a1 = 1 < b1 = exp a < a2 < b2 < …

réindexons c−2 c−1 c0 c1 c2 c3 c4 c5

A noter que, pour tout n, cn+2 = exp(cn).

Définissons gn : [cn , cn+1] → [cn+1 , cn+2] par récurrence, ainsi :

• Si n = 0 , g0 = g ;

• Si gn : [cn , cn+1] → [cn+1 , cn+2] est une bijection continue croissante, alors

gn+1(x) = exp((gn)−1

(x)) définit une bijection continue croissante [cn+1 , cn+2] → [cn+2 , cn+3] ;

• g−1(x) = (g0)−1

(exp x) définit une bijection continue croissante [c−1 , c0] → [c0 , c1] ;

• g−2(x) = (g−1)−1

(exp x) définit une bijection continue croissante [c−2 , c−1] → [c−1 , c0] ;

Le prolongement commun f de toutes les gn vérifie, pour tout x, (f o f)(x) = exp x.

Cela se vérifie pour tout n et tout x ∈ [cn , cn+1].

Exercice 25 : 1) Trouver les fonctions f : [0, 1] → [0, 1] telles que (∀x) ( f o f )(x) = 2f(x) − x.

2) Trouver toutes les fonctions continues f : R → R telles que (∀x) ( f o f )(x) = 2f(x) − x. [On pourra montrer qu’elles sont bijectives.] [ Oral Centrale 1991 ]

Solution. 1) La seule fonction f : [0, 1] → [0, 1] telle que (∀x) (f o f)(x) = 2f(x) − x est l’identité.

En effet, on a f n+2

(x) − 2f n+1

(x) + f n(x) = 0, pour tout n ∈ N et tout x.

On en déduit f n(x) = (f(x) – x).n + x pour tout n ∈ N (par récurrence ou équation caractéristique).

La suite (f n(x)) étant bornée, cela impose f(x) = x, pour tout x. Même résultat si f : [a, b] → [a, b].

2) Le cas réel est nettement plus difficile. • Les seules fonctions affines solutions sont les translations x → x + c. Nous allons montrer qu’il n’y a pas d’autres solutions.

• Soit f continue R → R telle que, pour tout x, (f o f)(x) = 2f(x) – x.

Lemme 1 : f est un homéomorphisme croissant de R sur R, et f−1

satisfait la même équation que f.

Preuve : f est injective, car f(x) = f(y) ⇒ x = 2 f(x) – f(f(x)) = 2 f(y) – f(f(y)) = y. Du coup, f est strictement monotone. Si f décroît, alors x → (f o f)(x) + x = 2 f(x) croît : contradiction ! Donc f croît. f(x) a une limite L, finie ou infinie, en +∞. Si L était finie, on aurait f(L) = 2L − ∞ : impossible !

Donc lim+∞ f(x) = +∞ . Idem en −∞. f est donc une bijection continue strictement croissante de R dans R. Si g est la bijection réciproque de f , on a, pour tout y : y = 2g(y) – (g o g)(y).

Lemme 2 : On a f n(x) = (f(x) – x).n + x pour tout n ∈ Z et tout x ∈ R.

Preuve : f n+2

(x) − 2f n+1

(x) + f n(x) = 0 , pour tout n∈N, donc f

n(x) = (f(x) – x).n + x pour tout n∈N.

La bijection réciproque de f vérifiant la même équation fonctionnelle que f , on a, pour tout x :

x = 21 [f(x) + f

−1(x)] , et aussi f

n+2(x) − 2f

n+1(x) + f

n(x) = 0 , pour tout n ∈ Z.

Elle implique aussitôt f n(x) = (f(x) – x).n + x pour tout n ∈ Z et tout x ∈ R.

Lemme 3 : Pour tout x ∈ R, Φ(x) ≡ limn→+∞ nxf n )(

= f(x) – x .

Φ est une fonction croissante telle que (∀x ∈ R) Φ(f(x)) = Φ(x).

Page 38: Exercices équations fonctionnelles

38

Preuve : nxf n )(

→ f(x) – x en vertu du lemme 2.

La fonction Φ est croissante comme limite simple de fonctions croissantes, les nf n

.

Enfin, Φ(f(x)) = f(f(x)) – f(x) = f(x) – x ; cela découle aussi de ce que n

xf n )(1+ =

1)(1

++

nxf n

nn 1+ .

Lemme 4 : La fonction Φ est constante.

Solution : On vient de voir que Φ(x) = f(x) – x est croissante.

Or f−1

satisfait à la même équation fonctionnelle que f. Donc Ψ(x) = f−1

(x) – x est aussi croissante. Or Φ(x) = − Ψ(f(x)) . Donc Φ est décroissante. Donc Φ est constante. Cqfd.

Conclusion : Les fonctions cherchées sont de la forme f(x) = x + cte.

Source : Oral Centrale 1991. La RMS juin 1992, p. 537, en donne un corrigé fort maladroit.

Exercice 26 : Trouver les fonctions f : R+ → R

+ telles que (∀x ≥ 0) ( f o f )(x) = 6x − f(x).

[ Oral ENS 2003 ]

Solution : Soit x ≥ 0. La suite u0 = x, un+1 = f(un) vérifie un+2 + un+1 – 6un = 0 : elle est récurrente

linéaire. On trouve un = a.2n + b.(−3)

n . Mais un ≥ 0 pour tout n ; cela impose b = 0. D’où u1 = 2u0 et

f(x) = 2x. Réciproque immédiate.

Remarque : voici une généralisation immédiate, proposée aux Olympiades 1992 : Soient a, b deux réels > 0. Prouver qu’il existe une unique fonction f : R

+ → R

+ telle que, pour tout

réel x ≥ 0 : f(f(x)) + a f(x) = b ( a + b ) x.

Exercice 27 : Trouver les fonctions f ∈ C(R, R) telles que (∀x) (f o f o f)(x) − 3(f o f)(x) + 6 f(x) = 4x.

Solution : [Oral Mines 2008 ] 1) Tout d’abord, f(x) = x est solution. De plus, c’est la seule fonction affine f(x) = ax + b solution. C’est même la seule fonction polynomiale solution pour des raisons de degré. 2) Si f est solution, f est injective, donc elle est strictement monotone. Ecrivons pour tout x : ( f o f o f )(x) + 6 f(x) = 3 ( f o f )(x) + 4x . Le second membre est une fonction strictement croissante de x, le premier membre est croissant si f est croissante, décroissant si f est décroissante. Donc f est croissante. Soit L la limite de f en +∞. Si L était finie on aurait ( f o f o f )(x) − 3 ( f o f )(x) + 6 f(x) → 4L ; or 4x tend vers +∞. Donc L = +∞. De même, f tend vers −∞ quand x tend vers −∞. En résumé, f est un homéomorphisme croissant de R sur R. 3)

Exercice 28 : 1) Trouver les fonctions continues f : R → R telles que (∀x) f(x) + f(2x) = 0.

2) Soient a, b, c trois réels > 0 distincts. Trouver les fonctions f : R → R de classe C∞

telles que (∀x) f(ax) + f(bx) + f(cx) = 0. [ Oral ENS 2007 ]

Solution : 1) Tout d’abord, x = 0 donne f(0) = 0 . Puis f(x) = − f(x/2), donc par récurrence f(x) = (−1)

n.f(x/2

n)

Fixons x et faisans tendre n vers l’infini. Il vient f(x) = 0 : f est la fonction nulle. Généralisation immédiate aux fonctions continues f : R → R telles que (∀x) f(ax) + f(bx) = 0, avec 0 ≤ |a| < |b|. 2) Supposons 0 < a < b < c.

Page 39: Exercices équations fonctionnelles

39

♣ Dérivons p fois. Il vient ap f

(p)(ax) + b

p f

(p)(bx) + c

p f

(p)(cx) = 0.

x = 0 donne f(p)

(0) = 0 pour tout p : f est une fonction « plate » en 0.

En vertu du théorème de Taylor-Young, pour tout p , f(x) = o(xp) au V(0).

Attention ! en général, une fonction plate n’est pas forcément nulle. Mais nous avons de bonnes raisons de conjecturer qu’ici elle est nulle.

♦ On a, pour tout n et tout x, f(cnx) = (−1)

n ∑

=

−n

k

knkkn xbafC0

)( .

Cela se montre par récurrence sur n, ou se déduit de la formule du binôme, car les opérateurs Pa et Pb

commutent, où (Pλ f)(x) = f(λx) . Du coup, f(x) = (−1)n ∑

=

−n

k

knkkn xcb

cafC

0

))()(( .

♥ Fixons x. Choisissons p tel que p

pp

cba +

≤ 1.

Comme f(x) = o(xp) au V(0), on a : ∀ε > 0 ∃α > 0 |y| ≤ α ⇒ | f(y) | ≤ ε |y|

p.

Alors ∃n0 ∀n ≥ n0 (cb )n

|x| ≤ α.

Du coup, ∀n ≥ n0 ∀k ∈ [0, n] (ca )k (

cb )n−k

|x| ≤ (cb )n

|x| ≤ α .

| f(x) | ≤ ε ∑=

−n

k

knppkkncb

caC

0

)()()( |x|p = ε ( p

pp

cba +

)n |x|

p .

x, p et ε sont fixés, n est ≥ n0. Si l’on fait tendre n vers +∞, il vient f(x) = 0.

♠ Conclusion : f est la fonction nulle.

Exercice 29 : Trouver les fonctions continues f : R → R telles que ∀(x, y) f(x + y) = f(x + f(y)).

Solution : 1) Les fonctions constantes répondent à la question, ainsi que f(x) = x. 2) Notons G l’ensemble des périodes de f ; c’est un sous-groupe additif fermé de R. Il est clair que, pour tout y, f(y) – y ∈ G. • Si G = {0}, alors f(y) = y pour tout y. • Si G = R, f est constante. • Sinon, G est de la forme aZ (a > 0). En vertu du théorème des valeurs intermédiaires, f(y) – y est constante : f(y) = y + c. Reportant, il vient x + y + c = x + y + 2c, donc c = 0 : f(y) = y… et G = {0}.

Exercice 30 : Trouver les fonctions continues f : R → R telles que ∀(x, y) f(x2 + f(y)) = y + f(x

2).

[ Oral X 2001 ]

Solution : 0) La fonction f(x) = x est solution… 1) y = 0 donne f(x

2 + f(0)) = f(x)

2 (*) , x = 0 donne f(f(y)) = y + f(0)

2 (**).

2) De (**) on déduit que f o f est bijective ; donc f l’est aussi. Du coup, f est strictement monotone.

3) De (*) on déduit que f(x)2 = f(−x)

2 , donc f(−x) = ± f(x).

Mais l’injectivité de f implique que f est impaire. 4) Récapitulons : f(x

2) = f(x)

2 , (f o f)(y) = y , f(R+) ⊂ R+ et f(R*+) ⊂ R*+.

f est strictement croissante. 5) x < f(x) ⇒ f(x) < (f o f)(x) = x : impossible ! x > f(x) ⇒ f(x) > (f o f)(x) = x : impossible itou ! Donc, pour tout x, f(x) = x. Variantes possibles. Exercice 31 : Fonctions idempotentes.

Page 40: Exercices équations fonctionnelles

40

1) Soit E un ensemble, f une fonction E → E. Montrer que : f o f = f ⇔ ∀x ∈ f(E) f(x) = x.

2) Si E est un ensemble à n éléments, montrer qu’il y a knn

k

knkC −

=∑

1

. applications de ce type.

3) Trouver toutes les fonctions continues f : [a, b] → [a, b] telles que f o f = f.

Solution : 1) et 2) sont laissés en exercice. 3) Soient J = [c, d] un segment inclus dans [a, b], g : [a, c] → [c, d] une fonction continue telle que g(c) = c, h : [d, b] → [c, d] une fonction continue telle que h(d) = d. La recollée f de g, de h et de idJ répond à la question. Réciproquement, soit f : [a, b] → [a, b] continue telle que f o f = f. J = f([a, b]) est un sous-segment [c, d] de [a, b]. f induit l’identité dans J. Les restrictions g et h de f à [a, c] et [d, b] satisfont aux propriétés précédentes, et f est leur recollée… Exercice 32 : 1) Trouver les fonctions f continues R → R telles que ∀x ∈ R ( f o f )(x) = f(x) + 1. 2) Donner un exemple de fonction non continue R → R telle que ∀x ∈ R ( f o f )(x) = f(x) + 1.

Solution : [ Oral X PC, 2009 ]

1) La condition : ∀x ∈ R ( f o f )(x) = f(x) + 1 équivaut à celle-ci : ∀y ∈ f(R) f (y) = y + 1. L’action de f sur l’ensemble image f(R) est parfaitement claire : f y induit la translation x → x + 1. Il en découle en particulier que l’ensemble image est stable par cette translation.

2) Si f est continue, l’ensemble I = f(R) est un intervalle, par le théorème des valeurs intermédiaires. Et cet intervalle est non majoré. C’est donc, soit R tout entier, soit une demi-droite ouverte ou fermée ] a, +∞[, [ a, +∞[.

• Si I = R, f est la translation x → x + 1. Réciproque immédiate.

• Si I = [ a, +∞[, on a ∀x ≥ a f(x) = x + 1. Reste à définir f sur ]−∞, a [. Notons g la restriction de f à cette demi-droite. Ce doit être une fonction continue, à valeurs dans [a, +∞[, telle que :

limx→a−0 g(x) = a + 1.

De plus, il faut que g prenne toutes les valeurs du segment [a, a + 1].

Réciproquement, si l’on se donne un réel a, et une fonction continue g : ]−∞, a[ → [a, +∞[, telle

que : limx→a−0 g(x) = a + 1 , et prenant toutes les valeurs de [a, a + 1[, alors la fonction f définie par f(x) = x + 1 si x ≥ a, f(x) = g(x) si x < a , répond à la question.

• Si I = ]a, +∞[, on a ∀x > a f(x) = x + 1. Reste à définir f sur ]−∞, a ]. Notons g la restriction de f à cette demi-droite. Ce doit être une fonction continue, à valeurs dans ] a, +∞ [, telle que g(a) = a + 1. De plus, il faut que g prenne toutes les valeurs de l’intervalle ] a , a + 1].

Réciproquement, si l’on se donne un réel a, et une fonction continue g : ]−∞, a] → ]a, +∞[, telle que g(a) = a + 1 , et prenant toutes les valeurs de l’intervalle ] a , a + 1], alors la fonction f définie par f(x) = x + 1 si x > a, f(x) = g(x) si x ≤ a , répond à la question. 3) Revenons au cas général. Il est facile de donner des exemples de fonctions f discontinues vérifiant la condition voulue.

♣ Soit g n’importe quelle fonction de R – Z dans Z, et f définie par : f(x) = x + 1 si x ∈ Z et f(x) = g(x) si x ∈ R − Z . La plus simple est la fonction f(x) = [ x ] + 1. ♦ Soit g n’importe quelle fonction de R – Q dans Q, et f définie par :

Page 41: Exercices équations fonctionnelles

41

f(x) = x + 1 si x ∈ Q et f(x) = g(x) si x ∈ R − Q . ♥ Soit g n’importe quelle fonction de R – N dans N, et f définie par : f(x) = x + 1 si x ∈ N et f(x) = g(x) si x ∈ R − N .

Exercice 33 : Trouver toutes les fonctions continues f : [a, b] → [a, b] telles que f o f o f = Id. [ Oral Mines 2001 ]

Solution : f est une bijection continue, donc un homéomorphisme. f est donc strictement monotone, et nécessairement strictement croissante (sinon id serait décroissante). • x < f(x) impliquerait x < f(x) < (f o f)(x) < (f o f o f)(x) = x ! • x > f(x) impliquerait x > f(x) > (f o f)(x) > (f o f o f)(x) = x ! Donc, pour tout x, f(x) = x. Exercice 34 : Trouver toutes les fonctions f : R → R continues en 0, et telles que :

(∀x) f(2x) = exp(−x2/2).cos f(x) .

Solution : [ Oral Mines 1991 ] 1) x = 0 donne f(0) = cos f(0), donc f(0) = a, où a est l’unique solution de a = cos a. 2) Existence de f. La méthode des approximations successives suggère de considérer la suite de

fonctions : f0(x) = a , fn+1(x) = e−x²/8

cos fn(2x ).

On a (∀n) (∀x) | fn(x) | ≤ 1 et | fn+1(x) − fn(x) | ≤ e−x²/8 | fn(

2x ) − fn−1(

2x ) |.

D’où en itérant : | fn+1(x) − fn(x) | ≤ exp(−

411

411

.8²

− nx ).| f1( n

x2

) − f0( nx2

) | ≤ na4.8

x².

La série ∑ −+ ))()(( 1 xfxf nn est absolument convergente, et normalement convergente sur tout |x| ≤ A.

Sa somme est donc continue sur R, et f(x) = limn fn(x) est définie et continue sur R. Elle satisfait aux conditions imposées. 3) Unicité de f. Soit g une fonction vérifiant les mêmes conditions.

| g(x) − fn+1(x) | ≤ e−x²/8 | g(

2x ) − fn(

2x ) | ≤ exp(−

411

411

.8²

− nx ) | g( n

x2

) − a | ≤ | g( nx2

) − a | → 0.

Donc g(x) = f(x). Remarques : 1) La continuité de f en 0 est indispensable pour assurer l’unicité : on pourrait définir f n’importe comment sur ]1, 2] et [−2, −1[, puis sur ]1/2, 1], [−1, −1/2[, etc.

2) f est paire, tend vers 0 en ±∞, croissante sur ]−∞, 0], décroissante sur [0, +∞[.

Exercice 35 : Trouver toutes les fonctions continues f : R → R telles que (∀x) f(x) = f( x2 + a ),

où a est un réel tel que 0 ≤ a ≤ 1/4. .

Solution : [Olympiades Bulgarie 1997, Oral Mines 2001]

Traitons le cas a = ¼. Soit x ≥ 0, (xn) la suite définie par x0 = x, xn+1 = xn2 + a.

Cette suite tend vers ½ (Faire un dessin, puis mettre en forme !). Par continuité, f(x) = f(xn) tend vers f(½) : f est constante. Les autres cas se traitent de même. Exercice 36 : Fonctions continues périodiques.

1) Que dire d’une fonction f : R → R continue et admettant pour périodes 1 et 2 ?

2) Que dire d’une fonction f : R2 → R continue et telle que :

Page 42: Exercices équations fonctionnelles

42

∀(x, y) f(x, y) = f(x + 1, y) = f(x, y + 1) = f(x + y, y) ?

3) Que dire d’une fonction f : R2 → R continue et telle que :

∀(x, y) f(x, y) = f(x + 1, y) = f(x, y + 1) = f(2x + y, x + y) ? [ Oral ENS 2003 ]

Solution. Cet exercice porte sur les fonctions périodiques d’une ou deux variables, ou encore sur les systèmes dynamiques sur le cercle et le tore.

1) Si f ∈ C(R, R), l’ensemble G des périodes de f est un sous-groupe additif fermé de R.

S’il contient 1 et 2 , il contient Γ = { a + b 2 ; (a, b) ∈ Z×Z } et son adhérence. Or Γ est dense dans R, donc G = R et f est constante. NB : La densité de Γ découle du théorème de structure des sous-groupes additifs de R : ceux-ci sont

soit monogènes soit denses. Or 2 est irrationnel, donc Γ ne peut être monogène.

2) Je dis que f(x, y) = p(y), où p ∈ C(R, R) est 1-périodique.

On a d’abord f(x, y) = f(x + 1, y) = f(x, y + 1) = f(x + y, y) = f(x + ny, y) pour tout n ∈ Z.

D’où f(x + y , ny

) = f(x, ny

) ; à la limite f(x + y, 0) = f(x, 0). Ainsi x → f(x, 0) est constante.

De plus pour tout (a, b) ∈ Z×Z , f(a + b 2 , 2 ) = f(a, 2 ) = f(0, 2 ).

Comme Γ est dense et f continue, f(x, 2 ) = f(0, 2 ).

Cela reste vrai si l’on remplace 2 par un irrationnel quelconque ω : (∀x ∈ R) (∀ω ∈ R − Q) f(x, ω) = f(0, ω). Par continuité de f et densité de R − Q, on a ∀(x, y) ∈ R×R f(x, y) = f(0, y). p(y) = f(0, y) est continue et 1-périodique. Réciproque facile.

3) Je dis que f est constante.

En effet, la matrice A =

1112 est hyperbolique en ce sens que Sp A = {

253+ ,

253− } .

On a P−1

.A.P =

α

α0

0 , où P =

−+22

5151 et α = 2

53+ , α = 2

53− .

A admet une direction dilatante et une direction contractante.

Soit D = R.(1− 5 , 2) la direction contractante, p l’orthoprojection sur D.

f est constante sur D car f(X) = f(An.x) → f(0, 0) quand n → +∞.

Mais si X ∈ R2, la suite (A

−n.X) « tend vers la droite D » quand n → +∞, en ce sens que d(A

−n.X,

D) = || A−n

.X − p(A−n

.X) || → 0. Comme f est uniformément continue sur R2, car continue et Z

2-

périodique, f(X) = f(A−n

.X) → f(0, 0). Cqfd. Exercice 37 : Trouver les couples ( f , g ) de fonctions continues de R dans R vérifiant : ∀(x, y) f(x + y) = f(x + g(y)) g(x + y) = g(x + f(y)).

Solution : Notons G l’ensemble des périodes de f , H celui de g, ϕ(y) = y − f(y). G et H sont deux sous-groupes additifs fermés de R, donc de la forme R ou aZ. On vérifie que (∀y) f(x + ϕ(y)) = f(x) , donc ϕ(y)∈G. • Si G = R, f est constante égale à a ; g(x + y) = g(x + a), donc g est constante. Réciproque facile. • Si G = aZ (a > 0), en vertu du théorème des valeurs intermédiaires, ϕ est constante. On en déduit

que f est g sont égales à idR. • Si G = {0}, alors (∀y) ϕ(y) = 0, f(y) = y, y = g(y). Réciproque immédiate.

Conclusion : f et g sont constantes, ou bien f = g = idR.

Page 43: Exercices équations fonctionnelles

43

Exercice 38 : Quelles sont les fonctions continues F : R*+2 → R telles que :

∀ x, y > 0 F(x, y) = F( 2

yx+, xy ).

Solution résumée : Notons T(x, y) = (2

yx+, xy ) pour tout couple (x, y) de réels > 0.

On a F(x, y) = F(T(x, y)) = F(Tn(x, y)) pour tout n. Or la suite (T

n(x, y)) converge vers la moyenne

arithmético-géométrique (M(x, y), M(x, y)) de x et y. Donc F(x, y) = F(M(x, y), M(x, y))…

Exercice 39 : Quelles sont les fonctions continues F : R+2 → R telles que :

∀ x, y ≥ 0 F(x, y) = F( y , 2

²² yx +).

Solution : Résoudre le problème de Centrale M 1989…

Exercice 40 : Provuer qu’il existe une unique fonction monotone f : [0, 1] → R vérifiant les

conditions : ∀x ∈ [0, 1] f(x) = 2 f(3x ) , f(x) + f(1 − x) = 1.

Solution : 1) Analyse. Faisons x = 0, il vient f(0) = 0, puis f(1) = 1 ; donc f est croissante.

Faisons x = 1, puis x = 31 ; il vient f(

31 ) = f(

32 ) =

21 ; donc f est constante sur [

31 ,

32 ].

On voit apparaître l’escalier du diable… Problème 41 : Equations fonctionnelles récursives.

1) Soient X un ensemble, h une fonction X → X, E un espace de Banach, A un fermé de E, Φ une fonction X×A → A, telles que :

a) ∃y0 ∈ A Φ(. , y0) est bornée sur X : ∀x ∈ X || Φ(x, y0) || ≤ M ;

b) Les Φ(x, .) sont « équicontractantes » :

∃k ∈ [0, 1[ ∀x ∈ X ∀(y, y’) ∈ A2 || Φ(x, y) − Φ(x, y’) || ≤ k.||y − y’||.

Montrer qu’il existe une unique fonction bornée f : X → A telle que ∀x ∈ X f(x) = Φ(x, f(h(x))). Montrer que si X est un espace métrique, et si h et Φ sont continues, alors f l’est aussi.

2) Montrer qu’il y a une seule fonction continue f : R → R telle que ∀x∈R f(x) = 2

sinx .cos f(2x ).

3) Montrer qu’il existe une unique fonction continue f : [0, 1] → R telle que f(0) = 0, f(1) = 1, et :

f(x) = 2

)3( xf si 0 ≤ x ≤

31 , f(x) =

21 si

31 ≤ x ≤

32 , f(x) =

2)23( −xf

si 32 ≤ x ≤ 1.

Montrer que f est dérivable et de dérivée nulle sur un ouvert dense de [0, 1]. Solution résumée : Soit B(X, E) l’espace des fonctions bornées de X dans E, muni de la norme uniforme. C’est un espace de Banach. B = { f ∈ B(X, E) ; f(X) ⊂ A } en est une partie fermée, donc complète. L’application T : f → F, où F(x) = Φ(x, f(h(x)) pour tout x ∈ X, est contractante de B dans B… Il reste à invoquer le théorème de point fixe… La question 3 conduit à la fonction de Cantor, dite « escalier du diable », déjà rencontrée dans l’exercice précédent.

Exercice 42 : Soit (un) une suite strictement croissante de réels de [0, 1], de limite 1.

Page 44: Exercices équations fonctionnelles

44

Trouver les f ∈ C([0, 1], R) telles que ∀x ∈ [0, 1] f(x) = ∑+∞

=

−+1 2

)1(n

nn xxuf

.

Solution : [ Oral Centrale 2009, RMS n° 775 ]

Notons d’abord que ∀(n, x) 0 ≤ un.x + 1 − x ≤ 1.

Nous allons montrer que f est constante. Soient M = max f(x) = f(a) et m = min f(x) = f(b).

M = f(a) = ∑+∞

=

−+1 2

)1(n

nn aauf

≤ ∑+∞

=1 2nn

M = M , donc ∀n f(un a + 1 – a) = M . A la limite, f(1) = M.

m = f(b) = ∑+∞

=

−+1 2

)1(n

nn bbuf

≥ ∑+∞

=1 2nn

m = m , donc ∀n f(un b + 1 – b) = m . A la limite, f(1) = m.

Ainsi, f(1) = m = M, et f est constante. Réciproque évidente

Remarque : Plus généralement, soit (un) une suite de fonctions [0, 1] → [0, 1] convergeant simple-

ment vers une constante. Les f ∈ C([0, 1], R) telles que ∀x ∈ [0, 1] f(x) = ∑+∞

=1 2))((

nn

n xuf sont les

fonctions constantes.

Exercice 43 : Trouver les fonctions f continues [0, 1] → R telles que ∀x ∈ [0 , 1] f(x) = ∑+∞

=1 2)(

nn

nxf.

Solution : [ Oral X 2005, RMS n° 212, etc. ]

1) Généralités. A toute f ∈ E = C([0, 1], R) associons g définie par ∀x ∈ [0, 1] g(x) = ∑+∞

=1 2)(

nn

nxf.

La série est normalement convergente, g est donc définie et continue. L’opérateur T : f → g est linéaire, continu, et tel que ||| T ||| = 1.

2) Points fixes de T . Montrons que T(f) = f ssi f est constante.

Fixons 0 < a < 1 ; soient b = min { f(x) ; x ∈ [0, a] } et c = max { f(x) ; x ∈ [0, a] }, b, c ∈ [0, a].

Alors ∀x ∈ [0, a] f(b) = ∑+∞

=1 2)(

nn

bf ≤ ∑

+∞

=1 2)(

nn

nbf = f(b) ≤ f(x) ≤ f(c) = ∑

+∞

=1 2)(

nn

ncf ≤ f(c) = ∑

+∞

=1 2)(

nn

cf.

Donc (∀n) f(c) = f(cn) et f(b) = f(b

n). A la limite f(b) = f(0) = f(c) ; f est constante sur [0, a], pour tout

a, donc sur [0, 1] par continuité.

6. Equations intégro-différentielles.

Exercice 1 : Trouver les fonctions f ∈ C(R, R) vérifiant ∀x ∈ R ∫+Tx

xdttf ).( = 0.

Solution : On suppose T > 0. Notant F(x) = ∫x

dttf0

).( , F est de classe C1, T-périodique.

Sa dérivée f est aussi T-périodique et telle que ∫T

dttf0

).( = 0. Réciproquement, si f est continue, T-

périodique et telle que ∫T

dttf0

).( = 0, alors ∀x ∈ R ∫+Tx

xdttf ).( = 0, par Chasles ou par dérivation.

Exercice 2 : Trouver les fonctions f ∈ C(I, R) vérifiant ∀x∈I ∫x

xdttf

2).( = 0 , lorsque I = ]0, +∞[, R.

Solution : 1) Si I = R. F(x) = ∫x

dttf0

).( est de classe C1 sur R, telle que F(2x) = F(x) et F(0) = 0.

Page 45: Exercices équations fonctionnelles

45

Donc F(2nx) = F(x), F(x) = F(x/2

n) → 0 quand n → +∞. Ainsi, F = 0, donc f = F’ = 0.

2) Si I = R*+ , F(x) = ∫x

dttf1

).( est de classe C1 sur I et telle que F(2x) = F(x) et F(1) = 0.

Dérivant, il vient (∀x > 0) 2f(2x) = f(x) et ∫2

1).( dttf = 0.

Réciproquement, si f vérifie ces deux conditions, F(2x) = F(x) pour tout x > 0.

La restriction g de f à [1, 2] est continue, et vérifie g(2) = g(1)/2 et ∫2

1).( dttg = 0 ; elle décrit un sous-

espace de codimension 2 de C([1, 2], R), car ce sont des formes linéaires indépendantes. On peut la prolonger à I via la formule f(2

nx) = f(x)/2

n (∀n∈Z).

Le prolongement répond aux hypothèses. Autre solution : G(t) = F(exp t) est C

1, ln 2 - périodique de R dans R, et telle que G(0) = 0.

Et f(x) = x1 G(ln x). Réciproque facile.

Exercice 3 : Trouver les fonctions f ∈ C(I, R) vérifiant ∀x ∈ I ∫²

).(x

xdttf = 0 ,

lorsque I = [0, +∞[, I = ]0, 1[, I = ]1, +∞[.

Solution partielle :

1) Supposons d’abord I = ]1, +∞[. Soit f ∈ C(I, R) vérifiant ∀x ∈ I ∫²

).(x

xdttf = 0.

Le chgt de variable u = ln t donne ∫x

x

uu dueefln2

ln.).( = 0.

La fonction g(u) = eu

f(eu) est définie, continue sur ]0, +∞[, et telle que ∀x > 0 ∫

x

xduug

2).( = 0.

Nous voilà ramenés à l’exercice précédent.

Exercice 4 : Trouver les fonctions f ∈ C(R, R) vérifiant ∀(x, y) ∈ R2 f(x) − f(y) = ∫

+

+

yx

xydttf

2

2).( .

Solution : [ Oral X PC, 2009 ].

Faisons y = 0 ; il vient : f(x) − f(0) = ∫x

xdttf

2).( . On en déduit que f est C

1, …, C

∞.

Revenons alors à : f(x) − f(y) = ∫+

+

yx

xydttf

2

2).( . Si l’on dérive en x, puis en y, il vient :

f’(x) = f(x + 2y) – 2 f(y + 2x) , puis 0 = 2 f’(x + 2y) – 2 f’(y + 2x) . Comme l’application (x, y) → (x + 2y, y + 2x) est bijective, f’ est constante, et f est affine.

Reportant f(x) = ax + b dans : f(x) − f(y) = ∫+

+

yx

xydttf

2

2).( , il vient, après calculs : a = b = 0.

Conclusion : f est la fonction nulle. Si c’était pour en arriver là…

Exercice 5 : Trouver les fonctions f ∈ C(R, R) vérifiant ∀(x, y) ∈ R2 f(x).f(y) = ∫

+

yx

yxdttf ).( .

Solution : [ Oral X 1989, Mines 2008, etc. ].

1) x = y = 0 donne f(0) = 0 ; x = 0 donne 0 = ∫−y

ydttf )( . Cela implique en dérivant que f est impaire.

2) Ecartons la solution f ≡ 0. Si f(y0) ≠ 0 , f(x) = )(

10yf ∫

+

0

0

)(yx

yxdttf , donc f est C

1, etc., C

∞.

Page 46: Exercices équations fonctionnelles

46

Dérivons f(x).f(y) = ∫+

yx

yxdttf ).( partiellement, en x, et en y, deux fois. Il vient :

f’(x).f(y) = f(x + y) – f(x − y) f(x).f(y) = f(x + y) – f(x − y)

f’’(x). f(y) = f’(x + y) – f’(x − y) , f’(x).f’(y) = f’(x + y) + f’(x − y) , f(x).f’’ (y) = f’(x + y) – f’(x − y)

Il vient f’’(x). f(y) = f(x).f’’(y), donc f’’(x) = )()(''

0

0

yfyf

f(x). Au passage, on note que f’(0) = 0 ou 2.

f est solution d’une équadiff linéaire d’ordre 2, donc f(x) = ax + b , f(x) = a.ch(ωx) + b.sh(ωx) ou f(x) = a.cos(ωx) + b.sin(ωx) (ω > 0).

Mais f est impaire, et f’(0)2 = 2f’(0), donc f’(0) = 0 ou 2. Mais f’(0) = 0 donne f = 0, cas exclu, donc :

f(x) = 0 , f(x) = 2x , f(x) = ω2 sh(ωx) ou f(x) = ω

2 sin(ωx) (ω > 0) . Réciproque facile mais à faire.

Exercice 6 : Trouver les fonctions f ∈ C(R, R) vérifiant ∀(x, y) ∈ R2 f(x).f(y) = ∫

+

yx

yxdttf ).( .

Solution [Oral Mines 1992].

y = 0 donne (∀x) 0 = ∫+x

xdttf ).( . Apppliquant ce résultat à x − y, il vient 0 = ∫

yx

yxdttf ).( , donc :

∀(x, y) ∈ R2 f(x).f(y) = ∫

+

yx

yxdttf ).( . Nous voilà ramenés à l’exercice précédent !

Réponse : f(x) = 0 , f(x) = 2x , f(x) = ω2 sh(ωx) ou f(x) = ω

2 sin(ωx) (ω > 0) .

La réciproque est facile : f étant impaire, pour tout x, 0 = ∫+x

xdttf ).( , donc f(x).f(y) = ∫

+

yx

yxdttf ).( se

déduit de f(x).f(y) = ∫+

yx

yxdttf ).( .

Exercice 7 : Trouver les fonctions f ∈ C(R, R) vérifiant resp. :

∀(x, y) ∈ R2 x ≠ y ⇒

xy−1 ∫

y

xdttf ).( =

2)()( yfxf +

.

∀(x, y) ∈ R2 f(x) ≥ 0 et x ≠ y ⇒

xy−1 ∫

y

xdttf ).( = )().( yfxf .

∀(x, y) ∈ R2 x ≠ y ⇒

xy−1 ∫

y

xdttf ).( = f(

2yx+

) .

Solution : 1) Réponse : les fonctions affines f(x) = ax + b.

Pour tout y , f(y) = y2 ∫

ydttf

0).( − f(0) et f(y) =

12−y ∫

ydttf

1).( − f(1) , donc f est C

1 sur R, etc.

Dérivons y.f(y) + y.f(0) = 2∫y

dttf0

).( . Il vient f(y) + y.f’(y) + f(0) = 2 f(y) , i.e. y.f’(y) + f(0) = f(y).

Redérivons : y.f’ ’(y) + f’(y) = f’(y), donc y.f‘’(y) = 0 . f’’ est nulle sur R*, donc sur R par continuité.

3) Réponse : f est affine. Tout d’abord f(x) = ∫+

2/1

2/1).(

x

xdttf , ce qui montre que f est C

∞.

En dérivant 2y f(x) = ∫+

yx

yxdttf ).( en y, il vient : 2 f(x) = f(x + y) + f(x − y) (Jensen).

Dérivons derechef en y. Il vient : 0 = f’(x + y) – f’(x – y). Donc f’ est constante, et f = ax + b. Réciproque facile.

Page 47: Exercices équations fonctionnelles

47

Exercice 8 : Trouver les fonctions f ∈ C(R, R) vérifiant :

∀(x, y) ∈ R2 ∫

y

xdttf ).( =

6xy−

[ f(x) + 4 f(2

yx+) + f(y) ].

Solution : On reconnaît la formule des trois niveaux, qui est satisfaite par tous les polynômes de degré ≤ 3. Il s’agit de montrer la réciproque…

1ère étape : montrer que f est C∞

. Ce n’est pas simple. Voici une démarche, qui n’est peut-être pas la seule.

L’équation fonctionnelle s’écrit ∀(x, y) ∈ R2 ∫ −+

1

0)).(( dtxyuxf =

61 [ f(x) + 4 f(

2yx+

) + f(y) ].

Notons φ(x, y) = ∫ −+1

0)).(( dtxyuxf . Intégrer cette égalité en y ∈ [a, b]. En déduire que f est C

1, etc.

2ème étape : Dérivons partiellement en y, puis en x. Il vient : 0 = f’(x) – f’(y) + ( y – x ) f’’(2

yx+).

Dérivons en y, puis en x derechef. Il vient : 0 = ( y – x ) f (4)(

2yx+ ).

Du coup f (4)(z) = 0 pour tout z (prendre y = z + 1, x = z – 1).

Ainsi, f est un polynôme de degré ≤ 3.

Exercice 9 : Trouver les fonctions f : R → R vérifiant :

∀(x, y) x ≠ y ⇒ min( f(x), f(y) ) ≤ yx

yfxf−− )()(

≤ max( f(x), f(y) ).

Solution : [Oral Mines 1993]. 1) Supposons d’abord f continue sur R. Faisant tendre y vers x par valeurs différentes, il vient

xyxfyf

−− )()( → f(x). Donc f est dérivable et f’(x) = f(x). Donc f(x) = a e

x. Réciproque facile.

2) Tout revient à montrer que f est continue à droite et à gauche en tout point. Supposons par absurde que xn → x + 0 et que f(xn) ne tende pas vers f(x).

f(xn) aurait une valeur d’adhérence α > f(x) ou α < f(x). Supposons f(xn(k)) → α.

Si α > f(x) , f(x) ≤ )(

)( )()(

kn

kn

xxxfxf

−−

≤ f(xn(k)), donnerait à la limite +∞ ≤ α, donc α = +∞.

D’autre part f(xn(k))(1 + x − xn(k)) ≤ f(x), donnerait à la limite +∞ ≤ f(x) : impossible.

Le cas α < f(x) se traite de même, ainsi que la continuité à gauche.

Exercice 10 : Montrer qu’il y a une unique fonction f ∈ C(R, R) vérifiant ∀x∈R f(x) = ∫ +−x t

dttf

e0

²

.)²(1

.

Que dire de la suite de fonctions (fn) définie par f0(x) = 0 , fn+1(x) = ∫ +−x

n

t

dttf

e0

²

.)²(1

? [ Oral X 2001 ]

Solution . 1) Si f existe, elle est C∞

, f(0) = 0 et f’(x) = )²(1

²

xfe x

+−

. Donc f’(x) + f’(x) f(x)2 = ²xe− .

En intégrant, il vient : 3)( 3xf

+ f(x) = F(x) , où F(x) = ∫ −x

t dte0

². .

G(y) = 3

3y + y est un C

∞-difféomorphisme de R sur R, et f(x) = G

−1(F(x)).

Page 48: Exercices équations fonctionnelles

48

Réciproque facile. Notons de plus que | f(x) | ≤ G−1

(2π ) ; f est bornée sur R.

On peut expliciter f à l’aide des formules de Cardan et de la fonction F, qui est « connue ». Maple fait cela dans la suite.

2) On pense bien sûr au théorème de point fixe de Picard-Banach. Soit E l’espace de Banach des fonctions continues bornées de R dans R muni de la norme uniforme,

et A = { g ∈ E ; (∀x ) | g(x) | ≤ G−1

(2π ) } ; A est une partie fermée, donc complète, de E.

Si g ∈ E, notons (Tg)(x) = ∫ +−x t

dttg

e0

²

.)²(1

. T induit un opérateur contractant de A dans A.

On montre que | Tg(x) − Th(x) | ≤ k.|| g − h ||∞ , où k = 24

2 ππ+ ∈ [0, 1[.

T admet un unique point fixe dans A ; c’est f. De plus, comme T(E) ⊂ A, toute suite f0 ∈ E , fn+1 = T(fn) tend uniformément vers f. Avec Maple : > G:=y->y+y^3/3;plot(G,-3..3,color=maroon,thickness=2);solve(G(y)=x,y); H:=x->1/2*(12*x+4*sqrt(4+9*x^2))^(1/3)-2/(12*x+4*sqrt(4+9*x^2))^(1/3); plot(H,-3..3,color=red,thickness=2);simplify(G(H(x)));

:= H → x − 12

( ) + 12x 4 + 4 9x2( )/1 3 2

( ) + 12x 4 + 4 9x2( )/1 3

x > F:=x->int(exp(-t^2),t=0..x);F(x);f:=x->H(F(x));simplify(f(x)); plot([f(x),H(sqrt(Pi)/2),-H(sqrt(Pi)/2)],x=-3..3,thickness=2);

12

π ( )erf x

:= f → x ( )H ( )F x

Exercice 11 : On définit par récurrence la suite (fn) de fonctions de [0, 1] dans R par :

f0(x) = 1 et fn+1(x) = 1 + ∫ −x

n dtttf0

²).( pour n ∈ N et x ∈ [0, 1].

Propriétés de la suite (fn) : convergence simple et uniforme. Propriétés de la limite ? Réciproque ?

Solution :

1) Transformons la suite en série et posons u0(x) = 1 et un(x) = fn(x) – fn−1(x) pour n ≥ 1.

Il vient un(x) = ∫ −−x

n dtttu0

1 ²).( . Le point décisif est que t – t2 décrit [0, ¼] quand t décrit [0, 1].

12

− ( ) + 6 π ( )erf x 2 + 16 9π ( )erf x 2( )/2 3

4

( ) + 6 π ( )erf x 2 + 16 9π ( )erf x 2( )/1 3

Page 49: Exercices équations fonctionnelles

49

Donc si on note mn = sup { | un(x) | ; x ∈ [0, ¼] } et Mn = sup { | un(x) | ; x ∈ [0, 1] } , il vient

| un(x) | ≤ x mn−1 , d’où l’on déduit Mn ≤ mn−1 et mn ≤ 41 mn−1 .

Par suite mn ≤ 141

−n m1 et Mn ≤ 241

−n m1 .

En conclusion, la série de terme général un(x) est normalement convergente, donc la suite (fn(x)) est uniformément convergente sur [0, 1]. Sa limite f est continue et vérifie

f(0) = 1 et ∀x ∈ [0, 1] f(x) = 1 + ∫ −x

dtttf0

²).( .

Du coup, f est de classe C1 (et même C

∞) et telle que f(0) = 1 et f’(x) = f(x – x

2).

2) Unicité de la fonction f.

Si g est continue sur [0, 1] et vérifie g(0) = 1 et ∀x ∈ [0, 1] g(x) = 1 + ∫ −x

dtttg0

²).( ,

la différence h = f – g vérifie h(0) = 0 et ∀x ∈ [0, 1] h(x) = ∫ −x

dttth0

²).( ,

Du coup, notant m = sup { | h(x) | ; x ∈ [0, ¼] } et M = sup { | h(x) | ; x ∈ [0, 1] } , il vient

| h(x) | ≤ x m , d’où l’on déduit M ≤ m et m ≤ 41 m, donc m = 0, puis M = 0. cqfd.

3) Il resterait à montrer que f est développable en série entière, en étudiant plus finement la suite

des fonctions fn(x) : ce sont des polynômes.

Exercice 12 : Trouver les fonctions f dérivables de R dans R, telles que ∀x ∈ R f’(x) + f(−x) = x.ex.

Solution : Si f est solution, il est facile de voir qu’elle est indéfiniment dérivable.

Du coup, f vérifie l’équation différentielle linéaire : f’’( x) + f(x) = ( x + 1 ).ex − x.e

−x (*)

La résolution de cette équation différentielle donne f(x) = xex.2

− xex −+ .21 + a.cos x + b.sin x.

Mais attention ! les solutions sont à chercher parmi ces fonctions.

Revenant à la condition originelle, on trouve f(x) = xex.2

− xex −+ .21 + a.(cos x − sin x).

Le problème posé était linéaire : ses solutions forment une droite affine incluse dans le plan affine des solutions de (*).

Autre solution : Introduisons la fonction auxiliaire g(x) = f(−x).

L’équation proposée implique g’(x) = f(x) + x e−x

.

Donc elle équivaut au système [ )(')('

xgxf ] =

0110 [ )(

)(xgxf ] + [ x

x

xexe

− ] .

Maple résout très bien ce système, et il n’y a pas de réciproque à faire…

> sys:={diff(f(x),x)=-g(x)+x*exp(x),diff(g(x),x)=f(x)+x*exp(-x)};

:= sys { }, = ∂∂x

( )f x − + ( )g x x eeeex = ∂∂x

( )g x + ( )f x x eeee( )−x

> dsolve(sys,{f(x),g(x)});

= ( )f x + − − + ( )sin x _C2 ( )cos x _C112

x eeee( )−x 1

2eeee

( )−x 12

x eeeex,{

= ( )g x − + − + − ( )cos x _C2 ( )sin x _C112

eeeex 12

x eeeex 12

x eeee( )−x

}

Exercice 13 : Trouver les fonctions f ∈ C2(R, R) telles que ∀x ∈ R f’’( x) + f(−x) = x.e

x .

Solution : f est C∞

. En dérivant deux fois on trouve f (4)

(x) − f(x) = (x + 2) ex + e

−x.

Page 50: Exercices équations fonctionnelles

50

> dsolve(diff(y(x),x,x,x,x)-y(x)=(x+2)*exp(x)+x*exp(-x),y(x));

( )y x38

x eeee( )−x 1

8eeeex x

716

eeeex 18

eeee( )−x

x2 18

x2 eeeex 516

eeee( )−x

_C1 ( )cos x− + − − + − + =

_C2 ( )sin x _C3eeeex _C4eeee( )−x

+ + +

> f:=x->-3/8*x*exp(-x)+1/8*exp(x)*x-7/16*exp(x)-1/8*exp(-x)*x^2+ 1/8*x^2*exp(x)-5/16*exp(-x)+A*cos(x)+B*sin(x)+C*exp(x)+D*exp(-x); diff(f(x),x,x)+f(-x)-x*exp(x);

f x38

x eeee( )−x 1

8eeeex x

716

eeeex 18

eeee( )−x

x2 18

x2 eeeex 516

eeee( )−x

A ( )cos x B ( )sin x− + − − + − + + → :=

C eeeex D eeee( )−x

+ +

− − − + + + + 14

eeee( )−x 1

4eeeex 2 B ( )sin x C eeeex D eeee

( )−xC eeee

( )−xD eeeex

Finalement il vient B = 0 et C + D = −1/4. On trouve un plan affine.

Autre solution : Introduisons la fonction auxiliaire g(x) = f(−x).

L’équation proposée implique g’’( x) = f’’( −x) = − f(x) − x e−x

.

Elle équivaut au système

''''

vguf

=

0001100001000010

vguf

+

− −x

x

xe

xe0

0, que l’on peut confier à Maple…

Mais il semble que Maple se trompe… (à vérifier).

Exercice 14 : Trouver les fonctions f dérivables de R dans R, telles que ∀x ∈ R f’(x) = f(1 − x) .

Solution : Si f est solution, on voit aussitôt que f est indéfiniment dérivable. Du coup, f vérifie l’équation différentielle linéaire : f’’( x) = − f(x) (*) La résolution de cette équation différentielle donne f(x) = a.cos x + b.sin x. Mais attention ! les solutions sont à chercher parmi ces fonctions. Reportant dans l’équation d’origine, et utilisant l’indépendance de sin et cos, on trouve f(x) = a.(cos x + sin(1−x)). Comme dans l’exercice 8, on trouve une droite vectorielle incluse dans le plan des solutions de (*). Variante : Les solutions de (*) s’écrivent aussi f(x) = r.cos(x + ϕ)…

Généralisation aux fonctions vérifiant f’(x) = f(v(x)), où v est une involution de classe C1.

Exercice 15 : Trouver les fonctions f dérivables de R*+ dans R telles que ∀x ∈ R*+ f’(x) = f(x1 ) .

Solution : Même méthode. Cette fois-ci, x2

f’’( x) + f(x) = 0, équadiff linéaire du second ordre du type

d’Euler. On en cherche des solutions de la forme xr, r complexe, et l’on trouve le plan vectoriel

f(x) = x [ a.cos(23 .ln x) + b.sin(

23 .ln x) ] .

Reportant dans l’équation d’origine, on finit par trouver f(x) = a x [cos(23 .ln x) + sin(

23 .ln x)] .

Exercice 16 : Trouver les fonctions f ∈ C(R, R) vérifiant ∀x ∈ R f(x) + ∫ −x

dttftx0

).().( = ax + b .

Solution : On a ∀x ∈ R f(x) + x.∫x

dttf0

).( − ∫x

dttft0

).(. = ax + b .

Cela montre que f est C1, etc. , donc C

∞. Dérivons deux fois, il vient :

Page 51: Exercices équations fonctionnelles

51

f’(x) + ∫x

dttf0

).( = a, puis f’’( x) + f(x) = 0 ; de plus f’(0) = a et f(0) = b.

Donc f(x) = b.cos x + a.sin x. Réciproque facile, directe ou en remontant.

Exercice 17 : Trouver les f ∈ C([0, 2π ], R) telles que ∀x ∈ [0,

2π ] f(x) = ∫ −

2/

0).()cos(

πdyyfyx .

Solution : [ Oral X 2012, RMS n° 217 ] Si l’on développe le second membre, on voit que f est de la forme f(x) = a cos x + b sin x. Et l’on reporte. Le résultat est décevant : f est la fonction nulle ! > f:=x->a*cos(x)+b*sin(x);

:= f → x + a ( )cos x b ( )sin x > S:=f(x)-int(cos(x-y)*f(y),y=0..Pi/2);

:= S + − − − − a ( )cos x b ( )sin x12

a ( )sin x14

a ( )cos x π14

b ( )sin x π12

b ( )cos x

> coeff(S,cos(x));coeff(S,sin(x));

− − a14

a π12

b

− − b12

a14

b π

> solve({coeff(S,cos(x))=0,coeff(S,sin(x))=0},{a,b}); { }, = a 0 = b 0

Exercice 18 : Trouver les fonctions f ∈ C(R, R) vérifiant ∀x ∈ R f(x) = 1 − ∫ −+x

dttxftx0

).().( .

Solution : Soit f ∈ C(R, R) vérifiant ∀x ∈ R f(x) = 1 − ∫ −+x

dttxftx0

).().( .

Le chgt de variable u = x – t donne : f(x) = 1 − ∫ −x

duufux0

).().2( = 1 − 2x∫x

duuf0

).( + ∫x

duuuf0

).( .

Il montre que f est C1, etc. , donc C

∞. Dérivons deux fois, il vient : f’(x) = − x f(x) − 2∫

xduuf

0).( ,

puis f’’( x) + x f’(x) + 3 f(x) = 0 , f(0) = 1 , f’(0) = 0. Maple donne f(x) = ( 1 − x2

).e−x²/2

.

> dsolve({diff(y(x),x,x)+x*diff(y(x),x)+3*y(x)=0,y(0)=1,D(y)(0)=0},y(x));

= ( )y x −eeee( )− /1 2x2

( )− + 1 x2

> f:=x->-exp(-1/2*x^2)*(-1+x^2):simplify(f(x)-1+int((x+t)*f(x-t),t=0..x)); 0

Exercice 19 : Trouver les fonctions f ∈ C(R, R) vérifiant ∀x ∈ R f(x) = ∫3/2

0).(

xdttf .

Solution : Montrons que f = 0. Notons M(x) = sup{ | f(t) | ; | t | ≤ x } pour tout réel x ≥ 0. M est une fonction croissante, tendant vers 0 en 0+ (car f(0) = 0 et f est continue en 0).

Il vient M(x) ≤ 32x M(

32x ) ≤

32x M(x), donc M(x) ≤ (

32x )n

M(x) pour tout n.

Par suite M(x) = 0 si x < 23 , et f(x) = 0 si |x| <

23 .

Par applications répétées de f(x) = ∫3/2

0).(

xdttf , on a f(x) = 0 si |x| < (

23 )

2 , puis |x| < (

23 )

3 , etc.

Page 52: Exercices équations fonctionnelles

52

Exercice 20 : Soit a∈R. Trouver les fonctions f∈C(R, R) telles que ∀x∈R ∫ −1

0).().cos( dttftx = af(x).

Solution : L’équation s’écrit cos x. ∫1

0).(.cos dttft + sin x. ∫

1

0).(.sin dttft = a f(x).

• Si a = 0, on trouve E = { f ∈ C(R, R) ; ∫1

0).(.cos dttft = ∫

1

0).(.sin dttft = 0 } ; c’est un sous-espace

de codimension 2 de C(R, R).

• Si a ≠ 0, f est de la forme f(x) = A.cos x + B.sin x.

Alors A (4

2sin +21 − a) + B

21²sin = 0 , A

21²sin + B (

21 −

42sin − a) = 0.

Si a ∉{21 ±

21 sin 1} , A = B = 0. Si a =

21 ±

21 sin 1, on trouve une droite vectorielle.

Exercice 21 : Trouver les fonctions f ∈ C(R, R) vérifiant ∀x ∈ R f(x) = x + ∫ −x

dttftx0

).().cos( .

Solution : Réponse : f(x) = x + 1 + 33 e

x/2.sin(

23 x) − e

x/2.cos(

23 x) .

Mêmes idées que dans l’ex. 17 ; f est C∞

et vérifie : f(0) = 0 , f’(x) = 1 + f(x) − ∫ −x

dttxtf0

).sin()( ,

f’(0) = 1, et f’’( x) = f’(x) − ∫ −x

dttxtf0

).cos()( = f’’( x) − f’(x) + f(x) = x.

> dsolve({diff(y(x),x,x)-diff(y(x),x)+y(x)=x,y(0)=0,D(y)(0)=1},y(x));

= ( )y x − + + 13

eeee( )/1 2x

sin

12

3 x 3 eeee( )/1 2x

cos

12

3 x 1 x

> f:=x->1/3*exp(1/2*x)*sin(1/2*sqrt(3)*x)*sqrt(3)-exp(1/2*x)*cos(1/2*sqrt(3)*x)+1+x: > simplify(f(x)-x-int(f(t)*cos(x-t),t=0..x));

0 Solution par calcul symbolique de Heaviside, i.e. transformation de Laplace. > with(inttrans) : > laplace(t,t,x);laplace(cos(t),t,x);

1

x2

x

+ x2 1

> solve(F=1/x^2+x/(x^2+1)*F,F);

+ x2 1

x2 ( ) + − x2 1 x

> invlaplace(1/x^2/(x^2+1-x),x,t);

+ − − t 1 eeee( )/1 2 t

cos

12

3 t13

3 eeee( )/1 2 t

sin

12

3 t

Exercice 22 : Trouver les fonctions f ∈ C(R, R) vérifiant ∀x ∈ R f(x) − 2∫ −x

dttftx0

).().cos( = 1.

Solution : [ Oral Mines 2006 ]. Même technique. On trouve en dérivant deux fois f’’ − 2f’ + f = 1, f(0) = 1 , f’(0) = 2. > dsolve({diff(y(x),x,x)-2*diff(y(x),x)+y(x)=1,y(0)=1,D(y)(0)=2},{y(x)});

Page 53: Exercices équations fonctionnelles

53

= ( )y x + 1 2 eeeex x > y:=x->1+2*x*exp(x);simplify(y(x)-2*int(cos(x-t)*y(t),t=0..x));

:= y → x + 1 2x eeeex

1

Exercice 23 : Trouver les fonctions f de classe C1 sur R, telles que :

∀x ∈ R f’(x) + 5∫ −x

dttxtf0

).cos().( = 10 , f(0) = 2 . [ Oral X ]

Solution . La réponse est f(x) = 35x +

31 +

35 cos( )6x +

18625 sin( )6x .

Solution élémentaire : f vérifie f’(x) + 5.cos x. ∫x

dtttf0

.cos)( + 5.sin x. ∫x

dtttf0

.sin)( = 10.

On en déduit que f est C∞

. Et F(x) = ∫ −x

dttxtf0

).cos()( = cos x. ∫x

dtttf0

.cos)( + sin x. ∫x

dtttf0

.sin)(

a pour dérivées F’(x) = f(x) − ∫ −x

dttxtf0

).sin()( et F’’(x) = f’(x) − ∫ −x

dttxtf0

).cos()( .

Donc f’’’ + 6 f’ = 10 , f(0) = 2 , f’(0) = 10 , f’’(0) = −10. Reste à résoudre cette équation, et à vérifier la réciproque. Maple fait cela fort bien : > dsolve({diff(y(x),x,x,x)+6*diff(y(x),x)=10,y(0)=2,D(y)(0)=10, (D@D)(y)(0)=-10},y(x));

= ( )y x + + + 53

( )cos 6 x2518

( )sin 6 x 653

x13

> f:=x->5/3*cos(sqrt(6)*x)+25/18*sin(sqrt(6)*x)*sqrt(6)+5/3*x+1/3: > simplify(diff(f(x),x)+5*int(cos(x-t)*f(t),t=0..x));

10

Solution par calcul symbolique d’Heaviside. On a f’ + 5. f * cos = 10 , f(0) = 2.

La transformée de Laplace F de f vérifie : x.F(x) − 2 + 1²+x

x F(x) = x

10 , d’où F(x) = etc., et utiliser la

transformée de Laplace inverse. Maple fait cela très bien aussi…

Exercice 24 : Soient f , g ∈ C(R, R). Montrer l’équivalence des propriétés suivantes :

(i) ∀x ∈ R g(x) = f(x) + ∫ −x

dttftx0

).().( (ii) ∀x ∈ R f(x) = g(x) − ∫ −x

dttgtx0

).().sin( .

Solution : généralisé dans la RMS 2002-2003, n° 3, p. 185. Il existe une solution élémentaire de cet exercice. Voici une solution par calcul symbolique. Il s’agit de démontrer que g = f + t ∗ f ⇔ f = g – sin ∗ g . Notons F et G les transformées de Laplace de f et g. Il s’agit de vérifier que

G(p) = F(p) + )(²

1 pFp

⇔ F(p) = G(p) − ).(.1²

1 pGp +

Ce qui est immédiat…

Exercice 25 : Trouver les fonctions f ∈ C2(R

2, R) vérifiant identiquement :

f(x, y) = f(y, x) , f(x, y) + f(y, z) + f(z, x) = 3 f(3

zyx ++,

3zyx ++

) .

Solution : 1) Les fonctions solutions forment un sous-espace vectoriel de C2(R

2, R), contenant les

fonctions constantes, mais aussi la fonction (x, y) → x + y.

Page 54: Exercices équations fonctionnelles

54

2) La symétrie de f implique que xf

∂∂

(u, v) = yf

∂∂

(v, u).

Notons h(u) = f(u, u), de sorte que h’(u) = xf

∂∂

(u, u) + yf

∂∂

(u, u) = 2xf

∂∂

(u, u).

Dérivons la seconde relation partiellement en x : xf

∂∂

(x, y) + yf

∂∂

(z, x) = h’(3

zyx ++)

C’est-à-dire xf

∂∂

(x, y) + xf

∂∂

(x, z) = 2 xf

∂∂

(3

zyx ++,

3zyx ++

) (*)

Dérivons ceci partiellement en y :

xy

f∂∂

∂²(x, y) =

32

²²xf

∂∂ (

3zyx ++

,3

zyx ++ ) + 32

xyf∂∂

∂² (3

zyx ++,

3zyx ++ ).

Prenant z = − x – y ; il vient xy

f∂∂

∂²(x, y) =

32

²²xf

∂∂ (0, 0) +

32

xyf∂∂

∂² (0, 0) = cte.

On en déduit, en réintégrant : f(x, y) = axy + ϕ(x) + ψ(y). Comme f(x, y) = f(y, x), ψ = ϕ, donc f(x, y) = axy + ϕ(x) + ϕ(y) , où ϕ ∈ C

2(R, R).

Il en découle xf

∂∂

(x, y) = ay + ϕ’(x).

Reportant en (*), ilvient : a ( y + z ) + 2.ϕ’(x) = 32a ( x + y + z ) + 2.ϕ’(

3zyx ++

) ,

c’est-à-dire : ϕ’(x) = 6a ( 2x − y − z ) + ϕ’(

3zyx ++

) .

Si l’on fait z = − x – y , il vient ϕ’(x) = 2a x + ϕ’(0) , donc ϕ(x) =

4a x

2 + bx + c .

Au final, on trouve les fonctions f(x, y) = 4a ( x

2 + 4xy + y

2 ) + b ( x + y ) + c .

3) Confions la réciproque à Maple : > f:=(x,y)->a/4*(x^2+y^2+4*x*y)+b*x+b*y+c;

:= f → ( ),x y + + + 14

a ( ) + + x2 y2 4 x y b x b y c

> expand(f(x,y)+f(y,z)+f(z,x)-3*f((x+y+z)/3,(x+y+z)/3)); 0

Conclusion : Les fonctions cherchées sont de la forme f(x, y) = 4a ( x

2 + 4xy + y

2 ) + b ( x + y ) + c .

Elles forment un sous-espace de dimension 3 de C2(R

2, R).

Exercice 26 : Trouver les fonctions f C∞

2π-périodiques telles que (∀x) f(2x) = 2.sin(x).f’(x) .

Solution : [ Oral Mines 1994, RMS n° 179 ]

Ecrivons f(x) = ∑+∞

−∞=n

inxn efc ).( , avec convergence normale ainsi que toutes les dérivées.

L’identité f(2x) = 2.sin(x).f’(x) s’écrit : i ∑+∞

−∞=n

nxin efc 2).( = ( eix

– e−ix

) ∑+∞

−∞=n

inxn inefc ).( .

On peut identifier les coefficients puisque ce sont les coefficients de Fourier des deux membres.

Il vient : (n − 1)cn−1(f) − (n + 1).cn+1(f) = cn/2(f) si n est pair , 0 si n est impair.

En écrivant cette condition pour n = 0, 1, −1, 2, −2, etc. il vient :

c−1(f) + c1(f) + c0(f) = 0 , tous les autres coefficients étant nuls.

Ainsi, f(x) = ∑+

−=

1

1

.n

inxn ec , avec c−1 + c1 + c0 = 0. Autrement dit f(x) = a.(1 – cos x) + b.sin x.

Page 55: Exercices équations fonctionnelles

55

Exercice 27 : Trouver les fonctions f de classe C1, 2π-périodiques, à valeurs réelles telles que :

(∀x) 2.f(x + 1) = f(x) + f(2x) .

Solution : [ Oral Mines 1996, RMS n° 262 ]

Ecrivons f(x) = ∑+∞

−∞=n

inxn efc ).( , avec convergence normale.

Donc f(x + 1) = ∑+∞

−∞=n

inxinn eefc .).( , f(2x) = ∑+∞

−∞=n

nxin efc 2).( .

En identifiant les développements en série de Fourier, il vient

2 cn(f).ein

= cn(f) si n impair , 2 cn(f).ein

= cn(f) + cn/2(f) si n pair.

On en déduit, d’abord que cn(f) = 0 si n est impair, puis que cn(f) = 0 si n est pair > 0.

Ce dernier point se montre par récurrence sur k = val2(n) ≥ 1. En résumé, f est constante !

Exercice 28 : Soit f de classe C∞

2π-périodique de R dans C vérifiant : f’(0) = 1 et ∀x ∈ R ∀n ∈ N

| f (n)(x) | ≤ 1 . Montrer que f est la fonction sinus.

Solution : [ Oral Mines 2010, RMS n° 512 ]

7. Fonctions eulériennes, etc.

Exercice 1 : Montrer qu’il existe une unique fonction F : ]−1, +∞[ → R vérifiant :

i) (∀ x ∈ ]−1, +∞[) F(x) = )²1(

1+x

+ F(x + 1) ii) limx→∞ F(x) = 0 . Etudier cette fonction.

Solution : Analyse. En réitérant i), il vient F(x) = )²1(

1+x

+)²2(

1+x

+ … +)²(

1nx+ + F(x + n).

Fixons x. Quand n tend vers +∞, F(x + n) → 0 en vertu de ii). Donc F(x), si elle existe, est la limite

de la suite Fn(x) = ∑= +n

k kx1 )²(1 , i.e. la somme de la série ∑

+∞

= +1 )²(1

n nx. Cela montre l’unicité de F.

Synthèse. Pour tout x > −1, la série ∑+∞

= +1 )²(1

n nx converge, car 0 <

)²(1nx+ ∼

²1n

. Soit F(x) sa somme.

De plus, la convergence est normale sur toute demi-droite [a,+∞[, a > −1, car 0 < )²(

1nx+ ≤

)²(1na+ .

En vertu du théorème d’interversion de limites, limx→∞ F(x) = ∑+∞

=+∞→ +1 )²(

1limn

x nx = 0. cqfd.

Exercice 2 : Montrer qu’il existe une unique fonction F : R*+ → R vérifiant :

i) (∀x > 0) F(x) + F(x + 1) = x1 ii) lim

x→+∞ F(x) = 0 . Etudier cette fonction.

Solution : Analyse. En réitérant i), il vient F(x) = x1 −

11+x

+2

1+x

− … +nx

n

+− )1(

+ (−1)n+1

F(x+n+1).

Page 56: Exercices équations fonctionnelles

56

Fixons x. Quand n tend vers +∞, F(x+n+1) → 0 en vertu de ii). Donc F(x), si elle existe, est la limite

de la suite Fn(x) = ∑= +

−n

k

k

kx0

)1(, i.e. la somme de la série ∑

+∞

= +−

0

)1(

n

n

nx. Cela montre l’unicité de F.

Synthèse. Pour tout x > 0, la série ∑+∞

= +−

0

)1(

n

n

nx converge, car elle obéit au critère des séries alternées.

Soit F(x) sa somme. La convergence est uniforme sur R*+, car |∑+∞

+= +−

1

)1(

nk

k

kx| <

11

++nx ≤

11+n

.

En vertu du théorème d’interversion de limites, limx→∞ F(x) = ∑+∞

=+∞→ +

−0

)1(lim

n

n

x nx = 0.

Mais cela découle aussi du fait que x1 −

11+x

≤ F(x) ≤ x1 . cqfd.

NB : Les fonctions rencontrées dans les exercices précédents se rattachent aux fonctions eulériennes.

Exercice 3 : Montrer qu’il existe une unique fonction S : R*+ → R vérifiant :

i) (∀x > 0) S(x + 1) = x.S(x) − 1 ; ii) S est bornée au V(+∞) .

Montrer que S(x) = e ∑+∞

= +−

0 )(!)1(

n

n

nxn = e dtte xt .

1

0

1∫ −− .

Solution : Analyse. On obtient S(x) = x1 +

)1(1+xx

+ … + ))...(1(

1nxxx ++ +

))...(1()1(nxxx

nxS++

++ en

appliquant i) de façon répétée. Fixons x. Quand n tend vers +∞, ))...(1(

)1(nxxx

nxS++

++ → 0 en vertu de ii).

Donc S(x), si elle existe, est somme de la série ∑+∞

= ++0 ))...(1(1

n nxxx. Cela montre l’unicité de S.

Synthèse. Pour tout x > 0, cette série converge en vertu du critère de d’Alembert. Soit S(x) sa somme. La convergence est normale sur tout [a, +∞[, a > 0, donc S est continue sur R*+ et le théorème d’interversion de limites s’applique : S(x) tend vers 0 en +∞. Pour justifier les deux autres formes de S(x), il suffit de les nommer T(x) et U(x) resp. et de montrer qu’elles vérifient i) et ii). Mais on peut aussi procéder autrement, par décomposition en éléments simples et séries doubles, etc. Exercice 4 : Soit D = {z∈ C ; |z| < 1}. Montrer qu’il existe une unique fonction F : D → C vérifiant :

i) F est continue en 0 et F(0) = 0 ; ii) ∀z ∈ D F(z) = z + F(z2) .

Solution : cf. mes problèmes sur les séries entières.

Exercice 5 : Soit q un complexe tel que |q| < 1. Trouver les fonctions F : C → C vérifiant :

i) F est continue en 0 et F(0) = 1 ; ii) ∀z ∈ C F(z) = ( 1 − qz ).F(qz) .

Solution : En réitérant ii), il vient, pour tout n, F(z) = (1 − qz)(1 − q2z) … (1 − q

nz).F(q

nz).

Fixons z. Quand n tend vers +∞, F(qnz) → F(0) = 1 en vertu de i).

Quant à la suite Pn(z) = (1 − qz)(1 − q2z) … (1 − q

nz), elle converge dans C.

En effet, transformons-la en série : un(z) ≡ Pn+1(z) − Pn(z) = − qn+1

z.Pn(z).

Or |Pn(z)| ≤ )1(1

∏=

+n

k

kzq ≤ exp(∑

=

n

k

kzq

1

) ≤ expq

qz−1

, en vertu de 1 + x ≤ exp x.

Page 57: Exercices équations fonctionnelles

57

Donc |un(z)| ≤ |qn+1z|.exp

qqz−1

. La série ∑+∞

=1

)(n

n zu est absolument convergente, donc convergente, et

la suite (Pn(z)) converge. Ainsi F(z) = lim Pn(z) = ∏+∞

=−

1

)1(k

kzq .

Moreover, la convergence de la série est normale sur tout disque |z| ≤ A, donc F est continue sur C, et en particulier en 0. Elle vérifie de plus ii). Exercice 6 : Fonction de Tschakaloff. Soit q un complexe tel que |q| < 1.

Montrer que T(z) = ∑+∞

=

+

0

2)1(

.n

nnn

zq est la seule fonction définie et continue sur C, telle que :

T(0) = 1 et (∀z) T(z) = 1 + qz.T(qz) .

Solution : cf. mes notes…

Exercice 7 : Soit D = {z∈C ; |z| < 1}. Montrer qu’il existe une unique fonction F : D → C vérifiant :

i) F est continue en 0 et F(0) = 1 ; ii) ∀z ∈ D F(z) = ( 1 + z ).F(z2) .

Expliciter F.

Solution : Voici une solution astucieuse. On note que ( 1 − z ).F(z) = ( 1 − z2

).F(z2).

La fonction G(z) = (1 − z).F(z) vérifie G(z) = G(z2) = … = G(

n

z2

) → 1, donc F(z) = z−1

1 .

Réciproque immédiate. Une autre solution, par produit infini, complète ces résultats, et montrerait

que (∀z ∈ D) z−1

1 = ∏+∞

=+

0

2)1(

k

k

z .

8. Farrago final.

Exercice 1 : Soit X un ensemble. 1) Trouver toutes les fonctions f : X×X → R vérifiant : ∀(x, y, z) ∈ X

3 f(x, z) = f(x, y) + f(y, z).

2) Trouver toutes les fonctions f : X×X → R vérifiant : ∀(x, y, z, t) ∈ X

4 f(x, y) + f(z, t) = f(x, t) + f(z, y).

3) Trouver toutes les fonctions f : X×X → R vérifiant : ∀(x, y, z) ∈ X3 f(x, z) = f(x, y).f(y, z).

Solution : 1) f vérifie f(x, x) = 0, puis f(y, x) = − f(x, y). Du coup, si l’on choisit un élément y0 dans

X, on a f(x, z) = f(x, y0) − f(z, y0). Ainsi, f(x, z) = g(x) − g(z), où g est une fonction X → R. Réciproque facile. Notons que la fonction g n’est pas unique, mais définie à constante près.

2) Noter que ∀(x, y, z, t) f(x, y) − f(x, t) = f(z, y) − f(z, t). La différence f(x, y) − f(x, t) est donc constante en x, et fonction de y et t seuls. Notons f(x, y) − f(x, t) = φ(y, t). Si l’on choisit un élément t0 dans X, on a f(x, y) = f(x, t0) + φ(y, t0). Ainsi, f(x, y) = g(x) + h(y), où g et h sont deux fonctions X → R. Réciproque facile.

Variante : ∀(x, y, z, t) f(x, y) = f(x, t) + f(z, y) − f(z, t) ; a fortiori f(x, y) = f(x, t0) + f(t0, y) − f(t0, t0),

si l’on choisit t0. Il reste à poser g(x) = f(x, t0), h(y) = f(t0, y) − f(t0, t0).

Le couple (g, h) n’est pas unique, mais peut être remplacé par (g + cte, h − cte)

3) Noter que f(x, x) = 0 ou 1. S’il existe x0 dans X tel que f(x0, x0) = 0, f est la fonction nulle. Sinon, f ne prend jamais la valeur 0, et, comme en 1), on montre que f(x, z) = g(x)/g(z), où g est une fonction quelconque X → R* .

Page 58: Exercices équations fonctionnelles

58

Exercice 2 : Trouver les fonctions f : R → R vérifiant ∀(x, y) f(x) + f(y) = f(2

yx+) + f(3x).

Solution : 1) Ces fonctions forment un sous-espace vectoriel de F(R, R), contenant les fonctions constantes. 2) Si f vérifie la condition donnée, alors x = y donne f(x) = f(3x).

Du coup, pour tout (x, y), f(y) = f(2

yx+) ; cela implique f(y) = f(z) pour tout z : f est constante.

Exercice 3 : Trouver les fonctions f : R → R vérifiant ∀(x, y) f(x + ey) = x + e

f(y).

Solution : 1) La fonction f(x) = x est solution. 2) Réciproquement, soit f vérifiant la condition. y = 0 donne, pour tout x : f(x + 1) = x + e

f(0) , donc f(x) = x − 1 + e

f(0) .

f est de la forme x → x + c. Alors ∀(x, y) x + ey + c = x + e

y+c , donc e

y + c = e

y+c .

Dérivons en y : il vient ey = e

y+c , donc c = 0.

Conclusion : la seule fonction vérifiant la condition donnée est f(x) = x.

Exercice 4 : Trouver toutes les bijections f : [0, 1] → [0, 1] vérifiant ∀x ∈ [0, 1] f(2x − f(x)) = x.

Solution : [Oral Centrale 1982, RMS n° 180, Oral INT 1997 ]

Si g est la bijection réciproque de f, on a ∀x ∈ [0, 1] x = 2

)()( xgxf +.

Si x < f(x), on a x < f(x) < f 2(x) < f

3(x) < … , chaque point de cette suite étant le milieu des deux

voisins. Cela contredit le fait que [0, 1] est borné. Idem si f(x) < x. Donc f est l’identité.

Remarque : dans l’exercice de Centrale 1982, on supposait f continue ainsi que sa réciproque ; cette hypothèse ne sert à rien. Exercice 5 : Trouver les fonctions f : R → R vérifiant respectivement :

a) ∀(x, y) ∈ R2 f(x + y) = x.f(x) + y.f(y) .

b) ∀(x, y) ∈ R2 f(x + y) = xy.[f(x) + f(y)] .

c) ∀(x, y) ∈ R2 (x + y).[f(x) + f(y)] = x.f(x) + y.f(y) .

Solution : a) x = y = 0 donne f(0) = 0. Et x = y donne f(2x) = 2xf(x) (*).

On en déduit en reportant f(x + y) = 2

)2()2( yfxf +, c’est-à-dire f(

2yx+ ) =

2)()( yfxf +

.

On en déduit par récurrence que ∀(n, x) ∈ N×R f(nx) = n.f(x). Mais en vertu de (*) 2n f(x) = f(2nx) = 2nx f(nx) = 2n

2x f(x) pour tout n ∈ N.

Cela impose f(x) = 0. En résumé, f = 0.

b) Faire y = 0… Il vient f(x) = 0 !

c) L’équation s’écrit y.f(x) + x.f(y) = 0 pour tout (x, y).

D’où xxf )(

= −yyf )(

pour tous x, y non nuls. Si x = y, il vient xxf )(

= 0, donc f(x) = 0 pour x∈R*.

Faisant x = 1, y = 0, on trouve f(1) + f(0) = 0, donc f(0) = 0. En résumé, f = 0.

Exercice 6 : Soit a un réel. Soit f : R → R telle que, pour tout réel x : f(x + a) = 21 + )²()( xfxf − .

a) Prouver que f est périodique. b) Pour a = 1, donner un exemple d’une telle fonction.

Page 59: Exercices équations fonctionnelles

59

(Olympiades 1968)

Solution : Ah ! 68 ! sous les pavés, la plage ! 1) Supposons que f existe. Alors f(x) – f(x)

2 ≥ 0, donc f(x)∈[0, 1], et f(x + a) ≥ ½ pour tout x, donc f(x)∈[1/2, 1] pour tout x.

Si a = 0, on trouve f(x) = 8154+ , qui est constante, donc périodique.

Si a ≠ 0, il vient f(x+a) – f(x+a)2

= [21 –f(x)]2

, donc f(x+2a) = 21 + |

21 – f(x)| =

21 + f(x) −

21 = f(x).

Conclusion : f est 2a-périodique. 2) Exemple : a = 1.

On veut que (∀x) f(x + 1) = Φ(f(x)), où Φ(y) = 21 + ²yy− .

Φ est une bijection décroissante [½ , 1] → [1, ½]. Choisissons une fonction quelconque g : [0, 1[ → [½ , 1]. Posons f(x) = g(x) sur [0, 1[.

Par appels répétés des formules (∀x) f(x + 1) = Φ(f(x)), f(x − 1) = Φ−1(f(x)) (cf. § 3.2), on fabrique

de proche en proche l’une unique fonction f prolongeant g et satisfaisant la propriété.

Voici un exemple explicite : f(x) = 21 [1 + |sin

2xπ |].

Exercice 7 : Trouver tous les polynômes P à coefficients réels tels que P(0) = 0 et, pour tout réel x :

P(x2 + 1) = (P(x))2

+ 1. (Putnam 1971)

Solution : Le polynôme P(x) = x convient. Montrons que c’est le seul. On a : P(0) = 0, P(1) = 1, P(2) = 2, P(5) = 5, P(26) = 26. Plus généralement soit (un) la suite u0 = 0, un+1 = (un)

2 + 1.

Elle vérifie pour tout n, P(un) = un, et elle est strictement croissante,. Le polynôme P(X) − X a une infinité de racines, donc P(X) = X.

Exercice 8 : Trouver les fonctions f : R*+ → R*+ vérifiant les deux conditions :

i) ∀(x, y) ∈ R*+2 f(x.f(y)) = y.f(x) ii) limx→+∞ f(x) = 0 (Olympiades 1983)

Solution : 1) x = y donne f(x.f(x)) = x.f(x) ; donc, pour tout x > 0, xf(x) est un point fixe de f.

2) Nous allons montrer que le seul point fixe de f est 1. On en déduira que (∀x > 0) f(x) = x1 .

Soit donc a un point fixe de f. x = 1, y = a donne a = a.f(1), donc f(1) = 1. On montre par récurrence sur n que f(a

n) = a

n.

• a > 1 contredirait le fait que f(x) → 0 quand x → +∞.

• a < 1 est également impossible, car x = a1 , y = a donne 1 = f(1) = f(

a1 f(a)) = a.f(

a1 ).

a1 serait donc un point fixe de f > 1, et on est ramené au cas précédent.

3) La réciproque est immédiate. En conclusion (∀x > 0) f(x) = x1 .

Autre solution : • Exploitons l’asymétrie des formules : f(x.f(y)) = y.f(x) et f(y.f(x)) = x.f(y) impliquent f(f(x.f(y))) = f(y.f(x)) = x.f(y). Posons y = 1. Il vient f(f(x.f(1))) = x.f(1), et ce, pour tout x, donc f(f(z)) = z pour tout z > 0. En résumé, f est involutive (donc bijective).

• Montrons que f est un morphisme de groupes R*+ → R*+ , i.e. que f(x.z) = f(x).f(z).

Soit y tel que z = f(y). Alors f(x.z) = f(x.f(y)) = y.f(x) = f(z).f(x).

Page 60: Exercices équations fonctionnelles

60

• Montrons que f est décroissante. Tout d’abord, x > 1 ⇒ f(x) < 1. Car x

n → +∞ ⇒ f(x

n) = f(x)

n → 0 .

Du coup, 0 < x < y ⇒ f(y/x) = f(y)/f(x) < 1 ⇒ f(y) > f(x).

• ln o f o exp est un endomorphisme décroissant de (R, +) [Cauchy !] et involutif.

Donc ln(f(exp(x)) = −x, et f(x) = 1/x pour tout x.

Exercice 9 : Trouver toutes les fonctions f : R+ → R+ vérifiant les trois propriétés :

• ∀(x, y) ∈ R+2 f(x.f(y)).f(y) = f(x + y)

• f(2) = 0 ; • pour tout x ∈ [0, 2[ , f(x) ≠ 0. (Olympiades 1986)

Solution : • x = y = 0 donne f(0) = 0 ou 1. En vertu de iii), f(0) = 1. • Par ailleurs f(x + 2) = f(2).f(x.f(2)) = 0 pour tout x ≥ 0, donc f(x) = 0 pour tout x ≥ 2. On a même f(x) = 0 ⇔ x ≥ 2. • Soit x tel que 0 < x < 2. Posons y = 2 − x. On a 0 < y < 2, donc f(y) ≠ 0.

Mais 0 = f(2) = f(x + y) = f(x.f(y)).f(y). Donc f(xf(y)) = 0, donc xf(y) ≥ 2 et f(y) ≥ x2 .

Ainsi ∀x ∈ ]0, 2[ f(2 − x) ≥ x2 , i.e. ∀x ∈ ]0, 2[ f(x) ≥

x−22 .

• Soit x tel que 0 < x < 2. on a : f()(

2xf

+ x) = f(2).f(x) = 0, donc )(

2xf

+ x ≥ 2, et f(x) ≤ x−2

2 .

Conclusion : f(x) = 0 pour tout x ≥ 2 et f(x) = x−2

2 pour tout x tel que 0 ≤ x < 2 .

Il reste toutefois à vérifier que cette fonction satisfait aux trois conditions de l’énoncé !!! Exercice 10 : Trouver toutes les applications f de R dans R vérifiant, pour tous les réels x et y :

f(x2 + f(y)) = y + f(x)

2. (Olympiades 1992)

Solution : 1) L’identité répond à la question. C’est la seule solution affine, et nous allons voir que c’est la seule solution. 2) Si l’on admet que f(0) = 0, alors il vient, en faisant x = 0, y = 0, puis, en posant z = f(y) :

f(f(y)) = y , f(x2) = f(x)

2 et f(x

2 + z) = f(z) + f(x)

2 = f(z) + f(x

2).

On en déduit en faisant z = − x2, que f est impaire, puis que f(z − x

2) + f(x)

2 = f(z).

Finalement f satisfait l’équation de Cauchy, et est croissante en vertu de f(x2) = f(x)

2 .

Donc f(x) = ax, et comme f(1) = 1 en vertu de f(1) = f(1)2 et f(1) ≠ f(0), f(x) ≡ x.

3) Il reste à montrer que f(0) = 0. Ce n’est pas facile. L’idée est de poser f(0) = a et de bidouiller les formules… Exercice 11 : Montrer qu’il n’existe aucune fonction f : R → R telle que, pour tout réel x, on ait :

f(f(x)) = x2 − 1996. (Tournoi des villes, 1996)

Solution : L’impossibilité va provenir de considérations purement ensemblistes. Notons g(x) = x

2 – 1996 et h = g o g, et cherchons leurs points fixes avec Maple.

> g:=x->x^2-1996;h:=x->expand(g(g(x))):h(x);

:= g → x − x2 1996

− + x4 3992x2 3982020 > solve(g(x)=x,x);fsolve(g(x)=x,x);

Page 61: Exercices équations fonctionnelles

61

, + 12

12

7985 − 12

12

7985

,-44.1794136045.17941360 > solve(h(x)=x,x);fsolve(h(x)=x,x);

, , ,− + 12

12

7981 − − 12

12

7981 + 12

12

7985 − 12

12

7985

, , ,-45.16822137-44.1794136044.1682213745.17941360 > simplify(g(-1/2+1/2*sqrt(7981)));simplify(g(-1/2-1/2*sqrt(7981)));

− − 12

12

7981

− + 12

12

7981

Ainsi, g a deux points fixes, a et b, h a quatre points fixes, a, b, c et d. De plus g(c) = d, et g(d) = c. Si f existe, f commute à g et h. On en déduit aussitôt que : f({a, b}) ⊂ {a, b} et f({a, b, c, d}) ⊂ {a, b, c, d}.

f induit une application de X = {a, b, c, d} dans X laissant stable {a, b}, dont le carré est

cdbadcba

Il est facile de voir qu’une telle application n’existe pas : • f(c) = a ou b impliquerait g(c) = (f o f)(c)∈{ a, b}, f( c) = c impliquerait g(c) = (f o f)(c) = c. • Donc f(c) = d, et de même, f(d) = c. Mézalor g(c) = (f o f)(c) = f(d) = c… Or g(c) = d !

Exercice 12 : Soit t ∈ ]0, 1[. Déterminer les fonctions f : R → R continues en 0 et telles que, pour

tout réel x : f(x) − 2f(tx) + f(t2

x) = x2 . (Olympiades Croatie 1996)

Solution : 1) Approche directe. Additionnons ∑=

++ +−n

k

kkk xtfxtfxtf0

21 ))()(2)(( = ∑=

n

k

kxt0

22 .

Il vient par télescopage : f(x) – f(tx) − f(tn+1

x) + f(tn+2

x) = x2

²11 22

tt n

−− +

; à la limite f(x) – f(tx) = ²1

2

tx− .

Réadditionnons ! ∑=

+−n

k

kk xtfxtf0

1 ))()(( = ∑= −n

k

k

txt

0

2

²1²

. Cela donne f(x) − f(tn+1

x) = ²1

2

tx− ²1

1 22

tt n

−− +

.

A la limite f(x) − f(0) = ²)²1(

2

tx− , donc f est de la forme f(x) = k +

²)²1(

2

tx− .

Réciproque facile, mais à faire.

2) Autre approche, plus adaptée à la nature affine de l’équation proposée :

• Chercher une solution particulière de la forme f(x) = ax. On trouve f0(x) = ²)²1(

2

tx− .

• Par soustraction g = f − f0 doit vérifier g(x) −2g(tx) + g(t2x) = 0. On montre qu’elle est constante

par les techniques de télescopage précédentes. Exercice 13 : Soit f : R → R telle que, pour tout réel x, | f(x) | ≤ 1 et

f(x + 4213 ) + f(x) = f(x +

61 ) + f(x +

71 ) .

Prouver que f est périodique. (Proposé Olympiades 1996)

Solution : 1) Les fonctions f : R → R telles que (∀x) f(x + 4213 ) + f(x) = f(x +

61 ) + f(x +

71 )

forment un sous-espace vectoriel E de F(R, R). En fait E = Ker(∆ o ∆’) = Ker(∆’ o ∆), où :

Page 62: Exercices équations fonctionnelles

62

∆f(x) = f(x + 61 ) − f(x) et ∆’ f(x) = f(x +

71 ) − f(x) .

E contient les fonctions affines (et l’on comprend le rôle de l’hypothèse supplémentare imposée à f),

et les fonctions t-périodiques, où t = 421 , et en fait les fonctions de la forme x.p(x) + q(x), où p et q

sont t-périodiques.

2) Nous allons montrer que si f ∈ E est bornée, elle est 1-périodique.

Tout d’abord, la fonction g = ∆f est 71 -périodique, et a fortiori 1-périodique.

Donc p(x) = f(x+1) − f(x) = ∑=

+−++5

0

)6

()61(

k

kxfkxf = ∑=

+5

0

)6

(k

kxg est fonction 1-périodique et bornée.

Je dis alors que f(x) = xp(x) + q(x), où q est 1-périodique. En effet q(x + 1) − q(x) = f(x + 1) − f(x) − (x + 1)p(x) + xp(x) = 0. Fixons x. Pour tout entier n, f(x + n) = (x + n)p(x) + q(x) est une suite bornée. Cela implique que p(x) = 0, donc f est 1-périodique.

3) Prenons le problème de plus haut. Plaçons-nous dans E = F(R, R), et introduisons l’opérateur de translation T : f → Tf, où Tf(x) = f(x + t). Alors ∆ = T

6 − I , ∆’ = T

7 − I , ∆ o ∆’ = P(T), où P(X) = (X

6 − 1)(X

7 − 1).

Or P(X) divise X84

− 2X42

+ 1 = (X42 − 1)(X

42 − 1) = (X

6 − 1)A(X)(X

7 − 1)B(X).

Ainsi, f(x + 2) – 2f(x + 1) + f(x) = 0, donc f(x) = xp(x) + q(x), etc.

Remarque : J’ai prolongé tout ceci dans un problème. Cf. aussi exercice suivant…

Exercice 14 : Soit a et b deux réels > 0, f une fonction bornée R → R vérifiant : ∀x ∈ R f(x + a + b) + f(x) = f(x + a) + f(x + b). Montrer que f est à la fois a et b périodique.

Solution : La fonction g(x) = f(x + b) – f(x) est a-périodique et bornée. On a, pour tout n ∈ N, par télescopage ou récurrence : f(x + a + nb) – f(x) = n g(x). La suite (n g(x)) est bornée, donc g(x) = 0 et f est b-périodique. Il reste à échanger a et b. Exercice 16 : Existe-t-il deux fonctions f : R → R et g : R → R telles que

(∀x ∈ R) f(g(x)) = x2 et g(f(x)) = x

3 ?

(Proposé Olympiades 1997)

Solution : La réponse est non. Raisonnons par absurde, en supposant que f et g existent. 1) Du fait que x → x

3 est bijective, g est surjective et f injective.

2) f(g(−x)) = x2 = f(g(x)) ; comme f est injective, g est paire.

3) On a, pour tout x : f(g(f(x))) = f(x)2 = f(x

3) et g(f(g(x))) = g(x)

3 = g(x

2).

4) Considérons les trois réels f(0), f(1) et f(−1). Ils sont distincts puisque f est injective. Mais, en vertu de f(x)

2 = f(x

3), ils sont tous trois solutions de l’équation du second degré a

2 = a.

Or une telle équation a deux solutions !

Exercice 17 : Déterminer tous les polynômes réels P tels que, pour tout réel x

(x − 16).P(2x) = 16(x − 1).P(x) (Olympiades Irlande 1997)

Solution : 1) Méthode algébrique. Par le théorème dit de Gauss, X − 16 et X − 2 divisent P(X) (car (X − 32)P(X) = 16(X − 2)P(X/2).)

Page 63: Exercices équations fonctionnelles

63

Posons P(X) = (X − 2)(X − 16)Q(X). Après substitution et simplification, il vient (X − 8)Q(2X) = 4(X − 2)Q(X). Toujours par Gauss, X − 8 et X − 4 divisent Q(X). Posons Q(X) = (X − 4)(X − 8)R(X). Il vient alors R(2X) = R(X). Cela équivaut à R(X) = cte. En effet R(1) = R(2) = R(4) = … , et R − R(1) aurait une infinité de racines. Conclusion : P(X) = a(X − 2)(X − 4)(X − 8)(X − 16) . Réciproque facile, et d’ailleurs déjà faite.

2) Approche plus fonctionnelle.

Itérons la relation P(x) =)32()2(16

−−

xx

P(2x ). Pour n ≥ 5 et x non puissance de 2, il vient :

P(x) = )32()2(16

−−

xx

)64()4(16

−−

xx

…)2()2(16

4+−−n

n

xx

P( nx2

) = )2)(2)(2)(2(

)16)(8)(4)(2.(164321 ++++ −−−−

−−−−nnnn

n

xxxxxxxx

P( nx2

).

Fixons x et faisons tendre n vers +∞ (mise en facteur des termes prépondérants).

Il vient P(x) = 1021 (x − 2)(x − 4)(x − 8)(x − 16)P(0), donc P(x) = a(x − 2)(x − 4)(x − 8)(x − 16).

C’est vrai pour une infinité de valeurs de x, donc partout. Réciproque évidente.

Remarque : Cet exercice relève du q-calcul. La 1ère méthode s’étend aux corps de caractéristique ≠ 2, la seconde méthode relève de l’analyse.

Exercice 18 : Trouver les fonctions f : R → R telles que, pour tous réels x et y :

f(x − f(y)) = 1 – x – y. (Olympiades de Slovénie 1999)

Solution : Prenons y = 0, a = f(0). Il vient f(x – a) = 1 – x, donc f(x) = 1 – x – a : f est affine, de la forme f(x) = c − x. Un simple calcul donne f(x) = ½ – x. Exercice 19 : Déterminer toutes les fonctions monotones f : R → R telles que, pour tout réel x :

f(f(f(x))) − 3.f(f(x)) + 6.f(x) = 4x + 3. (Olympiades de Roumanie 1999)

Eléments de solution : 1) On peut chercher f affine f(x) = ax + b. On trouve f(x) = x + 1. 2) L’idée est de considérer la suite (f

n(x)) des itérés de x par f…

On trouve fn(x) = n + A(x) + 2

n[B(x).cos3πn + C(x).sin

3πn ].

Soient x et y des réels tels que x < y. Alors pour tout n, fn(x) ≤ f

n(y), donc

A(x) − A(y) ≤ 2n [(B(y) − B(x)).cos

3πn + (C(y) − C(x)).sin

3πn ].

Un moment de réflexion montre que les fonctions B et C sont constantes. On conclut facilement.

Exercice 20 : Déterminer toutes les applications f : R → R telles que :

∀(x, y, z, t) ∈ R4 (f(x) + f(z)).(f(y) + f(t)) = f(xy − zt) + f(xt + yz). (Olympiades 2002)

Solution : 1) Recherche de solutions.

• Les solutions constantes f = c vérifient 4c2 = 2c, donc c = 0 ou ½.

• S’ajoute la fonction f(x) = x2, en vertu de l’identité de Diophante :

(x2 + y

2)(u

2 + v

2) = (xu – yv)

2 + (xv + yu)

2.

Cette identité est valable dans un anneau commutatif quelconque ; dans le cas réel, elle équivaut à

∀(z, z’) ∈ C×C |zz’|2 = |z|

2.|z’|

2.

Nous allons montrer que les trois fonctions f(x) ≡ 0, f(x) ≡ ½, f(x) ≡ x2 sont les seules solutions de

l’équation fonctionnelle proposée.

Page 64: Exercices équations fonctionnelles

64

2) Premières propriétés de f.

• x = y = u = v = 0 donne 4f(0)2 = 2f(0), donc f(0) = 0 ou ½.

• x = y = u = 0 donne 2f(0)( f(0) + f(v) ) = 2f(0). Si f(0) = ½, alors f(v) = ½ pour tout v. Supposons désormais f(0) = 0. • y = v = 0 donne f(x)f(u) = f(xu) ; x = u = 0 donne f(y)f(v) = f(−yv) Ainsi ∀(x, y) f(xy) = f(x)f(y) = f(−xy) (M). • On en déduit f(1)

2 = f(1), donc f(1) = 0 ou 1. f(1) = 0 implique f(x) = f(1)f(x) = 0 pour tout x.

Supposons désormais f(1) = 1. • u = v = 1 donne ∀(x, y) 2( f(x) + f(y) ) = f(x + y) + f(x – y) (F). • x = u, y = −v donne ∀(x, y) ( f(x) + f(y) )

2 = f(x

2 + y

2) (C).

• Il découle de (M) ou de (F), que f est paire.

3) Etude de l’équation fonctionnelle (F). Cette équation est bien connue : cf. pb 3 en fin de chapitre • (F) implique par récurrence sur n, (∀n ∈ N) (∀x ∈ R) f(nx) = n

2f(x).

• Par parité de f, (∀n ∈ Z) (∀x ∈ R) f(nx) = n2f(x).

• Enfin, (∀r ∈ Q) (∀x ∈ R) f(rx) = r2f(x), car si r = qp

(p, q) ∈ Z×N*, q2f(rx) = f(px) = p

2f(x).

• Il en découle que (∀r ∈ Q) f(r) = r2f(1) = r2.

NB : On ne peut conclure sans hypothèse additionnelle, car si g satisfait à l’équation de Cauchy, f(x) = g(x

2) vérifie (F) ; or il bien des fonctions g.

4) Montrons que f est croissante sur R+. Cela va découler de (M) et de (C). En effet (M) implique f(x

2) = f(x)

2, donc u ≥ 0 ⇒ f(u) ≥ 0. Par parité, f est positive sur R.

De plus, soit a ≥ b ≥ 0, y = b , x = ba− . (C) donne f(a) = (f(x) + f( b ))2 ≥ f( b )

2 = f(b).

Soit alors x un réel ≥ 0, (rn) et (sn) deux suites adjacentes de rationnels positifs tendant vers x.

On a f(rn) = rn2 ≤ f(x) ≤ f(sn) = sn

2 , donc, par les gendarmes : f(x) = x

2. Conclure par parité.

5) Solution alternative, par méthode d’Euler.

De 4) il découle que f est croissante sur R+, décroissante sur R−. Elle est donc réglée sur tout segment. Intégrons (F) en y ∈ [0, 1]. Il vient :

2f(x) = ∫ +1

0).( dyyxf + ∫ −

1

0).( dyyxf − 2∫

1

0).( dyyf = ∫

+1).(

x

xduuf + ∫ −

x

xduuf

1).( − 2∫

1

0).( dyyf .

On en déduit que f est continue, de classe C1, puis de classe C

∞.

En dérivant en x, puis en y, il vient 0 = f’’(x + y) − f’’(x − y) : f’’ est constante. Donc f(x) = ax

2 + bx + c. Mais f(0) = 0, f(1) = 1 et f est paire…

Exercice 21 : Déterminer toutes les fonctions f : R → R telles que, pour tout réel x :

f(x3 + x) ≤ x ≤ f(x)

3 + f(x).

(Crux mathematicorum, 2003)

Solution : La fonction P(x) = x3 + x est une bijection strictement croissante de R dans R.

Soit g la bijection réciproque. • (∀x) f(P(x)) ≤ x équivaut, en posant y = P(x), à (∀y) f(y) ≤ g(y).

• (∀x) x ≤ P(f(x)) équivaut, par stricte croissance de g, à (∀y) g(y) ≤ f(y). En définitive, f = g. Réciproque déjà faite ! > with(plots):plot([x^3+x,x,x=-1.5..1.5],color=blue);

Page 65: Exercices équations fonctionnelles

65

Exercice : expliciter g.

Exercice 22 : Déterminer tous les polynômes f à coefficients réels tels que, pour tous réels a, b, c vérifiant ab + bc + ca = 0, on ait : f(a − b) + f(b − c) + f(c − a) = 2f(a + b + c). (Olympiades 2004)

Solution :

1) E = {P∈R[X] ; ∀(a, b, c)∈R3 ab + bc + ca = 0 ⇒ P(a − b) + P(b − c) + P(c − a) = 2P(a + b + c)}

est un sev de R[X], qui contient X2 et X

4, donc leurs combinaisons linéaires. En effet :

> q:=n->factor((x-y)^n+(y-z)^n+(z-x)^n-2*(x+y+z)^n); > q(2);q(4);

− − − 6 x y 6 y z 6 zx

−6 ( ) + + x y y z z x ( ) + + + + + 2 y2 x y y z 2 z2 z x 2 x2

2) Montrons la réciproque. Soit P ∈ E. • a = b = c = 0 donne P(0) = 0. • b = c = 0 donne P(a) + P(−a) = 2P(a) ; P est pair. N’ayant pu trouver de preuve algébrique élégante, je renvoie à P. Bourgade pour la suite. ____________

Bibliographie

G. Valiron : Equations fonctionnelles (Masson) D. Knuth : Fundamental algorithms (Addison-Wesley) J.-M. Monier : Cours de taupe (Dunod) R. Antetomaso : RMS déc. 1989 , avril 1990 (p. 433), janvier 1992, 2002 n° 3 (p. 185) M. Aassila : Olympiades internationales de mathématiques 1998-2002 (Ellipses) P. Bourgade : Olympiades internationales de mathématiques 1976-2005 (Cassini) L.-G. Vidiani : Note sur les équations fonctionnelles. P. Bornsztein et M. Omarjee : Equations fonctionnelles (Saint-Malo, 29 juillet 2003) H. Pépin : L’équation fonctionnelle f o f + a.f + b.Id = 0 (RMS octobre 2008) B.M. Makarov, M.G. Goluzina, A.A. Lodkin, A.N. Podkorytov :

Problèmes d’analyse réelle (Cassini, 2010)

Problèmes :

XP’ 1978 : L’équation fonctionnelle ϕ(ψ(x)) = ϕ(x) + 1.

Concours général 1979 : L’équation fonctionnelle f(x) = f(21+x ) + f(

21−x ).

Centrale M 1982 : L’équation fonctionnelle f’(x) = f(x2).

XM’ 1989 et XM’ 2006 : L’équation fonctionnelle f’(x) = f(λx).

Page 66: Exercices équations fonctionnelles

66

XP’ 1993, 1ère composition : L’équation fonctionnelle f(x + 1) − f(x − 1) = λ f’(x). Mines 1994 : Equation fonctionnelle f’(x) = λ.f(x + 1) Agrégation interne 1996 : Propriétés spectrales d’un opérateur fonctionnel. Centrale M 1996 : L’équation fonctionnelle F(x) − λ.F(x + a) = f(x). Capes 2002, 1ère épreuve : L’équation fonctionnelle f(x) = x f’(x/2). ENS Ulm Cachan 2003 : Entropie mathématique.

___________


Recommended